You are on page 1of 301

General 

History  Print

Techniques of History Taking

The most difficult part of the examination is time maintenance. The examination must be
completed in 15 minutes. You must make the optimum use of your time.  It is important that
you use appropriate transition sentences and ask open­ended questions as noted below.

Introduction and greeting:

"Hello Mr. XYZ. Good morning or good afternoon. I am Dr. XYZ. It's nice to meet you
(shake hand).

Technique one (HPI last method):

Starting with the past medical history first allows one to get the patient’s background history
and gives clues for what to focus on in the HPI.

Chief complaint: Start by asking the patient why they are here today.

“Can you please tell me what we can do for you today?”

Plan for interview:

"Before we continue, I would first like to get a little background information
about you so that I know what to focus on with your present problem.”

Start with past medical history

“What medical conditions has a doctor diagnosed you with in the past, such as
diabetes or high blood pressure?”

*Tip: It is best not to use medical words such as hypertension but simple
English, such as high blood pressure, to keep the interview as a fluid
conversation.

“Have you been hospitalized for any medical problems in the past?”
"Have you seen a psychiatrist before or been diagnosed with any psychiatric
conditions?”

Female patients

"How many pregnancies have you had?”
“How many children do you have?”
“Were they natural deliveries?”
“Were there any complications?”

Past Surgical History

“What operations have you had in the past?”

Medications
“What medications are you taking that a doctor has prescribed for you?”
“What vitamins or supplements are you taking over­the­counter that a doctor has
not prescribed for you?”

Allergies

“What medications are you allergic to?”
“What allergic reaction does the medicine cause?”
“Are you allergic to anything in the environment or any food?
“What reaction do you get?”

Family history:

*Use transition sentences ­ “I would like to ask you about your family’s medical
conditions.”
“How old is your father? What medical conditions does (or did) he have?”
“How old is your mother? What medical conditions does (or did) she have?”
"How many brothers do you have? Do any of them have any medical
conditions?”
"How many sisters do you have? Do any of them have any medical conditions?”

Social History:

*Transition sentence: “I would like to now ask you about your living situation.”

“Who do you live with at home?”
"What kind of work do you do?”

Tobacco use

“Do you smoke?”
If yes, “what age did you start smoking?”
“How much do you smoke and for how many years?”

Alcohol intake

“Do you drink alcohol or beer?”
If yes, “how many drinks do you have at one time when you drink?”
“What age did you start drinking?”

*Tip: Always keep in mind about the CAGE questionnaires for suspected
alcohol abuse cases (ex. upper GI bleeding, right upper quadrant pain,
epigastric pain).

"Have you ever tried to cut down on alcohol drinking?"
"Have you ever been annoyed by other people for your drinking?" or
"Have you ever annoyed other people by your drinking?"
"Have you ever had guilty feelings about your alcohol drinking?"
"Do you drink alcohol early in the morning?"

Illegal drug use
“Have you used any illegal drugs in the past?”
“What did you use?”
“When did you start?”
“When did you stop?”
“Have you had any blood transfusions? Have you had any tattoos?”

History of present illness (HPI)

*Transition sentence ­ “Now that I have your background, l will now focus on
what brings you in today.”
After this, ask open ended questions: “Tell me more about the chest pain.”

Ask the following components of the chief complaint in the HPI

Location

“Show me where the problem is?”
“Point with one finger where it hurts.”

Onset and duration­ Always ask for the onset and duration of the problem so
you can know whether the problem is acute, subacute, or chronic.

“When did it first start?”
“How long does it last?”

Intensity­ One should always ask about the intensity or severity of the problem,
especially if the complaint is pain.

"On a scale of 1­10, with 1 being the least painful and 10 being the most
painful, which number would describe your pain?" or "How would you
grade your pain on a scale of 1­10?"

Quality

“Describe the pain.”
“What does it feel like?”
“Is it sharp?”

Frequency

“How many times a day are you having the symptom?”
“Is it constant or happening on and off?”

Radiation

“Does it move anywhere?”

Aggravating factors

“What makes it worse?”

Alleviating factors
“What makes it better?”

Associated symptoms

“I want to ask you about other symptoms that you may be having.”

*Tip: Ask the relevant associated symptoms with the chief complaint

General Review of Systems (ROS) ­ brief overview

General:

“Do you have fever or chills?”
“Have you lost any weight?”

Skin:

 “Have you had any rashes or changes in skin color?”
“Have you noticed any easy bruising?”

HEENT:

“Do you have any vision or hearing problems?”
“Do you have any headache or dizziness?”
“Do you have any nasal discharge or obstruction?”
“Do you have any difficulties swallowing?”
“Have you had any sore throat?”

Cardiac:

“Do you have any chest pain?”
“Do you feel that your heart races or goes fast?”

Pulmonary:

“Have you had any cough? If so, are you coughing anything up?”
“What color is the sputum? Is there any blood?”
“Have you had any shortness of breath?”

Gastrointestinal:

“Have you had any nausea or vomiting?”
“Have you had any constipation or diarrhea?”
“Have you had any abdominal pain?”

Genitourinary:

“Have you had to go to the bathroom more often?”
“Does it burn when you urinate?”
"Do you have blood in the urine?”

Musculoskeletal:
“Do you have any weakness in your legs or arms?”
“Do you have any pain in your joints?”

Neurologic:

“Do you have numbness or tingling?”
“Have you passed out at all?”

Technique 2 (HPI first method)

This is the same as above except starting with the HPI first and then proceeding with the past
medical history onwards. The Review of Systems would be performed at the end of the
history.

Summary and plan of action

In both techniques, always summarize the history with the patient. Standardized
patients are usually on the lookout for this.
Always list a plan of action. Standard patients many times will ask for this or look
for you to explain this.
“Let me make sure that I have the correct details. You were well until 5 days ago when
you started having fever, chills, cough with yellow sputum, and shortness of breath.
You started taking ibuprofen and Mucinex without relief. I am concerned about the
following things that could be going on­ pneumonia, bronchitis, etc… I would like to do
the following next steps to help figure out what could be going on.”

This copy of the material is licensed to Mohammed  Mirza.
Copyright © 2015. All Rights Reserved http://www.uworld.com
Complete ROS  Print

How to take history of complete Review of Systems

General:

“Do you have fever or chills?”
“Have you lost any weight?”
“Have you had any changes in your appetite?”
“Have you noticed any weakness in any part of the body?”
“Do you feel tired?”
“Have you had any sweats that drench your clothes or the sheets at night?”
“Have you been exposed to any radiation in the past?”

Skin:

“Have you had any rashes or changes in skin color?”
“Have you had increased itching in any part of the body?”
“Have you had bumps develop on your skin, known as hives?”
“Have you developed bruises more easily than usual?”
“Have you had eczema before?”
“Is your skin drier than usual?”
“Have you had any changes in your hair?”
“Have you had any changes in your nails?”

HEENT:

Head:

“Do you have headaches?”
“Do you have any dizziness?”
“Have you passed out?”
“Have you had any injury to your head, such as falls?”

Eyes:

“Do you have any vision problems, such as double vision or blurry vision?”
“Have you been having more watery eyes?”
“Do you have pain in your eyes?”
“Are your eyes more sensitive to light recently?”
“Have you had any redness or discharge from your eyes?”
“Have you had any injury to your eyes?”

Ears:

“Have you had any hearing loss or difficulties?”
“Do you have pain either inside or on the outside of the ear?”
“Have you had any discharge from your ears?”
“Do you have ringing in your ears or hear an unusual sound?”

Nose:
“Do you have any nasal discharge?”
“Are you having nosebleeds?”
“Do you have any nasal blockage?”
“Have you had any injury to your nose?”
“Do you have any history of sinus infections?”

Throat and mouth:

“Have you had bleeding from your gums?”
“Have you had any sore throat?”
“Do you have any changes in your voice?”
“Have you had drainage in your throat?”
“Do you have any difficulties swallowing?”

Neck:

“Have you noticed any lumps in your neck?”
“Do you have any neck pain when you move it?”
“Does your neck hurt when it is touched?”

Cardiac:

“Do you have any chest pain?”
“Do you feel that your heart races or goes fast?”
“Do you get short of breath when you walk? If so, how far can you walk before
becoming short of breath?”
“Do you get short of breath when you lay down to sleep?”
“Do you wake up in the night with shortness of breath that improves after you sit up for
10­15 minutes?”
“Have you had rheumatic fever in the past?”

Pulmonary/Chest:

“Have you had any cough? If so, are you coughing anything up?”
“What color is the sputum?”
“Are you coughing up any blood? If so, how much blood are you coughing up?”
“How much are you coughing up?”
“Does it hurt in your chest when you take a deep breath?”
“Have you had any shortness of breath?”
“Have you been tested for TB before?”
“Have you had the BCG vaccine for TB?”

Vascular:

“Do you have pain in your legs, calves, or thighs while walking?”
“Have you had swelling in your legs?”
“Do you have varicose veins?”
“Have you had any blood clots in your legs before?”
“Do your arms or legs feel cooler than the rest of the body?”
“Have you had hair loss on your legs?”
“Have you had color changes in your arms or legs?”
“Have you had skin ulcers in your legs or arms before?”
Breasts:

“Have you noticed any lumps in either breast?”
“Have you had any discharge from the nipples?”
“Do you have pain in either breast?”
“Have you noticed any skin color changes?”

Gastrointestinal:

“Have you been more thirsty than usual?”
“Have you had any nausea or vomiting? If so, are you throwing up blood?”
“Have you had any constipation or diarrhea?”
“Have you had pain after eating meals or at night?”
“Have you had any belly pain?”
“Do you have any blood in your stool?”
“Has your stool become thinner recently?”
“Does your stool appear black?”
“Has your belly become bigger?”
“Have you had any pain with bowel movements?”
“Have you had yellow eyes or skin?”

Psychiatric:

“Have you had difficulty falling asleep or staying asleep at night?”
“Have you noticed a change in your mood recently? If so, do you think that it is more
depressed than usual?”
“Have you seen or heard things that others do not see or hear?”

Endocrine:

“Do you feel warm if everyone else in the room appears comfortable?”
“Do you feel cold if everyone else in the room appears comfortable?”

Immunology:

“Do you have any seasonal allergy symptoms to the environment?”
“Have you had any recent infections?”

Hematological/Lymphatic:

“Have you noticed any lumps or bumps anywhere on the body?”
“Have you been bleeding more easily than usual?”

Urinary:

“Have you had to go to the bathroom more often?”
“Do you feel like you have to go more often but do not urinate very much?”
“Does it burn when you urinate?”
“Do you have blood in the urine?”
“Do you have difficulty starting your urination?”
“Have you had difficulty controlling your urine such that you leak during the day or
night?”
“Do you have any pain in the side of your lower back?”
“Do you wake up from sleep to urinate at night? If so, how many times?”
“Are you unable to empty your urine?”
“Have you had kidney stones in the past?”

Male genitalia:

“Have you developed any sores on your penis? If so, are they painful?”
“Have you had any discharge from your penis?”
“Have you had any pain in the penis, scrotum, or testicles?”
“Have you had any prostate problems in the past?”
“Have you been treated for any sexually transmitted diseases in the past? If so, which
ones and what treatment did you receive?”
“Do you have any difficulty with sexual relations?”
“Have you had a hernia before?”

Female Genitalia:

“Have you had any sores or growths on the vaginal area?”
“Have you had any itching in the vaginal area?”
“Have you had any vaginal discharge?”
“Do you have any problems with sexual relations, such as pain?”
“What age did you start having periods?”
“How often are your periods?”
“How many days do your periods last?”
“How many pads or tampons do you use each day and on the heaviest day of your
periods?”
“When was your last period?”
“Have you had any bleeding between periods?”
“Have you had any irregular periods?”
“Have you had excessive bleeding than usual during your periods?”
“Do you have excessive pelvic pain during your periods or before your periods?”
Symptoms of menopause
“Have you stopped having periods? If so, when?”
“Do you have episodes when you feel hot but everyone else feels comfortable in
the room?”
“Have you or others around you noticed any changes in your mood?”
“Have you had any bleeding since your periods stopped?”
“Have you had pain with intercourse?”
“Do you have any vaginal dryness?”

Musculoskeletal:

“Do you have any weakness in your legs or arms?”
“Have you had any stiffness in your muscles?”
“Do you have any stiffness when moving any of your joints?”
“Do you have any pain in your joints?”
“Have you had any swelling in your joints?”
“Have you been diagnosed with arthritis?”
“Do you have any back pain? If so, where?”
“Do you have any muscle cramps?”
Neurologic:

“Do you have numbness or tingling in any part of the body?”
“Have you passed out at all?”
“Have you noticed a tremor in any part of your body?”
“Have you had any problems with your memory?”
“Do you have any difficulty with your speech?”
“Have you had problems with walking?”
“Has anyone noted any changes in your thinking or behavior?”
“Do you feel that you don’t know where you are or become lost?”

This copy of the material is licensed to Mohammed  Mirza.
Copyright © 2015. All Rights Reserved http://www.uworld.com
OBGyn History  Print

Obstetric and Gynecology History: (for female patients and OB/Gyn complaints)

"When was your last menstrual period?"
"Are/Were your cycles regular?"
"How old were you when you had your first period?"
"Are your periods regular?"
"How many days does your period last?"
"Have you ever bled between cycles?"
"How many pads or tampons do you use each day and on the heavy days during your
period?"
"Do you have abdominal cramps/pain before or during your period?"
"Did you ever notice any bleeding after intercourse?"
"Have you been getting regular Pap smears? When was your last Pap smear?"
"Have you ever had any vaginal discharge?"
If YES, then ask "What is the color of the discharge?”
“Does the discharge have any bad odor?”
“Do you have any vaginal itching?"
"Have you had any sores or infections around the vagina?"
“How many pregnancies have you had?”
“How many children have you had?”
“Were they natural deliveries or C­sections?”
“Were there any complications during the deliveries?”

This copy of the material is licensed to Mohammed  Mirza.
Copyright © 2015. All Rights Reserved http://www.uworld.com
Sexual History  Print

Sexual History

This can be asked here as well as in special situations, if pertinent. Before taking the history,
let the patient know that you will be asking about her/his sexual history, so you will be posing
a transition question.

"Ok Mr. Brown, now I would like to ask a few questions about your sexual history.
Please understand that it will be kept confidential between you and me. Try to be as
honest as possible. Is that ok with you?" Continue as follows: "Are you sexually
active?"
If 'YES,'
"Who is your sexual partner?”
“Do you have any other sexual partners?"
"Do you have male, female, or both as partners?”
“Are you satisfied with your sexual life?"
If 'NO,' inquire the reason.
"Do you have any problems in your sexual life?”
“Any loss of interest in sex?”
“Are you able to reach an orgasm?"
"Do you use any means of contraception?"
If 'YES,' "What type of contraception do you use?”
“Do you use it regularly?”
For high risk groups, like patients who are not using barrier methods of contraception,
patients with multiple sexual partners, and patients with a homosexual history, continue
with the following questions: (Note: most of the time they have this history, so never
miss it.)
"Have you ever been tested/treated for sexually transmitted diseases?"
"Have you ever been tested for HIV?" 

This copy of the material is licensed to Mohammed  Mirza.
Copyright © 2015. All Rights Reserved http://www.uworld.com
Psychiatry History  Print

Ask what brings the patient in today.
Ask  what  the  patient  thinks  the  problem  could  be  due  to  (This  would  give  you  a
concrete  answer  if  the  psychiatric  manifestations  were  due  to  reactive  causes.  Be
prepared for a negative answer in most of the cases.)
Ask if the patient has anybody to talk to when she is in distress. (Support systems)
Ask  if  the  patient  has  had  any  unusually  traumatic  episodes  during  the  past  few
months or in the remote past. (PTSD)
Ask about any changes in appetite.
Ask  about  any  changes  in  sleep  patterns,  i  e,  problem  falling  asleep;  problem
maintaining  sleep;  problem  with  early  morning  awakening;  Ask  also  about
nightmares and dreams.
Ask about any weight loss/gain.
Ask about the daily routine of the patient. ("Could you describe to me a typical day
in your life?")
Ask about the patient's interests and hobbies.   Ask if  they  give  her  the  same  kind
of pleasure that they gave her earlier.
Ask about her mood most of the day.
Ask about the duration of these symptoms.
Ask  if  the  patient  is  frequently  forgetting  things  or  feels  that  she  is  losing  her
memory
Ask  the  patient  about  her  general  attitude  towards  life.  ("Do  you  tend  to  look  at
things In a positive frame of mind or in a negative frame of mind?")
Ask about the patient's sexual life.
Ask if the patient has ever considered ending her life.
Ask if the patient has any plans regarding how to end her life.
Ask if there are pills or guns at home.
Ask about family life and the affinity of the patient towards her family members.
Ask about any excessive coffee intake.
Ask  about  drugs,  alcohol,  and  recreational  drugs.  If  yes  then  ask  about  the  last
time  that  the  patient  took  these  drugs.  (The  psychiatric  manifestation  may  be  due
to a withdrawal syndrome.)
Ask  about  any  delusions  or  hallucinations.  ("Do  you  listen  and  see  what  others
can't?  Have  you  heard  voices  talking  to  you  only  and  nobody  else  around  you
does?")
Ask about heat or cold intolerance and other thyroid manifestations.
Do  a  Mini  Mental  Status  Examination.  (  Look  up  the  CNS  history  taking  for  the
MMSE )
Ask if the patient realizes that she has a problem.
Ask if the patient is willing to get help.

Challenging psychiatric situations

"The  Silent  Patient":    If  the  patient  is  not  answering  you,  stay  silent  for  a  minute;
establish eye contact. Put a hand on her shoulder and say, "I know that this is very
hard  for  you.  Will  you  share  it  with  me?  I  am  here  to  help  you.  We  can  do  it
together."
"The  Over­  talkative  Patient":If  you  are  not  able  to  get  a  word  in  edgeways,  stop
the  patient  and  firmly  say,  "I  know  that  all  these  things  bother  you  but  my
number1 priority right now is you. So lets talk about you for now."

Sample documentation of a psychiatric patient note

Patient disheveled
Speech: scant, goal directed
Mood: dysthymic
Affect: mood congruent
Memory: Recent and remote: Intact
Delusions and hallucinations: none
MMSE results
Concentration:
Suicidal intent
Abstraction and Judgment
Insight
Duration of symptoms
Precipitating factors
Thyroid: not palpable 

This copy of the material is licensed to Mohammed  Mirza.
Copyright © 2015. All Rights Reserved http://www.uworld.com
Special Situations  Print

Special Situations

Angry patient:

"Mr. XYZ, you seem to be very angry. Could you please tell me why that is so? Is there
any way that I can help you?"

Uncooperative patient:

"Mr. XYZ, to properly understand your problem, I have to do this test. It won't take
more than a minute. I am here to assist you, ok?"

Pain in hand:

"Does your job involve repetitive hand movements like key board operation?" (carpal
tunnel syndrome).

Insect bite:

"Do you remember being bitten by any insects like ticks and/or mosquitoes?" (in any
rash case)

Trauma patient:

Sometimes you will see trauma patients with serious injuries, bruises, or gunshot
wounds. Avoid painful maneuvers while diagnosing their injuries. Also, be aware that
some severely injured patients without insurance will try to refuse expensive treatments.
For example, a trauma patient with significant injury to the chest who has signs and
symptoms of hemothorax may say he doesn't want to have a chest x­ray. In a case such
as this one, explain that, "We have a social worker. She will help with the financial
details. Right now we must take an x­ray in order to diagnose your condition.” Almost
every hospital in the US will have a social worker to deal with these kinds of problems.

Over­talkative patient:

Sometimes the patient may talk endlessly about irrelevant topics. If so, respond like
this, "Excuse me Mr. XYZ, sorry to interrupt you. I know these things have really been
bothering you. We will try to get back to this again. However, let me first focus on your
present problem."
Some patients will respond normally, but some patients will say, "Are you interrupting
me?" (Don't worry they have been told to act like that). Say the same thing again and
say sorry once again.

General:

If you have to say, "I don't know," say, "I don't know yet" or, "I don't know but I'll find
out and will let you know."

Note: This is a general way to take case histories. We have included questions for general
history taking, as well as questions for specific health issues. You do not need to ask all of
these questions for every case. Prioritize what you need to know and ask those questions. The
ability to prioritize becomes easier the more cases you practice.

This copy of the material is licensed to Mohammed  Mirza.
Copyright © 2015. All Rights Reserved http://www.uworld.com
Physical Examination  Print

The aim here is to teach you how to examine a patient in the examination room.
Always wash  your  hands  with  soap  before  the  physical  examination.  So as not to
lose  time,  while  washing  your  hands,  turn  your  head  toward  the  patient  and
continue your conversation.
Never  examine  through  the  gown.    Always  keep  the  patient's  body  covered  or
draped as much as possible. Expose only the part being examined.
Don't hesitate  to  examine  the  heart  in  a  female  patient.  Leaving  the  bra  on,  ask
her to lift her breast for PMI (Point of maximum impulse) and auscultation.
It  is  permissible  to  place  the  stethoscope  down  between  the  breasts  to  continue
listening. Don't do a breast examination.
The dictum is to inform the patient what you are doing during the examination.

Ex: Start like this:

You:    "Ok,  Mr.  Samson  now  I  would  like  to  do  a  physical
examination on you. For that, I need to untie your gown. Is that
ok with you?" 
SP:  "Yes, it's ok"
You: "Thank you"

If  the  patient  resists  you,  then  you  have  to  explain  the  importance  of  the
examination to understand the disease process. Something like this:

You:  "Mr.  Samson,  I  can  understand  your  feelings  but  to  clearly  understand
the disease I have to do this examination. It won't take more than a minute.
Don't worry. I'm here to assist you. I will tell you whatever I'm going to do."

Some physical  findings  can  be  expressed  on  the  SP's  face  (faked).  So,  be  careful
and always observe SP's face while examining him. Sometimes, you may get a SP
with hypertrophic tonsils, as a case of a sore throat.
Always do a focused, relevant examination. You won't have time to do everything.
Prioritizing will come only with repeated practice.

Respiratory Examination:

Ideally, the patient should be sitting on the end of an exam table.

Inspection:
You: "Now, I would like to examine your chest. Is that OK with you?"

Observe the rate, rhythm, depth, and effort of breathing. 
Listen for obvious abnormal sounds with breathing such as wheezes.
Observe  for  retractions  and  use  of  accessory  muscles  (sternocleidomastoids,
abdominals).
Observe  the  chest  for  asymmetry,  deformity,  or  increased  anterior­posterior  (AP)
diameter. 

Palpation:
You: "I would like to palpate your chest. Let me know if it hurt"

Look for tracheal deviation
Identify any areas of tenderness or deformity by palpating the ribs and sternum. 
Assess expansion and symmetry of the chest by placing your hands on the patient's
back and thumbs together at the midline. Ask him to breathe deeply.
Check for tactile vocal fremitus. 
Look for PMI  (Point of Maximum Impulse).

Percussion:
You: "Now, I am going to tap on your lungs to look for any fluid or excess air"

Percuss both anterior and posterior chest from side to side then from top to bottom

Interpretations:

If dull­ Pleural effusion or Lobar pneumonia
Hyperresonant ­ Pneumothorax or Emphysema

Auscultation:
You:  "Now, I would like to listen to your lungs. Can you take some nice big deep breaths
for me?"

Use the diaphragm of the stethoscope.
Auscultate  anterior,  posterior  and  lateral  sides  of  the  chest  from  side  to  side  then
from top to bottom.

Tactile Fremitus:
You: "Can you please say "ninety­nine, ninety­nine" repeatedly for me"

Palpate with your lateral border of palm.
Increased tactile fremitus suggests consolidation of lung.

Bronchophony:
You: "Can you please say "ninety­nine, ninety­nine" repeatedly for me?"

This  time  instead  of  palpation  you  do  auscultation.  Auscultate  several  symmetrical
areas over each lung.
The  sounds  you  hear  should  be  muffled  and  indistinct.  Louder,  clearer  sounds  are
called bronchophony (indicates consolidation).

Whispering pectoriloquy:
You: "Can you please whisper "ninety­nine, ninety­nine" repeatedly for me?"

Auscultate several symmetrical areas over each lung.
In  normal  people  you  should  hear  only  faint  sounds  or  nothing  at  all.  If  you  hear
the  sounds  clearly  this  is  referred  to  as  whispering  pectoriloquy  (indicates
consolidation).

Special Points:

Never percuss or auscultate on scapula.
Let  the  patient  fully  exhale  or  inhale  before  auscultating  (shifting  to)  the  other
area.
For every  case  of  the  respiratory  system,  don't  forget  to  auscultate  the  heart  and
check for PMI. 

Cardiovascular System:

Always examine  from the patient's right side.
Always examine in 3 positions.

You:  "OK  Mr.  Samson,  now  I  would  like  to  examine  your  heart.    First,  in  a  sitting
position, then lying down, and finally in a sitting position again."
 
While the patient is in sitting position:
You: "I need to check your pulse."

Note whether the pulse is regular or irregular.
Observe for carotid pulse.
Auscultate  for  carotid  bruits  (a  sign  of  arterial  narrowing  and  risk  of  a  stroke)  in
middle aged or elderly patients.
Check for dependent edema.
Don't  measure  the  blood  pressure  unless  otherwise  asked  (they  usually  give  the
B.P).

Then ask the patient to lie down:
You:  "I  need  to  check  your  heart.  Could  you  please  lie  down  for  me?  I'll  pull  out  the  leg
extension to make it a little more comfortable for you"

Look for Jugular Venous Distention (JVD). Position the patient supine with the head
of the table elevated to 30 degrees.
Inspect for precordial movement.
Then palpate for precordial activity.
You may feel thrills or exaggerated ventricular impulses.
Palpate for the point of maximal impulse (PMI or apical pulse).
Note the location, size, and quality of the impulse.

Auscultation:
You: "Now, I would like listen to your heart."

Position the patient supine with the head of the table slightly elevated.
Record S1, S2, (S3), (S4), as well as the grade and configuration of any murmurs
("two over six" or "2/6", "pan systolic" or "crescendo").
Listen  with  the  stethoscope  diaphragm  at  the  right  2nd  intercostal  space  near  the
sternum (aortic area).
Listen at the left 2nd intercostal space near the sternum (pulmonary area).
Listen  at  the  left  3rd,  4th,  and  5th  inter  costal  spaces  near  the  sternum  (tricuspid
area).
Listen at the apex (mitral area).  Next, ask the patient to turn on his left side and
then listen to the mitral area.
Record the rate, rhythm, and any extra sounds you hear.
Ask the patient to sit up again .
Again auscultate all the 4 areas.
Ask him to lean forward and listen at the base of heart.

Examination of the Abdomen:

The patient should be lying supine on the exam table and appropriately draped.
Watch the patient's face for signs of discomfort during the examination.
Consider  the  inguinal/rectal  examination  in  males  and  pelvic/rectal  examination  in
females.
Don't do these examinations but tell the SP that you want to do them later.

Inspection:
You: "I am looking at your belly for any gross abnormalities".

Look for scars, swellings, and hernias.
Look for any visible peristalsis or pulsations.
Note the abdominal contour (Flat, scaphoid, or any abdominal distention).

Auscultation:  
You: " Now, I would like to listen to your bowel sounds"

It is very important to auscultate prior to palpation

1.  Listen for bowel sounds. 
2.  Listen for bruits over the renal arteries, iliac arteries, and aorta.

Percussion:  
You: "Now, I would like to tap on your belly."

Percuss all four quadrants.
Percuss the liver span

1.  Percuss  downward  from  the  chest  in  the  right  midclavicular  line  until
you detect the top edge of liver dullness.
2.  Percuss upward from the abdomen in the same line until you detect the
bottom edge of liver dullness.  
3.  The distance between the points is the liver span.

Check for splenic dullness

1.  Percuss  the  lowest  costal  interspace  in  the  left  anterior  axillary  line.
This area is normally tympanic.
2.  Ask  the  patient  to  take  a  deep  breath  and  percuss  this  area  again.
Dullness in this area may be a sign of splenic enlargement. 

Palpation: 
You:  "Now,  I  would  like  to  press  on  your  belly  first  lightly  then  deeply.  Please  let  me
know if it hurts"

Start  with  light  (superficial)  palpation.  This  is  to  look  for  any  areas  of  tenderness.
Look  at  the  patient's  face  not  at  the  abdomen.  The  most  sensitive  indicator  of
tenderness  is  the  patient's  facial  expression.  Also  look  for  guarding  (Voluntary  or
involuntary).
Proceed to deep palpation after light palpation.  
Palpate  the  Liver:  Place  your  fingers  just  below  the  right  costal  margin  and  press
firmly. Ask the patient to take a deep breath.  
Palpate the aorta: If you suspect any aneurysms
Palpate the spleen:

Special Tests:
Rebound Tenderness:

This is a test for peritoneal irritation.
Tell  the  patient  what  you  are  about  to  do.  "Now,  I  would  like  to  press  in  and
release. Please let me know if it hurts when I press in or let go."
Press  deeply  on  the  abdomen  with  your  hand.  After  a  moment,  quickly  release
pressure.
If it hurts more when you release, the patient has rebound tenderness.

Costovertebral angle (CVA) Tenderness:

CVA tenderness is often associated with renal disease.
Tell the patient what you are about to do. "Now, I would like to give few thumps on
your back. Please let me know if it hurts."

1.  Ask the patient to sit up.
2.  Use  the  heel  of  your  closed  fist  to  strike  the  patient  firmly  over  the
costovertebral angles.
3.  Compare the left and right sides.

Shifting dullness:

Can be done for fluid in the abdomen.

Psoas sign:

This  is  a  test  for  appendicitis.  First  ask  the  patient  to  turn onto  his  left  side.  Then
extend  the  patient's  right  leg.  This  will  cause  stretching  of  the  psoas  muscle  and
will cause pain if it is irritated by inflamed appendix. (SP can act exactly like a real
patient, so watch his face.)

Obturator sign:
This is also a test for appendicitis.

1.  Raise the patient's right leg with the knee flexed.
2.  Rotate the leg internally at the hip.
3.  Increased abdominal pain indicates a positive obturator sign.

Murphy's sign:
You:  "Now, I  would  like  to  press  on  your  belly.  Please  take  a  deep  breath  and  let  me
know if it hurts."

This  is  for  suspected  cholecystitis.  Hook  the  finger  of  your  right  hand  underneath
the  right  costal  margin  at  the  point  where  the  lateral  border  of  the  rectus  muscle
intersects with the costal margin.  Then ask the patient to take a deep breath. If he
complains of pain then the test is +ve.

Neurologic Examination:

Always consider left to right symmetry.
Organize your thinking into seven categories:

1.  Mental Status
2.  Cranial Nerves
3.  Motor system
4.  Coordination and Gait
5.  Reflexes
6.  Sensory system
7.  Special Tests

Mental Status:

 Level of Consciousness: You: "Hello Mr. Swanson. Can you hear me? If you hear

me, can you smile for me?"
Orientation  in  time,  place  and  person:  You:  "Where  are  we?  What  is  the  date
today? Can you please tell me your full name?"
Assess  Judgment:  You:  "What  would  you  do  if  you  found  a  self­addressed
envelope on the street?"

You have to test for memory of recent, remote, and registration.

  Test memory:  Recent  remote  registration.  You:  "Mr.  Swanson,  I'm  going  to  say

three  words.  I  want  you  to  say  them  after  me.  I  will  ask  you  to  repeat the same
words again after 5 minutes.  Do you understand?   Say, "pen, car, and chair". Ask
again  at  the  end  of  the  mental  status.  For  long­term  memory  you  can  ask
something like, "What happened on September 11th?"
 Abstraction: You: "Can you please spell the word, "WORLD" backwards for me?"

Three  Object  Command: "Mr.  Swanson,  I  am  going  to  write  a  sentence.  Please
read the sentence and do as it says. Ok?" You write, 'Close your eyes' on a piece of
paper.    (Some  people  do  the  serial  '7's  test,  but  it  is  time  consuming.    We  don't
advise  that.  If  you  do  the  above  six  things,  that  is  more  than  sufficient  for  this
exam.)

Cranial nerves:
Observe for any gross abnormalities like

Ptosis (III).
Facial droop (or) asymmetry (VII).
Abnormal articulation of words (V, VII, X, XII).
Abnormal eye position (III, IV, VI).
Abnormal or asymmetrical Pupils (II, III).

Olfactory:

Usually not tested for this exam.

Optic:

You: "Now, I need to examine your eyes. Can you please count my fingers?"
Examine the optic fundi.
Test visual acuity by using Snellen's chart or simply by using your fingers.
Screen visual fields by confrontation.
Stand two feet in front of the patient and have them look into your eyes. Hold your
hands  about  one  foot  away  from  the  patient's  ears,  and  wiggle  a  finger  on  one
hand.  Ask  the  patient  to  indicate  which  side  they  see  the  finger  move.  Repeat  two
or three times to test both temporal fields. If an abnormality is suspected, test the
four quadrants of each eye while covering the other with the patient's hand.
Test pupillary reactions to Light.

1.  Dim the room lights as necessary.
2.  Ask the patient to look into the distance.
3.  Shine a bright light obliquely into each pupil in turn.
4.  Look  for  both  the  direct  (same  eye)  and  consensual  (other  eye)
reactions.
5.  Record pupil size and any asymmetry or irregularity.

Oculomotor:

Observe for ptosis.
Test extra ocular movements.
Ask  the  patient  to  follow  your  finger  with  their  eyes  without  moving  their
head.
Check gaze in the six cardinal directions using a cross or "H" pattern.
Pause during upward and lateral gaze to check for nystagmus.
Check  convergence  by  moving  your  finger  toward  the  bridge  of  the  patient's
nose.
Test pupillary reactions to light.

Trochlear:

Test extra ocular movements (Inward and downward movement).  

Trigeminal:

Test motor strength.
Ask the patient to clench his/her teeth.
Palpate the temporal and masseter muscles as they do this.
Test the 3 divisions for pain sensation.
Explain what you intend to do.
Use  a  suitable  sharp  object  to  test  the  forehead,  cheeks,  and  jaw  on  both
sides.
Substitute a blunt object occasionally and ask the patient to report "sharp" or
"dull."
Test the corneal reflex (Not required for this exam).

Abducens:

Test extra ocular movements (Lateral movement).  

Facial:

Observe for any facial droop.
Ask patient to do the following, note any lag, weakness, or asymmetry:
Raise eyebrows ("Can you please raise your eyebrows for me?").
Close  both  eyes  to  resistance  ("I  will  try  to  open  your  eyes,  don't  let  me
open.").
Smile ( "Can you please smile for me?").
Frown  ("Can you please frown for me?").
Show teeth ("Can you please show me your teeth?").
Test the 'corneal reflex' (Not required for this examination).

Acoustic:

Screen hearing
Test hearing by making simple noise with fingers. 
If abnormal, then do Weber and Rinne tests. 
Test for lateralization (Weber)
Use a 512 Hz tuning fork.
Place the base of the vibrating tuning fork firmly on top of the patient's head.
Ask the patient where the sound appears to be coming from (normally in the
midline).
Compare air and bone conduction (Rinne)
Use a 512 Hz tuning fork.
Place  the  base  of  the  vibrating  tuning  fork  against  the  mastoid  bone  behind
the ear.
When the patient no longer hears the sound, hold the end of the fork near the
patient's ear (air conduction is normally greater than bone conduction).
Vestibular function is not normally tested.

Glossopharyngeal:

Not required for this examination.

Vagus:

Listen to the patient's voice, is it hoarse or nasal?
Ask the patient to swallow.
Ask the patient to say "Ah".
Watch the movements of the soft palate and the pharynx.
Test  'gag  reflex'  (unconscious/uncooperative  patient).  (Not  required  for  this
examination.)

Accessory:

From behind, look for atrophy or asymmetry of the trapezius muscles.
Ask  the  patient  to  shrug  his  shoulders  against  resistance  ("Can  you  please  shrug
your shoulders for me?").
Ask  the  patient  to  turn  his  head  against  resistance.  Watch  and  palpate  the
sternomastoid muscle on the opposite side.  

Hypoglossal:

Listen to the articulation of the patient's words.
Ask patient to stick out his tongue. Ask him to move his tongue from side to side.
("Can you please stick out your tongue? Move it from side to side.")

Motor :
Observation:

Involuntary movements
Muscle symmetry and atrophy. Always compare both (right and left) sides.  Check if
there is any difference between the proximal and distal muscle groups.
Gait

Muscle tone:
You: "I would like to check your muscle tone. Please relax your muscles. OK?"

Flex and extend the patient's fingers, wrist, and elbow.
Flex and extend patient's ankle and knee.
There is normally a small, continuous resistance to passive movement.
Observe for decreased (flaccid) or increased (rigid/spastic) tone.

Muscle Strength: 
You: "Now I would like to check your muscle strength/power".

Test strength by having the patient move against your resistance.
Always compare one side to the other.
Grade strength on a scale from 0 to 5 "out of five": 

0/5 No muscle movement at all  
1/5 Just flicker of movement.
Movement  at  the  joint,  but  not
2/5
against gravity  
Movement  against  gravity,  but
3/5
not against added resistance  
Movement  against  resistance,
4/5
but less than normal  
5/5 Normal strength
 
Test the following:
Spine: 

  "I would  like  to  check  the  flexibility  of  your  spine.  Please  lean  over  at  the  waist,

and try to touch your toes."

Upper limb:

Flexion at the elbow ("Can you please pull in?")
Extension at the elbow ("Can you please push out?")
Flexion and Extension at the wrist. ("Can you please pull up and push down?")
Squeeze  two  of  your  fingers  as  hard  as  possible.  ("Can  you  please  squeeze  my
finger as tightly as you can?")
Finger abduction (ulnar nerve) ("Can you please spread your fingers out?")
Opposition of the thumb (median nerve).

Lower limb:

Flexion at the hip (iliopsoas): ("Can you please pull in?")
Adduction at the hips (adductors): ("Can you pull your thighs together?")
Abduction at  the  hips  (gluteus  medius  and  minimus):  ("Can  you  please  push  your
thighs apart?")
Extension at the hips (gluteus maximus): ("Can you please push your thighs out?")
Extension at the knee (quadriceps): ("Can you please kick out?")
Flexion at the knee (hamstrings): (" Can you please pull in?")
Dorsiflexion at the ankle: ("Can you please pull in?")
Plantar flexion (S1): ("Can you please push down?")

Coordination and gait:
Cerebellar Function Tests:
You: "Now I would like to check your balance and movements"

Rapid  alternating  movements  i.e.  ask  him  to  do  rapid  supination  and  pronation  of
hands.
Point­to­Point Movements: "Can you please touch your finger to my finger and then
to your nose". Ask the patient to touch your index finger and their nose alternately
several times. Move your finger about as the patient performs this task.
Ask the patient to place one heel on the opposite knee and run it down his shin to
his  big  toe.  Repeat  with  the  patient's  eyes  closed.  (If  you  do  the  above  test  you
don't need to do this one.)

Romberg's' test:
You: "Can you please stand up with your feet together? Close your eyes. Don't  worry,  I
won't let you fall".

Be prepared to catch the patient if they are unstable.
Ask  the  patient  to  stand  with  his  feet  together  and  eyes  closed  for  5­10  seconds
without support.
The  test  is  said  to  be  positive  if  the  patient  becomes  unstable.  (It's  a  test  for
sensory ataxia not for cerebellar ataxia.)

Gait:
You: "Can you please walk a few steps for me?"

Ask  the  patient  to  rise  from  a  sitting  position,  walk  across  the  room,  turn,  and
come back. ('Get Up and Go Test')
Ask the patient to walk in a straight line: "Can you please walk in a straight line for
me?"

Reflexes:
You:  "Miss  CSA,  I  would  like  to  check  your  jerks.  First,  let  me  begin  with  your  arms.
Then, let me check your legs. Please relax your muscles."
Deep Tendon Reflexes:

The patient must be relaxed and positioned properly.
Reflexes  can  be  reinforced  by  asking  the  patient  to  perform  isometric  contraction
of other muscles (clenched teeth): ("Can you please clench your teeth").
Reflexes should be graded on a 0 to 4 "plus" scale:

0 Absent
Hypoactive
1+  (diminished)

2+  "Normal"
Exaggerated
3+ 
without clonus
Exaggerated  with
4+ 
clonus

Biceps (C5, C6).
Triceps (C6, C7).
Brachioradialis (C5, C6).
Abdominal (T8, T9, T10, T11, T12).
Knee (L2, L3, L4).
Ankle (S1, S2) .

Clonus:

If the reflexes are exaggerated, test for ankle clonus.
Plantar response (Babinski) : Stroke with a blunt object along the lateral border of
the  feet.  Extension  of  the  big  toe  with  fanning  of  fingers  is  abnormal.  This  is
referred to as a positive Babinski.

Sensory:
You: "Now I would like to check your sensory perception  in different areas of your body."
General:

Explain each test before you do it.
Patient's eyes should be closed during the testing.
Compare symmetrical areas on the right and left side.
Also compare distal verses proximal.

Vibration:

Use a low­pitched tuning fork (128Hz).
Test  with  a  non­vibrating  tuning  fork  first  to  ensure  that  the  patient  is  responding
to the correct stimulus.
Place the stem of the fork over the distal interphalangeal joint of the patient's index
fingers and big toes.
Ask the patient to tell you if he feels the vibrations.
If vibration sense is impaired proceed proximally

1.  Upper limb: From wrists, elbows etc:
2.  Lower limb: From medial malleoli, patella, anterior superior iliac spine.

Position Sense:

Grasp the patient's big toe and show the patient "up" and "down."
With the patient's eyes closed ask the patient to identify the direction you move his
toe.
If position sense is impaired move proximally to test other joints.
Test the fingers in a similar fashion.
If vibration, position sense, and subjective light touch are normal in the fingers and
toes you may assume the rest of this exam will be normal.

Pain:
You: "Please understand, this is sharp and this is dull. Now, close your eyes and tell me
know which is which"

Use a suitable object to test "sharp" and "dull" sensations.

Test the following areas:

Shoulders (C4).
Inner and outer aspects of the forearms (C6 and T1).
Thumbs and little fingers (C6 and C8).
Front of both thighs (L2).
Medial and lateral aspect of both calves (L4 and L5).
Little toes (S1).

Temperature:

Often omitted if pain sensation is normal.

Light touch:

Use a fine wisp of cotton or your fingers to touch the skin lightly.
Ask the patient to respond whenever he feels something touching him.

Test the following areas:

Shoulders (C4)
Inner and outer aspects of the forearms (C6 and T1)
Thumbs and little fingers (C6 and C8)
Front of both thighs (L2)
Medial and lateral aspect of both calves (L4 and L5)
Little toes (S1)

Discrimination:
Since  these  tests  are  dependent  on  touch  and  position  sense,  they  cannot  be  performed
when the tests above are clearly abnormal.
Two  point  discrimination  ­  Using  two  sharp  pins,  touch  the  pad  of  the  patient's  finger  in
two  places  simultaneously.  Alternate  with  a  one­point  touch.  The  distance  between  the
two points should be <5 mm on the finger pads.
 
Special tests:

Kerning's Sign: Here the patient lies on his back. Flex both knee and hip then try to
extend  the  knee  only.  In  meningitis,  patient  will  complain  of  pain  or  resistance  to
knee extension.
Brudzinski's  Sign:  When  you  flex  the  neck,  there  may  be  a  flexion  of  the  patient
hips and knees. This is also seen in meningitis patients.
Neck  stiffness:  Have  the  patient  lie  on  his  back. Place  your  hand  behind  his  neck
and  flex  it  until  his  chin  touches  his sternum.    (In  meningitis  he  will  complain  of
neck pain and you will notice resistance to motion.)

Examination of the Head and Neck:
Head:

Look for scars, lumps, rashes, hair loss, or other lesions
Look for facial asymmetry, involuntary movements, or edema.
Palpate to identify any areas of tenderness or deformity.

Eyes:
You: "Now, I need to examine your eyes".

Test the visual acuity by using the Snellen's chart.

Inspection:

Ask  the  patient  to  look  up  and  pull  down  both  lower  eyelids  to  inspect
the  conjunctiva  and  sclera.  Note  any  discoloration,  redness,  discharge,
or lesions. Note any deformity/lesions of the iris and cornea.
Extra  ocular  muscles:  "I'm  going  to  check  your  eyes.    Please  look  at
that object while I shine this light in your eyes."

Corneal Reflections:

Shine a light from directly in front of the patient.
The corneal reflections should be centered over the pupils.
Asymmetry suggests extra ocular muscle pathology.
Test for extra ocular movements.
Test visual fields.
Look pupillary reactions to light.

Ophthalmoscopic exam:

Darken the room as much as possible.
Adjust  the  ophthalmoscope  so  that  the  light  is  no  brighter  than
necessary.
Use  your  left  hand  and  left  eye  to  examine  the  patient's  left  eye.  Use
your right  hand and right eye to examine the patient's right eye.
Ask the patient to look at a point on the wall.
Examine retina and fundus and note any abnormalities.

Ears:

Inspect the auricles and move them around gently. Ask the patient if this is painful.
Palpate the mastoid process for tenderness.
Hold the otoscope then pull the ear upwards and backwards to straighten the canal.
Insert  the  otoscope.  Inspect  the  ear  canal  and  middle  ear  structure.  Note  any
redness, drainage, or deformity.

Nose:
Inspect the nose
Tilt the patient's head back slightly.
Insert  the  otoscope  into  the  nostril.  Try  to  avoid  contact  with  the  septum.  Inspect
the visible nasal structures and note any swelling, redness, drainage, or deformity.

Throat:
You: "I need to look inside your mouth for any ulcers or dental problems."

Ask the patient to open his mouth.
Note any ulcers, white patches (leucoplakia), or other lesions.
Inspect  the  posterior  oropharynx  by  depressing  the  tongue  with  tongue  depressor
and  asking  the  patient  to  say,  'Ah'.  Note  any  tonsillar  enlargement,  redness,  or
discharge.

Neck:

Inspect the neck for asymmetry, scars, or other lesions.
Palpate the neck to detect areas of tenderness, deformity, or masses.

Lymph Nodes:
You: "I would like to check for swollen glands. Let me press on this area."

Systematically palpate the various lymph node groups.
Check superficial cervical, supra clavicular, and the deep cervical lymph nodes. The
deep  cervical  chain  of  lymph  nodes  lies  below  the  sternomastoid  and  cannot  be
palpated  without  getting  underneath  the  muscle.  Inform  the  patient  that  this
procedure  will  cause  some  discomfort.  Hook  your  fingers  under  the  edge  of  the
sternomastoid  muscle.  Ask  the  patient  to  bend  their  neck  toward  the  side  you  are
examining. Move the muscle backward and palpate for the deep nodes underneath.
Note  the  size  and  location  of  any  palpable  nodes  and  whether  they  were  soft  or
hard, non­tender or tender, and mobile or fixed.

Thyroid gland:
You: "Let me check your thyroid, I am going to press very gently gently on this area"

 Inspect the neck looking for the thyroid gland.
Note the size, symmetry, and position of the lobes, as well as the presence of any
nodules. The normal gland is often not palpable.  

Special Tests:
Facial tenderness:
You:  "I  would  like  to  check  your  sinuses.  I'm  going  to  press  on  your  forehead  and
sinuses. Please let me know if it hurts."

Ask the patient to tell you if these maneuvers cause excessive discomfort or pain.
Press upward under both eyebrows with your thumbs.
Press upward under both maxillas with your thumbs.
Excessive discomfort on one side or significant pain suggests sinusitis.

Sinus transillumination:

Darken the room as much as possible.
Place a bright point light source on the maxilla.
Ask the patient to open his mouth and look for an orange glow on the hard palate.
A  decreased  or  absent  glow  suggests  that  the  sinus  is  filled  with  something  other
than air.

Temporomandibular joint:
You: "I need to examine your jaw joint."

Place the tips of your index fingers  directly in front of the tragus  of each ear. Ask


the patient to open and close his mouth.
Note any decreased range of motion, tenderness, or swelling.

Musculoskeletal System:
Inspection:

Inspect the area.
Always compare with the opposite area.
Always compare other joints for any signs of abnormality.

Palpation:

Palpate the area for warmth, and look for any swelling.
Palpate for tenderness and crepitus .
Check range of motion.
Always  check  the  neurological  status  (motor,  sensory,  reflexes.)  and  vascular
status of that area.  

Special situations:

Knee:

Always  do  the  Drawers  test  and  Mc.Murray's  test  if  patient  has  a  H/O
trauma to the knee.

Shoulders:

Range of motion is crucial. Ask him to touch the opposite shoulder.

Wrist:

Check the neurological (dermatomes) status.

Spine:

Check the flexibility, gait, and reflexes

Leg:

Homan's sign ­ this is usually done for suspected deep vein thrombosis.
Forceful dorsiflexion of the foot will cause pain in the calf.

This copy of the material is licensed to Mohammed  Mirza.
Copyright © 2015. All Rights Reserved http://www.uworld.com
Communication Skills  Print

Communication Skills

The following are points to keep in mind during your encounter with the SP.

Introduction

Always knock before entering the room.
When you enter the room, introduce yourself by name and greet the SP warmly.
Always use the SP’s name to address him/her.
Maintain good eye contact.  This demonstrates self­confidence and creates a sense
of trust and credibility.
During abdominal palpation, for example, observe the patient’s face for any
signs of pain or discomfort and maintain eye contact.

History

Start with open­ended questions before specific questions, as this is the best way
to elicit a patient’s history.
“What caused you to come in today?”
“Could you please tell me more about what’s going on?”
Ask 1 question at a time and do not ask leading questions.
Ask a question, pause, and wait for the answer before proceeding to the next
question.
For example, “Does anyone in your family have high blood pressure?” (Pause
and wait for the answer).
“Diabetes?” (Pause and wait for the answer).
Always pay attention and listen without interrupting.
Try to acknowledge the SP’s emotions.
Use layman’s language.
Try not to use medical terms, such as hypertension for high blood pressure.
Use appropriate transitional sentences.

Physical examination

Wash your hands before starting the physical examination.
Tell the SP what you are going to do (one step at a time, not the entire procedure).
Do not examine through the gown.
Ask the SP’s permission before untying the gown.
Use appropriate draping techniques.
The general rule is that as little of the body should be exposed as necessary
for a set of maneuvers to be performed.
To auscultate the heart or lungs, do NOT raise the gown up from the waist,
exposing the entire torso.
Rather, the SP should lower the gown from the top, exposing only the upper
chest and shoulders.
Offer to help the SP on and off the table during the examination.
Never repeat painful maneuvers and always apologize immediately for any pain
they caused.
Express empathy.
Try to make the SP comfortable during the examination.
Extend the leg rest when the SP lies back and push it in when the SP sits up.
If the SP is in pain, ask if there is anything you can do to help him/her feel
more comfortable.

Conclusion
Summarize the history and explain the physical examination findings.
Make appropriate reassurances but do not give false reassurances.
Never forget to ask “Do you have any questions or concerns?”

This copy of the material is licensed to Mohammed  Mirza.
Copyright © 2015. All Rights Reserved http://www.uworld.com
Closing the encounter  Print

Closing the Encounter

“All right, Mr. XYZ, thank you so much for your cooperation.  Now, I’d like to let
you know what I think so far.  First, let me summarize.”
“You just told me that __ and __.  Also, you said that __ and __.  Is that
right?”

“According to the information I got from you and the examination, I am considering
a couple of possibilities.”
“It may be __ (your probable diagnosis) or possibly __(differential
diagnosis).”
“I need to run some tests to find out exactly what the problem is.”
“As soon as I get the results, we can meet again to go over everything.”
“At that time, I’ll explain the details and we will talk about your options for
treatment.  Does this sound OK?”

If this is a psychiatric case involving depression, grief, anxiety, or dementia, ask
the following question:
“Ms. XYZ, would you be willing to talk to a counselor or go to a support
group?”

If Mr./Ms. XYZ smokes, drinks alcohol, eats fatty food, does not exercise, uses
recreational drugs, has multiple sexual partners, does not use condoms, etc., give
the following suggestions:
“Mr./Ms. XYZ, I have noticed that you__ (address the problem).”
“Are you willing to quit?  If you need any more help from me, just let me
know.  I’ll be glad to help.”

“Ms. XYZ, do you have any concerns or questions you’d like to ask before I go?”

“OK then, I’m glad that I was able to work with you.  We will try to make you feel
better.”

“Thanks for your cooperation and have a good day.”

This copy of the material is licensed to Mohammed  Mirza.
Copyright © 2015. All Rights Reserved http://www.uworld.com
Documentation of Case  Print

Documentation of History

History of present illness
Write present history and positive/negative symptoms
(HPI)

Review of systems
Include pertinent positive/negative associated symptoms
(ROS)

Include past medical conditions diagnosed by doctor and
Past medical history psychiatric conditions
(PMH) Include ob/gyn history in women
Include birth and developmental history in children

Past surgical history
Include previous operations (including cesarean sections)
(PSH)

Medications Include over­the­counter and prescription medications

Allergies Include drug allergies and the patient's reaction

Family history Include parents and siblings (if pertinent)

Include occupation (if pertinent) and support system (who
patient lives with)
Social history Include smoking, alcohol, and illicit drug use history
Outside stressors that may be causing symptoms (eg, work,
relationship)

Documentation of Physical Examination

Always write vital signs first.
Give a brief comment about patient's general appearance.
General tips
Then focus on main systemic examination noting abnormal
and relevant positive/negative findings.

Always write the most specific tests first.
List tests in order of priority.
Write all related tests in a single line (eg, CBC, ESR).
Investigations Do not write referrals or consultations.
Do not write prescriptions.
Recommend breast, renal, pelvic, or genital examinations
(if needed).
Vital signs Temperature, blood pressure, pulse, and respiratory rate

Head
Normocephalic and atraumatic (NCAT)
Eyes
Pupils equally round and reactive to light (PERRLA)
Visual acuity 20/20 with intact visual fields
Extraocular muscles intact (EOMI)
Fundi without papilledema, lesions, or exudates
Conjunctiva without erythema
No icterus
Ears
Pinnae without inflammation or tenderness
HEENT Tympanic membranes intact without erythema or effusion
Weber midline, Rinne with air conduction > bone conduction
(AC > BC)
Hearing grossly intact bilaterally
Nose
Septum midline with patent nares
No nasal polyps or lesions
No sinus tenderness to palpation
Throat
Oropharynx clear without tonsillar erythema or exudates
Uvula midline
Normal dentition and gums without ulcers or lesions

Supple without lymphadenopathy or thyromegaly
Neck Carotid pulse 2+ without jugular venous distention (JVD)
Trachea midline without accessory muscle use

No chest wall lesions, scars, or tenderness to palpation
Fremitus symmetrical
Lungs/chest Resonant to percussion bilaterally
Clear to auscultation bilaterally with vesicular breath sounds
No wheezes, crackles, or rhonchi

No visible heaves or lifts
Heart Point of maximal impulse (PMI) at 5th intercostal space in left mid­
clavicular line
Normal S1 and S2 without murmurs, gallops, or rubs

Nondistended without scars, bruises, or visible pulsations
Normoactive bowel sounds throughout without bruits
Tympanic to percussion in all 4 quadrants
Abdomen
Soft and nontender with no rebound tenderness or peritoneal signs
No hepatosplenomegaly
No costovertebral angle (CVA) tenderness

No cyanosis, clubbing, or edema
Extremities
Pulses 2+ bilaterally throughout
Patient is alert and oriented to person, place, and time
Cranial nerves (CN) II­XII intact
Motor 5/5 bilaterally in all muscle groups
Neurologic Sensory grossly intact bilaterally
Deep­tendon reflexes (DTR) 2+ bilaterally, downgoing toes bilaterally
Gait normal without dysmetria or dysdiadochokinesia
Normal Romberg test

No pain or tenderness to palpation in all joints
Musculoskeletal Normal range of motion (ROM) in all joints
No abnormalities in spine or paraspinal tenderness to palpation

This copy of the material is licensed to Mohammed  Mirza.
Copyright © 2015. All Rights Reserved http://www.uworld.com
Case Investigation  Print

These  are  the  common  investigations  that  you  should  keep  in  mind  while  writing  Pt
notes.
 
HEENT
X­ray, CT, MRI of head
Eye­ Snellen's chart, Visual acuity
Ear­ Complete audiometry and tympanometry, Culture/Sensitivity for any discharge
Routine CBC with diff, ESR

CNS
Routine CBC with diff, ESR
X­ray, CT, MRI
Lumbar puncture
Carotid Doppler study
EEG
Electromyography and Nerve conduction studies
Echocardiogram for suspected embolic phenomena

Musculoskeletal
Routine CBC with diff, ESR
X­ray
Joint aspiration for culture/ sensitive, cytology, crystals
Rheumatic factor, HLA­B27,
Serum uric acid levels
Antinuclear antibodies, anti dsDNA
Muscle biopsy

CVS
EKG and echocardiogram
Cardiac enzymes (CPK­MB, Troponin, LDH)
Chest X­ray
Lipid profile
Thyroid screen
Serum electrolytes

Respiratory
Routine CBC with diff, ESR
Chest X­ray
Sputum studies (culture/sensitivity, gram stain, AFB)
Pulmonary function tests and spirometry
PPD
ABG and pulse oximetry

Abdominal
Routine CBC with diff, ESR
Abdominal X­ray
Ultrasound of abdomen
LFTs
CT abdomen/pelvis
Upper GI series­Barium swallow, endoscopy, ERCP
Lower GI series­ enema, Colonoscopy
Test for fecal occult blood/rectal examination
Pancreatic enzymes (amylase, lipase)
Renal function tests

Endocrine
Routine CBC with diff, ESR
Blood sugar
Serum electrolytes
Serum calcium
Thyroid screen T4/T3/TSH
24hr urinary catecholamines and metabolites
Urine for ketones and sugar

Psychiatry
CBC and ESR
CT and MRI of brain
Thyroid screen
Electrolytes
Urine analysis
Drug screen / HIV

This copy of the material is licensed to Mohammed  Mirza.
Copyright © 2015. All Rights Reserved http://www.uworld.com
Challenging Qs  Print

Challenging Questions

During your patient encounter, you may be asked some questions that catch you off
guard.  These questions may be about the patient's fears and concerns or may simply
reflect the patient's curiosity.  Such questions are intentionally included to assess your
communication and counseling skills, as well as your concern for the patient.

The scenarios and challenging questions below can provide a good idea of the kinds of
questions you may encounter in the examination.  Keep in mind that although these cases
are similar to those in the examination, you may be asked about a different case with a
different challenging question.  Your objective is to be familiar with these kinds of cases
and questions rather than to simply memorize them.

Using the examples below as a guideline and not as a script will give you an idea of how
to respond to such questions.

General tips

Expect the unexpected and adapt to the best of your ability.
Keep your composure and do not panic if you do not know the answer.
To give yourself more time to think about a question (translation: stalling
tactic), you can start by saying, “That's a very good question,” and analyze
the question as you speak.
Always look confident and speak with conviction.
Once you have answered the question to the best of your knowledge, make
sure that the patient is satisfied with your reply.
Ask, “Did I address that particular concern or question of yours?” or
“Do you have any more questions?”

Patients who want a diagnosis or medical opinion

Some patients may ask for a diagnosis or your “expert opinion” during the encounter. 
Just answer the question to the best of your knowledge.
Explain why one diagnosis is more likely than others.  If you are not sure about the
diagnosis, don't panic.
Calmly explain that there are several causes that could be responsible for symptoms
and that further investigation is required.
Make sure the patient has an adequate support system before giving a definite life­
altering diagnosis.

Patient: “Do I have Alzheimer's disease?”
Response
“Alzheimer's disease is certainly a possibility.  However, there
Demented
are also other conditions that can cause memory difficulties,
patient
such as thyroid problems or vitamin deficiencies.”
“We need to first do some blood tests and then I can give you a
better idea of what may be going on.”

Patient:  “Do you think I have HIV?”
Patient with
Response
suspected HIV
(eg, fever, "You do have some risk factors for HIV, which can cause some
chronic of your symptoms."
diarrhea, weight "However, there are other treatable conditions that may cause
loss) these symptoms."
"We need to do some blood tests, including an HIV test, before
meeting again to discuss the results."

Patient:  “Am I having a heart attack?  Am I going to die?”
Response
Chest pain “Your symptoms and physical examination findings don't
patient with suggest a heart attack.”
suspected “Your symptoms are more consistent with inflammation of the
pericarditis heart membranes.”
“We need to do some blood tests, an ECG, and possibly an
echocardiogram.”

Patient:  “Do you think I've had a stroke?”
Response
Patient with “At this point, I don't know the exact cause of your symptoms.”
weakness or “A stroke is a possibility, but there could also be other causes
numbness such as a pinched nerve, infection, spinal injury, etc.”
“I need to order some blood work and imaging studies before I
can tell you for sure.”

Patient:  “Do you think this is colon cancer?” or “Do you think I'll
get colon cancer too?”
Elderly patient
Response
with
“Your family history does increase your chance of colon cancer,
constipation and
but constipation can also be due to medication side effects,
significant
thyroid problems, etc.”
family history of
“I want to do a very thorough investigation with blood work and
colon cancer
imaging studies before discussing diagnosis and treatment
options.”

Patient:  “Do you think I'm just getting old?”
Response
“Age could play an important role in decreased sexual
Elderly patient function.”
with erectile “However, there are also certain reversible causes that could
dysfunction be causing your problem such as medications, diabetes, and
high blood pressure.”
“I would first like to do some blood tests to rule these out and
then discuss treatment options with you.”

Patient:  “Do you think I need surgery?”
Response
“I understand you might be anxious or scared about possible
Acute
surgery, just like most patients.”
appendicitis
“I cannot be sure yet that you will need surgery.”
patient
“I would first like to order some blood tests and imaging
studies to evaluate you further and we can then discuss
treatment options.”

Patient:  “Do you think I need surgery?”
Response
“I'll need to order some imaging tests first to see what is
Chronic back
causing your back pain.”
pain patient
“If it can be treated using medication, we will give you pain
medication first.”
“Otherwise, surgery may be an option.”

Patient:  “Do you think I'm going to lose my baby?”
Response
Pregnant
“Losing your baby can cause bleeding, but there are many
patient with
causes of vaginal bleeding in a pregnant patient.”
vaginal bleeding
“I will need to do some blood tests and imaging to see what is
causing the bleeding.”

Patient:  “What's causing this?  Is it my thyroid?”
Patient with
Response
palpitations and
“Thyroid problems can cause palpitations in addition to other
a family history
conditions such as heart disease.”
of thyroid
“I'll need to run some tests first before discussing your
problems
diagnosis and treatment options.”

Sensitive issues

Some patients may have concerns that prevent access to adequate health care such as
financial constraints, confidentiality issues, personal fears, etc.
Express empathy for the patient regardless of how trivial the issue may seem.
Healthcare promotion is more effective with adequate doctor and patient satisfaction.

Patient:  “Should I tell my wife?  I'm scared.”
Response
“It can be very uncomfortable and difficult to discuss your
medical condition.”
HIV patient
“It's usually better to tell the truth since your wife may also be
infected with HIV and needs testing.”
“If you'd like, I can help you with the process of informing and
testing her.”

Patient:  “I don't have insurance and can't afford an operation. 
Do I really need surgery?  Are there other options?”
Acute Response
appendicitis “I understand your concerns, but surgery is really the best
patient option for your condition.”
“The hospital has a social worker who can see what financial
assistance the hospital can offer you.”
Patient:  “Do you think my insurance will cover these
Patient needing
expenses?”
extensive
Response
laboratory
“I'm not sure if it will or not since I don't have that information
workup or
right now.”
diagnostic
“However, our social worker can help evaluate your situation.”
procedures
“I can also write a letter to your insurance company describing
the procedure and its importance to your health to see if it will
cover the costs.”

Patient:  “Do you think my partner is cheating on me?”
Response
“Since the disease is transmitted sexually, the most likely
STD patient with
possibility is from your sexual partner.”
a single sexual
“It's a possibility that your partner obtained it from somebody
partner
else.  It would be best if you both discuss this further.”
“I would like your partner to also get treated to prevent
reinfection.”

Patient:  “Will I ever get better?”
Response
Patient with
“Your problem could be due to many causes such as a thyroid
chronic fatigue
problem, anemia, etc.”
and nonspecific
“I need to first investigate with blood work to rule out possible
pain
causes before going over treatment options.”
“We will then see how you do from there.”

Medication issues

Standardized patients sometimes ask about medications, their proper use, and side
effects.
It is important that you answer these questions confidently and with conviction.
If you are not sure, inform patients that you will look up the information and let them
know.

Patient:  “I'm tired of this medication.  I want to switch to an
herbal medication.”
Response
Elderly patient
“Chronic pain can be very frustrating, and I understand that you
with chronic
are not happy with your current treatment.”
osteoarthritis of
“Herbal medications may help some people, but the studies are
the knee
somewhat limited.”
“I will look at some of these studies and get back to you after
reviewing them.”

Menopausal
Patient:  “Can I go on hormone replacement therapy?”
patient with hot
Response
flashes and
“You're certainly a candidate for hormone replacement therapy,
strong family
since HRT is indicated for the treatment of hot flashes.”
history of breast
“However, HRT may increase your risk of breast cancer.”
cancer
Patient:  “I've had this before.  Can't you just give me
antibiotics?”
Response
Patients with “Your symptoms and examination are more consistent with a
apparent viral likely viral infection.”
upper “Antibiotics treat bacterial infections and are not effective
respiratory tract against viruses.”
infection “Giving antibiotics repeatedly for viral infections may lead to
increased resistance to the drugs in the future.”
“Antibiotics can also cause side effects such as diarrhea and
nausea.”

Explaining medical jargon

It is best to avoid using medical jargon while talking to the patient.
Medical jargon creates a feeling of alienation or unfamiliarity and can distance you from
the patient.
If you accidentally use a medical word, explain it in simple terms.

Patient:  “A colonoscopy?  What's that?”
Response
“A colonoscopy involves placing a tube with a camera from
Patient in need
below (in the rectum) to take a look at the whole intestine on a
of colonoscopy
television screen.”
“If there are any growths, called polyps, they can be cut out
and examined under a microscope.”

Difficult patients

These patients may be angry, demanding, or very impatient and prevent you from
having an effective and meaningful interaction.
Do not disregard these statements/questions even though they may be annoying.
Always try to resolve the situation first (ie, calm the patient down) in order to
effectively discuss the medical problem.

Patient:  “Stop asking me these stupid questions.  I'm in pain, so
just give me pain medications.”
Patient with Response
acute abdominal “I understand that you're in a lot of pain right now.”
pain “However, I need to ask you a few questions first and quickly
examine you to get a better idea of what is going on.”
“I can then decide which therapy is best to decrease your pain.”

Patient:  “I've been waiting for a long time.  What took you so
long?”
Response
Demanding “I'm very sorry that you had to wait so long.”
patient “I had some complications with my other patients that took
longer than expected to treat.”
“Let's discuss what is going on and I'll try to help you as much
as I can.”

This copy of the material is licensed to Mohammed  Mirza.
Copyright © 2015. All Rights Reserved http://www.uworld.com
List of Cases  Print

Practice Cases
Contents
Case 1: 30­year­
History taking old female
complaining of
Physical Examination Abdominal Pain

Physical Exam Videos Case 2: 27­year­
old female
Closing the encounter complaining of
rash
Documentation of Case
Case 3: 65­year­
Case Investigation old female
complaining of
Psychiatry History taking arm and leg
weakness
Pt Notes
Case 4: 29­year­
Abbreviations old female known
sickle cell
Miscellaneous anemia pt c/o
chest pain
Key to Success
Case 5: 35­year­
Communication Skills old male with
recent onset
Guidance for better cough
practice
Case 6: 50­year­
On the Day of Exam old
male complaining
CSA FAQ of fatigue and
loss of weight
Sample Cases
Case 7: 35­year­
Alcoholism old male with
acute onset
Back pain diarrhea  

Confusion  Case 8: 25­year­
old female
Chest Pain  complains of
sore throat
Chronic Cough
Case 9: 56­year­
Chronic Diarrhea old male for BP
check and refill
Dark urine of the
medications
Depression 
Case 10: 66­
Diabetic Drug Refill  year­old male
complaining of
Dizziness  constipation

Case 11: 50­
Domestic Violence  year­old male
complaining of
Enuresis  impotence

Forgetfulness/Alzheimer's  Case 12: Mother
of 1 Yr. O/Baby
Headache  With Fever

Heel pain  Case 13: 45­
year­old female
Hemoptysis c/o acute, right
upper quadrant
Insomnia  abdominal pain

Menopause  Case 14: 24­
year­old female
Night sweats came for prenatal
visit for the first
Obesity  time

Preemployment checkup  Case 15: 60­
year­old male
Shoulder pain  complaining of
acute shortness
of breath
Spells 
Case 16: 40­
Vaginal Bleeding 
year­old female
with increased
Vomiting
urination
Terminal cancer 
Case 17: 35­
year­old female
Telephone Encounter for evaluation of
jaundice
Other
Case 18: 35­
Palpitations year­old female
complaining of
Counseling Chest Pain
Other imp Cases Case 19: 45­
year­old male
complaining of Rt
lower abdominal
pain

Case 20: 55­
year­old male
with bilateral leg
pain

Case 21: 40­
year­old male
with vomiting of
blood

Case 22: 55­
year­old male
complaining of
Chest Pain

Case 23: 70­
year­old male
complaining of
Frequent Falls

Case 24: 35­
year­old male
complaining of
cough and chest
pain

Case 25: 60­
year­old male
complaining of
lower abdominal
pain

Case 26: 35­
year­old male
complains of
fatigue

Case 27: 65­
year­old female
complaining of
loss of hearing  

Case 28: 53­
year­old male
with right knee
pain and swelling

Case 29: 50­
year­old male
with blurred
vision

Case 30: 32­
year­old female
with multiple
bruises

Case 31: 20­
year­old female
with burning
urination

Case 32: 50­
year­old male
with difficulty
swallowing

Case 33: 30­
year­old male for
HIV drug refill

Case 34: 16­
year­old female
with amenorrhea

Case 35: 35­
year­old female
with acute right
lumbar and lower
abdominal pain

Case 36: 70­
year­old male
with insomnia

Case 37: 65­
year­old male
patient with
difficulty
urinating

Case38: 45­year­
old female
complains of
breathlessness
and anxiety

Case 39: 53­
year­old male
with a long
history of
epigastric pain

Case 40: 45­
year­old male
complaining of
bloody vomiting

Case 41: 60­
year­old male
complains of
dizziness

Case 42: 30­
year­old male
with new onset of
seizure

Case 43: 23­
year­old male
with rectal
bleeding

This copy of the material is licensed to Mohammed  Mirza.
Copyright © 2015. All Rights Reserved http://www.uworld.com
Alcoholism Case  Print

Doorway information about patient
The patient is a 55­year­old man brought in by his wife to
discuss ways for him to stop drinking alcohol.

Vital signs

Temperature, 98.0 F (36.7 C)
Blood pressure, 110/70 mm Hg
Heart rate, 80/min
Respiratory rate, 14/min

History

When did you start drinking alcohol?
On average, how many drinks do you have a day?
On average, how many days a week do you drink?
Have you ever tried to cut down on your drinking?
Has anyone ever criticized your drinking?
Have you ever felt bad or guilty about drinking?
HPI Have you ever had a drink first thing in the morning?
How do you feel about yourself?
Have you had any mood changes?
Do you get anxious over small things?
Have you ever had seizures or fits?
If you don’t drink for 2­3 days do you get any shakes (tremors)?
Do you have any other complaints?

Do you have any other medical problems (eg, diabetes mellitus,
Past medical
peptic ulcer disease)?
history
Have you ever been admitted to a hospital?

Do you take any medications (prescription and over­the­counter)?
Medications/
Are you allergic to any medications?
allergies
If so, what reaction do you have?

How old are your father and mother?
Family history
Do they have any medical conditions?

Who do you live with?
What kind of work do you do?
Do you have any marital or sexual problems?
Social history Have you had any family problems?
Do you have any financial problems?
Do you smoke?
Do you use recreational drugs (IV drugs)?
Physical examination

Wash your hands.
General Perform proper draping techniques.
Examine without the gown, not through the gown.

Skin Look for evidence of cirrhosis (eg, telangiectasia, jaundice).

Examine the conjunctiva for pallor and jaundice.
HEENT
Examine the oral cavity and dentition.

Auscultate lungs for adventitious sounds and heart for murmurs or
Lung/heart
gallops.

Inspect for ascites or signs of cirrhosis (eg, caput medusae).
Auscultate for bowel sounds.
Abdomen Percuss the abdomen looking for shifting dullness if ascites present.
Check for hepatomegaly.
Palpate abdomen and for hepatic tenderness.

Extremities Check for cyanosis, clubbing, or edema.

Conclusion

Summarize history and findings.
Counsel patient on the following:
Review the quantity and frequency of current
drinking.
Explain the risks associated with alcoholism.
Explain patient's responsibility to reduce or stop
drinking.
Set up a drinking diary.
Motivate the patient by discussing available
resources/support groups.
Schedule a follow­up appointment.
Investigations
CBC
Liver function tests
Gamma­glutamyl transpeptidase (GGT)

This copy of the material is licensed to Mohammed  Mirza.
Copyright © 2015. All Rights Reserved http://www.uworld.com
Backpain Case  Print

Doorway information about patient
The patient is a 60­year­old woman who comes to the clinic
complaining of 2 months of back pain.

Vital signs

Temperature, 98.0 F (36.7 C)
Blood pressure, 122/80 mm Hg
Heart rate, 90/min
Respiratory rate, 14/min

History

When did the pain start?
Can you show me exactly where the pain is?
What were you doing when the pain began?
On a scale of 1 to 10, how severe is the pain?
Please describe the pain (eg, sharp, burning, crushing, heavy).
Does this pain wake you up at night?
Does anything make the pain better?
Does anything make the pain worse?
HPI Does it move anywhere, such as your legs?
Do you have any numbness or tingling in your legs?
Do you have any weakness in your legs?
Are you able to control your urine or have you had any accidents?
Are you able to control your stool or have you had any accidents?
Have you had a fever?
Have you had any trauma to your back?
How is your appetite?
Have you lost any weight?

Have you had a similar problem before?
When was it diagnosed and treated?
Have you had any bone fractures?
Do you have any other medical problems (especially cancer or recent
Past medical
infection)?
history
Do you have pain in any other joints?
Are you in menopause?
Have you had any hormone replacement therapy?
Have you used any steroid medications?

Do you take any medications (prescription and over­the­counter)?
Medications/
Are you allergic to any medications?
allergies
If so, what reaction do you have?

How old are your father and mother?
Do they have any medical conditions?
Family history
How old are your brothers and sisters?
Do they have any medical conditions?
Who do you live with?
What kind of work do you do?
Do you smoke? At what age did you start and how much do you
Social history
smoke?
Do you drink alcohol, including beer? How much and how often?
Do you use recreational drugs (IV drugs)?

Physical examination

Wash your hands.
General Perform proper draping techniques.
Examine without the gown, not through the gown.

Test motor strength of lower extremity.
Evaluate sensory function of lower extremity.
Neurologic Test lower­extremity reflexes.
Perform straight­leg test.
Evaluate patient's gait.

Palpate back for spinal or paraspinal tenderness.
Musculoskeletal
Evaluate range of motion of lumbar spine.

Extremities Check lower­extremity pulses.

Conclusion

Summarize history and findings.
Counsel patient on the following:
Explain physical examination findings.
Explain differential diagnosis.
Explain the next steps in workup.
Suggest range­of­ motion exercises for back.
Set up follow­up appointment.
Investigations
Back x­rays.
Consider MRI of back.
ESR.

This copy of the material is licensed to Mohammed  Mirza.
Copyright © 2015. All Rights Reserved http://www.uworld.com
Chest Pain Case  Print

Doorway information about patient
The patient is a 65­year­old man who comes to clinic with 1
day of episodic chest pain.

Vital signs

Temperature, 98.0 F (36.7 C)
Blood pressure, 122/80 mm Hg
Heart rate, 90/min
Respiratory rate, 14/min

History

When did the chest pain begin this time?
Do you still have the chest pain or has it resolved?
For active chest pain
What were you doing when the pain began?
Can you think of anything that may have caused the pain?
For resolved or intermittent chest pain
How long ago did the chest pain episodes begin?
How often do the episodes occur?
How long do the episodes last?
What brings on the pain (eg, walking, exertion)?
How far can you walk before you develop chest pain or
shortness of breath?
Point with 1 finger to your chest to show me where the pain is.
HPI
Does it move anywhere else?
On a scale of 1 to 10, how severe is the pain?
Describe the pain (eg, sharp, burning, crushing, heavy).
Does anything make the pain better?
Does anything make the pain worse?
Do you have any other symptoms associated with the pain (eg,
shortness of breath, palpitations, nausea, vomiting, sweating,
lightheadedness)?
Have you had a fever?
Have you had a cough?
Have you had any recent chest trauma?
Do you have any swelling in the legs?
Do you have leg pain while walking?

Have you had a similar problem before?
When was it diagnosed and treated?
Past medical
Have you been diagnosed with any other medical conditions by a
history
doctor (eg, high blood pressure, diabetes, heart problems, high
cholesterol)?

Do you take any medications (prescription and over­the­counter)?
Medications/
Are you allergic to any medications?
allergies
If so, what reaction do you have?
How old are your father and mother?
Do they have any medical conditions?
Family history How old are your brothers and sisters?
Do they have any medical conditions?

Who do you live with?
What kind of work do you do?
Do you smoke? At what age did you start and how much do you
Social history
smoke?
Do you drink alcohol, including beer? How much and how often?
Do you use recreational drugs (IV drugs)?

Physical examination

Wash your hands.
General Perform proper draping techniques.
Examine without the gown, not through the gown.

HEENT Examine the conjunctiva for pallor.

Assess jugular venous pressure.
Palpate PMI and note location.
Cardiac
Auscultate for murmurs, rubs, or gallops.
Listen to neck for carotid bruits.

Lungs Auscultate lungs.

Check peripheral pulses.
Extremities
Examine lower extremities for edema.

Conclusion

Summarize history and findings.
Counsel patient on the following:
Explain physical examination findings.
Explain differential diagnosis.
Explain the next steps in workup.
Explain possible admission to the hospital.
Differential diagnosis
Acute coronary syndrome
Pericarditis
Aortic dissection
Pulmonary embolism
GERD
Costochondritis
Anxiety
Investigations
EKG
Cardiac enzymes
Chest x­ray
ECHO
Fasting lipid panel
Blood pressure in both arms
ABG (if patient has dyspnea)
V/Q scan if high suspicion for PE

This copy of the material is licensed to Mohammed  Mirza.
Copyright © 2015. All Rights Reserved http://www.uworld.com
Chronic Cough  Print

Doorway information about patient
The patient is a 35­year­old man who comes to the clinic
complaining of episodes of coughing for 3 months.

Vital signs

Temperature, 98.0 F (36.7 C)
Blood pressure, 122/80 mm Hg
Heart rate, 90/min
Respiratory rate, 14/min

History

When did the cough start (acute cough < 3 weeks, chronic cough > 3
weeks)?
Is it a dry cough or does it bring up some sputum?
What color is the sputum?
Has there been blood in the sputum at any time?
Is there anything that makes your cough worse?
Is there anything that makes your cough better?
Have you noticed any dripping sensation in your throat or the
frequent need to clear your throat (postnasal drip)?
Do you have any facial pain or tooth pain (sinusitis)?
HPI Do you get short of breath?
Have you noticed any wheezing? Any nighttime wheezing?
Have you had frequent heartburn, regurgitation, or sour taste
(GERD)?
Do you have any chest pain?
Do you have any pets?
Did you buy any new furniture recently?
Does your cough get worse when you are at your workplace?
Have you had a fever? Chills? Night sweats?
Have you lost any weight? How is your appetite?
Have you been exposed to anyone who has tuberculosis?

Have you had a similar problem before?
Past medical When was it diagnosed and treated?
history Have you been diagnosed with any other medical conditions by a
doctor (eg, allergies, asthma, frequent sinus infections)?

Do you take any medications (prescription and over­the­counter)?
Medications/ Look for ACE inhibitors specifically.
allergies Are you allergic to any medications?
If so, what reaction do you have?

How old are your father and mother?
Do they have any medical conditions?
Family history
How old are your brothers and sisters?
Do they have any medical conditions?
Who do you live with?
What kind of work do you do?
Do you smoke? At what age did you start and how much do you
Social history
smoke?
Do you drink alcohol, including beer? How much and how often?
Do you use recreational drugs (IV drugs)?

Physical examination

Wash your hands.
General Perform proper draping techniques.
Examine without the gown, not through the gown.

Examine oropharynx for any lesions.
Examine nasal passages for inflammation, masses, or polyps.
HEENT/ neck
Palpate sinuses to assess for tenderness.
Palpate for enlarged cervical lymph nodes.

Cardiac Auscultate for murmurs, rubs, or gallops.

Palpate to assess for symmetrical fremitus.
Lungs Percuss to assess resonance.
Auscultate lungs.

Abdomen Look for hepatomegaly

Extremities Look for peripheral edema

Conclusion

Summarize history and findings.
Counsel patient on the following:
Explain physical examination findings.
Explain differential diagnosis.
Explain the next steps in workup.
Differential diagnosis
Postnasal drip syndrome and sinusitis
Asthma
GERD
Chronic bronchitis
Bronchiectasis
Cough secondary to ACE inhibitors
Malignancy
Cough secondary to occupational exposure
Tuberculosis
Investigations
CBC with differential
Chest x­ray
Sputum Gram stain/AFB and culture, as needed
Pulmonary function tests, as needed
High­resolution CT scan, as needed

This copy of the material is licensed to Mohammed  Mirza.
Copyright © 2015. All Rights Reserved http://www.uworld.com
Confusion Case  Print

Doorway information about patient
The patient is a 65­year­old man who is brought to the clinic
by his wife for 2­3 months of confusion.

Vital signs

Temperature, 98.0 F (36.7 C)
Blood pressure, 132/84 mm Hg
Heart rate, 80/min
Respiratory rate, 14/min

History

Your wife is concerned about your being confused.  Does this happen
all the time?
Are there any specific times or situations that cause this confusion?
Do you notice any problems with your memory?
Has your wife noticed any problems with your memory?
Have you had any weakness in your arms or legs?
Have you had tingling or numbness in your arms or legs?
Have you had any episodes of dizziness?
Have you ever had any jerky hand movements or seizures?
Do you have any history of head trauma?
Have you had any fever?
HPI
Do you have headaches?
Have you ever passed out?
Have you had diarrhea or constipation?
Have you had problems with urination?
Are you able to do your daily activities such as dressing, eating,
walking, and personal hygiene (eg, bathing)?
Are you able to go shopping?
Are you able to do any housework?
Are you able to drive?
Are you able to cook or prepare food?
Please describe a typical day.

Have you had a similar problem before?
When was it diagnosed and treated?
Past medical
Have you been diagnosed with any other medical conditions by a
history
doctor (eg, diabetes, HTN)?
Have you had any surgeries?

Do you take any medications (prescription and over­the­counter)?
Medications/
Are you allergic to any medications?
allergies
If so, what reaction do you have?

How old are your father and mother?
Do they have any medical conditions?
Family history
How old are your brothers and sisters?
Do they have any medical conditions?

Who do you live with?
What kind of work do you do?
Do you smoke?  At what age did you start and how much do you
Social history
smoke?
Do you drink alcohol, including beer?  How much and how often?
Have you use recreational drugs (IV drugs)?

Physical examination

Wash your hands.
General Perform proper draping techniques.
Examine without the gown, not through the gown.

HEENT Palpate for lymphadenopathy or thyroid enalrgement.

Cardiac Auscultate for murmurs, rubs, or gallops.

Lungs Auscultate lungs.

Adomen Palpate for masses and tenderness.

Perform mini mental status examination.
Examine cranial nerves relatively quickly.
Neurologic
Test gait and reflexes.
Evaluate muscle strength and conduct gross sensory examination.

Conclusion

Summarize history and findings.
Counsel patient on the following:
Explain physical examination findings.
Explain differential diagnosis.
Explain the next steps in workup.
Differential diagnosis
Trasnient ischemic attack
Multi­infarct dementia
Electrolyte abnormalities
Medications
Alzheimer's dementia
Investigations
CBC with differential
Urinalysis
Basic metabolic panel, TSH
ECG
CT scan of the head
Carotid Doppler
Chest x­ray

This copy of the material is licensed to Mohammed  Mirza.
Copyright © 2015. All Rights Reserved http://www.uworld.com
Dark urine  Print

Doorway information about patient
The patient is a 20­year­old man who comes to the clinic
complaining of dark urine for 5 days.

Vital signs

Temperature, 98.0 F (36.7 C)
Blood pressure, 122/80 mm Hg
Heart rate, 74/min
Respiratory rate, 14/min

History

Describe what you mean by dark urine.
When did you first notice the dark urine?
Is/was there any blood in the urine?
Is it consistently the same color throughout the day?
Have you noticed a decrease or increase in the quantity of urine each
day?
Do you have any pain associated with urination?
Have you had any abdominal pain or back pain?
HPI Have you had any fever or chills?
Do you have any nausea or vomiting?
Do you have any diarrhea?
Have you eaten anything (eg, berries, colored candy, or beets) that
could cause this change in color?
Have you had any recent injury/trauma?
Do you do heavy exercise?
Have you had any recent infections, such as a sore throat?
Have you had any previous urinary or kidney problems?

Have you had a similar problem before?
When was it diagnosed and treated?
Past medical
Have you been diagnosed with any other medical conditions by a
history
doctor (eg, diabetes, HTN)?
Have you had any surgeries?

Do you take any medications (prescription and over­the­counter)?
Medications/
Are you allergic to any medications?
allergies
If so, what reaction do you have?

How old are your father and mother?
Do they have any medical conditions?
Family history
How old are your brothers and sisters?
Do they have any medical conditions?

Who do you live with?
What kind of work do you do?
Social history Do you smoke? At what age did you start and how much do you
smoke?
Do you drink alcohol, including beer? How much and how often?
Have you used recreational drugs (IV drugs)?

Physical examination

Wash your hands.
General Perform proper draping techniques.
Examine without the gown, not through the gown.

HEENT Examine the oropharynx.

Cardiac Auscultate for murmurs, rubs, or gallops.

Auscultate for bowel sounds.
Abdomen Palpate both superficially and deeply.
Examine for CVA tenderness.

Conclusion

Summarize history and findings.
Counsel patient on the following:
Explain physical examination findings.
Explain differential diagnosis.
Explain the next steps in workup.
Differential diagnosis
Glomerulonephritis
Benign familial hematuria
Exercise­induced
Kidney or bladder stones
Urinary tract infections
Tumors of the kidney and bladder
Acute tubular necrosis
Rhabdomyolysis
Medication­induced
Food­induced
Investigations
Urinalysis
CBC with differential
Creatinine phosphokinase
Basic metabolic panel
Cystoscopy depending on above results
CT scan of the abdomen depending on above
results

This copy of the material is licensed to Mohammed  Mirza.
Copyright © 2015. All Rights Reserved http://www.uworld.com
Depression Case  Print

Doorway information about patient
The patient is a 40­year­old woman who comes to the clinic
stating that she feels “down.”

Vital signs

Temperature, 98.0 F (36.7 C)
Blood pressure, 122/80 mm Hg
Heart rate, 90/min
Respiratory rate, 14/min

During the entire patient encounter, the SP will be in a disinterested mood and talk in a feeble
voice.  The doctor should always make eye contact with the patient.  The bored responses
from the patient should not frustrate the doctor.

History

What brings you in today?
How long have you been feeling this way?
Is there anything in particular that has brought this on?
Do you have anybody to talk to you when you feel down?
How's your appetite?
Have you lost or gained any weight recently?
Are you having any trouble falling asleep or staying asleep?
Do you feel guilty about anything?
Do you feel abnormally tired?
Tell me about your daily routine.
What are your hobbies and interests?
Do you have trouble concentrating?
Do you find yourself forgetting things?
HPI
What's your favorite thing to do?
Are you still feeling that way?
Have you ever felt like life wasn't worth living?
Have you ever thought about hurting yourself?
Do you ever think about how you would do it?
Do you have guns or pills at home?
Do you feel cold if everyone else appears comfortable in the room?
Are you losing any hair?
Are your bowel movements regular?
Have you had any shortness of breath?
Do you hear or see things that other people don't?
Are you willing to get help from a counselor?
Would you talk with a counselor if I set it up?

Have you had a similar problem before?
When was it diagnosed and treated?
Past medical
Have you been diagnosed with any other medical conditions by a
history
doctor (eg, diabetes, HTN)?
Have you had any surgeries?
Do you take any medications (prescription and over­the­counter)?
Medications/
Are you allergic to any medications?
allergies
If so, what reaction do you have?

How old are your father and mother?
Do they have any medical conditions?
Family history
How old are your brothers and sisters?
Do they have any medical conditions?

Who do you live with?
What kind of work do you do?
Do you smoke? At what age did you start and how much do you
Social history
smoke?
Do you drink alcohol or beer? How much and how often?
Have you used recreational drugs (IV drugs)?

Physical examination

Wash your hands.
General Perform proper draping techniques.
Examine without the gown, not through the gown.

Neck Examine the thyroid.

Cardiac Auscultate for murmurs, rubs, or gallops.

Lungs Auscultate for breath sounds.

Abdomen Palpate for any masses and tenderness.

Check reflexes, noting any delayed relaxation phase.
Neurologic
Perform MMSE (mini mental status examination) in elderly.

Conclusion

Summarize history and findings.
Counsel patient on the following:
Explain physical examination findings.
Explain differential diagnosis.
Explain the next steps in workup.
Differential diagnosis
Depression
Hypothyroidism
Anemia
Investigations
Serum TSH
CBC with differential
Comprehensive metabolic panel (CMP)

This copy of the material is licensed to Mohammed  Mirza.
Copyright © 2015. All Rights Reserved http://www.uworld.com
Diabetic Drug Refill  Print

Doorway information about patient
The patient is a 50­year­old man who comes to the clinic for
refill of his diabetes medications.

Vital signs

Temperature, 98.0 F (36.7 C)
Blood pressure, 135/70 mm Hg
Heart rate, 74/min
Respiratory rate, 16/min

History

When were you diagnosed with diabetes?
Are you currently taking any medications for diabetes?
Are you taking your medications regularly?
Do you think that your medicine is effectively controlling your
diabetes?
Have you ever taken insulin?
Do you check your blood sugar regularly?
What is the usual range of your blood sugar?
Do you have any problems that you would like to talk about?
Have there been any changes in your vision?
When was your last eye examination?
HPI
Do you feel any abnormal sensations in your legs (pins or needle­
prick sensations, tingling, or numbness)?
Have you ever had any chest pain?
Do you have any breathing problems?
Are you sexually active?
Do you have any problems during sexual intercourse?
Do you have regular bowel movements?
Do you have any problems with urination?
How is your appetite?
Have you lost or gained any weight lately?
Have you had your urine tested for protein?

Have you been diagnosed with any other medical conditions by a
Past medical
doctor (eg, HTN)?
history
Have you had any surgeries before?

Do you take any other medications (prescription and over­the­
Medications/ counter)?
allergies Are you allergic to any medications?
If so, what reaction do you have?

How old are your father and mother?
Do they have any medical conditions?
Family history
How old are your brothers and sisters?
Do they have any medical conditions?
Who do you live with?
What kind of work do you do?
Do you smoke? At what age did you start and how much do you
Social history
smoke?
Do you drink alcohol, including beer? How much and how often?
Have you used recreational drugs (IV drugs)?

Physical examination

Wash your hands.
General Perform proper draping techniques.
Examine without the gown, not through the gown.

HEENT Examine the fundi for retinopathy.

Neck Auscultate for carotid bruits.

Palpate for PMI.
Cardiac
Auscultate for murmurs, rubs, or gallops.

Conduct sensory examination on both feet (ask SP to remove shoes
Neurologic/
and socks if they are on).
extremities
Check distal pulses in at least 2 places in lower extremities.

Conclusion

Summarize history and findings.
Counsel patient on the following:
Explain physical examination findings.
Explain preventive diabetes care.
Explain the next steps in workup.
Investigations
CBC with differential count
Spot urine protein, creatinine ratio
HbA1C
BUN and serum creatinine
Lipid profile

This copy of the material is licensed to Mohammed  Mirza.
Copyright © 2015. All Rights Reserved http://www.uworld.com
Chronic Diarrhea  Print

Doorway information about patient
The patient is a 34­year­old man who comes to the clinic
complaining of 4 weeks of diarrhea.

Vital signs

Temperature, 98.0 F (36.7 C)
Blood pressure, 118/78 mm Hg
Heart rate, 86/min
Respiratory rate, 14/min

History

Please explain what you mean by diarrhea.
How many times a day are you going?
Do you have a large volume or small volume?
When did the diarrhea start (differentiate between acute and
chronic)?
What is the pattern of diarrhea? Do you have episodes of normal
bowel movement in between (continuous or intermittent)?
How often do you have diarrhea?
Does the stool have blood?
Is there pus or mucus in the stool?
Is the diarrhea greasy with fat on top?
HPI
Do you have abdominal pain (inflammatory bowel disease and
irritable bowel syndrome)?
Have you lost weight (malabsorption or malignancy)?
Is the diarrhea triggered by anything you are eating (eg, milk
products for lactose intolerance)?
Have you had increased stress recently?
Does anything make your diarrhea worse?
Does anything make your diarrhea better?
Do you have a history of recent travel?
Do you have any sick contacts?
Have you had fever or chills?

Have you had a similar problem before?
When was it diagnosed and treated?
Past medical Have you been diagnosed with any other medical conditions by a
history doctor (eg, diabetes, HTN)?
Have you had any surgeries?
Have you been exposed to any radiation in the past?

Do you take any medications (prescription and over­the­counter)?
Medications/
Are you allergic to any medications?
allergies
If so, what reaction do you have?

How old are your father and mother?
Do they have any medical conditions?
Family history How old are your brothers and sisters?
Do they have any medical conditions?

Who do you live with?
What kind of work do you do?
Do you smoke?  At what age did you start and how much do you
Social history
smoke?
Do you drink alcohol, including beer?  How much and how often?
Do you use recreational drugs (IV drugs)?

Physical examination

Wash your hands.
General Perform proper draping techniques.
Examine without the gown, not through the gown.

Examine oropharynx for any lesions.
HEENT/
Palpate neck for enlarged cervical lymph nodes, thyromegaly, and
Neck
masses.

Cardiac Auscultate for murmurs, rubs, or gallops.

Lungs Auscultate lungs.

Auscultate for bowel sounds.
Adomen Percuss for bowel gas pattern.
Palpate abdomen both superficially and deeply.

Conclusion

Summarize history and findings.
Counsel patient on the following:
Explain physical examination findings.
Explain differential diagnosis.
Explain the next steps in workup.
Differential diagnosis
Secretory diarrhea (bacterial toxins, ileal bile
acid malabsorption, endocrine diarrhea)
Osmotic diarrhea (osmotic laxatives,
carbohydrate malabsorption)
Inflammatory diarrhea (inflammatory bowel
disease, infectious diseases ­ Giardia)
Fatty diarrhea (celiac disease, short bowel
syndrome)
Investigations
CBC with differential
FOBT
Comprehensive metabolic panel
Serum TSH
Stool studies (eg, WBC, O+P)

This copy of the material is licensed to Mohammed  Mirza.
Copyright © 2015. All Rights Reserved http://www.uworld.com
Dizziness Case  Print

Doorway information about patient
The patient is a 65­year­old woman who comes to the clinic
complaining of episodes of dizziness.

Vital signs

Temperature, 98.0 F (36.7 C)
Blood pressure, 122/80 mm Hg
Heart rate, 90/min
Respiratory rate, 14/min

In real life (as well as in the Step 2 CS), evaluating a patient with dizziness can be challenging
and frustrating for the clinician.  “Dizziness” is a nonspecific term.  Patients complaining of
dizziness may be experiencing vertigo, non­specific “dizziness,” disequilibrium, presyncope, or
near­syncope.  It is therefore very important to obtain an extensive history to narrow down the
differential diagnosis.

History

How long have you had these dizzy spells?
Describe an episode and what you mean by dizziness.
Do you feel that the room is spinning?
How long does the episode last?
How often are you having episodes?
What brings on an episode?
What makes the symptoms better?
What makes the dizziness worse once it starts?
When did the dizziness first occur?
Do you have any warning signs that the episode is about to start?
Does it occur at any particular time of the day or night?
Do changes in position (eg, getting up out of a chair) make you
dizzy?
When you get dizzy, do you have a tendency to fall?  To which side?
HPI
Have you ever lost consciousness?
Do you have loss of balance when walking?
Have you had any headaches when you get dizzy?
Have you had any nausea or vomiting when you get dizzy?
Have you had a feeling of your heart racing when you get dizzy?
Have you had shortness of breath?
Have you had feelings of panic when you get dizzy?
Have you noticed any difficulty with your hearing?
Do you hear any ringing or buzzing noise in your ears?
Do you have any problems with double or blurry vision?
Have you ever noticed any weakness in your arms or legs?
Do you have any numbness in your face, arms, or legs?
Do you have any problems with your bowel movements?
Do you have any problems with urination?

Have you had similar a problem before?
Past medical When was it diagnosed and treated?
history Have you been diagnosed with any other medical conditions by a
doctor (eg, diabetes, HTN)?
Have you had any surgeries?

Do you take any medications (prescription and over­the­counter)?
Medications/
Are you allergic to any medications?
allergies
If so, what reaction do you have?

How old are your father and mother?
Do they have any medical conditions?
Family history
How old are your brothers and sisters?
Do they have any medical conditions?

Who do you live with?
What kind of work do you do?
Do you smoke?  At what age did you start and how much do you
Social history
smoke?
Do you drink alcohol or beer?  How much and how often?
Have you used recreational drugs (IV drugs)?

Physical examination

Wash your hands.
General Perform proper draping techniques.
Examine without the gown, not through the gown.

Vital signs Check for orthostatic hypotension.

Gross hearing test (if history of hearing loss), Rinne, Weber
HEENT
Otoscopic examination (if history of tinnitus or ear discharge)

Examine the thyroid.
Neck
Auscultate for carotid bruits.

Cardiac Auscultate for murmurs, rubs, or gallops.

Check for nystagmus while doing Dix­Hallpike maneuver.
Neurologic Check gait and Romberg.
Check dysmetria and dysdiadochokinesia (cerebellar tests).

People with dizziness are often reluctant to move because of the fear of falling although they
are able to walk.  Before you check the gait or perform a Romberg test, say “I can imagine
how uncomfortable it is, but I am here to assist you.  I will help you in every aspect of the
examination.  This won't take more than a couple of minutes.”

Conclusion

Summarize history and findings.
Counsel patient on the following:
Explain physical examination findings.
Explain differential diagnosis.
Explain the next steps in workup.
Differential diagnosis
Benign positional vertigo
TIA
Stroke
Postural hypotension/orthostatic hypotension
Arrhythmias
CNS tumors/Meniere's disease
Drug­induced/polypharmacy
Nonspecific dizziness
Peripheral neuropathy
Thyroid abnormalities
Anemia
Metabolic disturbances (eg, hypoglycemia)
Investigations
CBC with differential
Serum electrolytes
Thyroid function tests
Carotid Doppler ­ if there is a relevant history for
stroke/TIA
MRI of brain ­ for suspected acoustic neuroma or
any CNS tumor
24­hour Holter monitoring ­ in patients with
history of palpitations and cardiac disease

This copy of the material is licensed to Mohammed  Mirza.
Copyright © 2015. All Rights Reserved http://www.uworld.com
Domestic Violence  Print

Doorway information about patient
The patient is a 45­year­old woman who comes to the clinic
complaining of bruises on her arms and face after falling
down.

Vital signs

Temperature, 98.0 F (36.7 C)
Blood pressure, 122/80 mm Hg
Heart rate, 90/min
Respiratory rate, 14/min

There are many ways to ask direct questions about domestic violence or an abusive
relationship.  However, it is very important to be sensitive and supportive.  No single question
is right or wrong, as long as you are sensitive and supportive.  In real life, many battered
women are hesitant to initiate information about abuse but are relieved to answer when
someone asks.  For this reason, you are expected to recognize these kinds of cases during the
Step 2 CS examination.

History

How did you fall down?
I noticed that you have a number of bruises.  Did someone do this to
you?  (If the answer is yes, ask the following questions.)
Can you please tell me what happened to you?
Are you currently in a relationship where you are physically hurt,
threatened, or feel afraid?
If so, have you ever been attacked with a weapon?
How long have you been in this abusive relationship?
Has this happened before?
Are you afraid it will happen again?
Has your partner ever made you have sex when you didn’t want to?
You mentioned that your partner loses his temper with you.  How are
things between him and your children?
HPI
Does this person you live with use alcohol or drugs?
Have you ever left home?  When?
If not, have you ever wished you could leave?  What has prevented
it?
Are you planning to leave/divorce your partner?
Has your partner ever threatened or tried to commit suicide?
Do you think of suicide as a way out of the relationship?  If the
answer is yes, ask, Do you have a plan or method by which you
would kill yourself or your partner?
Do you have an emergency plan?
Are your friends or family aware of your situation?
During the encounter, tell her repeatedly that she does not deserve
to be beaten.  Battering is against the law.

Have you been diagnosed with any other medical conditions by a
Past medical
history doctor (eg, diabetes, HTN)?
Have you had any surgeries?

Do you take any medications (prescription and over­the­counter)?
Medications/
Are you allergic to any medications?
allergies
If so, what reaction do you have?

How old are your father and mother?
Do they have any medical conditions?
Family history
How old are your brothers and sisters?
Do they have any medical conditions?

What kind of work do you do?
Do you smoke?  At what age did you start and how much do you
Social history smoke?
Do you drink alcohol, including beer?  How much and how often?
Have you used recreational drugs (IV drugs)?

Physical examination

Wash your hands.
General Perform proper draping techniques.
Examine without the gown, not through the gown.

Cardiac Auscultate for murmurs, rubs, or gallops.

Lungs Auscultate for breath sounds.

Auscultate for bowel sounds.
Abdomen
Palpate for tenderness.

Examine affected body part(s) for bruises.
Skin/extremities
Palpate for any tenderness.

Conclusion

Summarize history and findings.
Counsel the patient on the following (a major part of the examination in any abuse
case):
Assure the patient you will do everything possible to maintain her safety.
Assure the patient that her medical condition will be treated appropriately
without forcing her to do anything against her will.
Tell her that her children (if present) will be cared for and kept safe.
Assure confidentiality by explaining that only with her signed consent will
her medical records be released to any other source.
Tell her that violence never ends on its own and that it almost always
escalates in severity and frequency over time.
Explain that the only way to end the abuse is to get away from the abuser.
Always be respectful and nonjudgmental by saying, “I believe you.  It’s not your
fault.  You’re not crazy and you are not alone.  Help is available.”
Before you leave, ask, “Do you think it’s safe to go home?  Do you have a safe
place to stay?  Would you like to speak with a domestic violence counselor?”

This copy of the material is licensed to Mohammed  Mirza.
Copyright © 2015. All Rights Reserved http://www.uworld.com
Enuresis Case  Print

Doorway information about patient
You will be speaking with the mother of a 5­year­old boy
who frequently wets his bed.

History

What brought you in today?
Tell me more about it. Has your son ever been dry at night or has he
started wetting his bed after being dry for a long time?
Does he have any daytime bedwetting?
On average, how many nights a week does your child wet his bed?
How many episodes of bedwetting does he have each night?
How much urine does he void during each episode?
Is there a particular time when these episodes occur?
How much fluid does he take in during the day?
Does he drink a lot of fluid before going to bed?
HPI
Does he ever have to run to the bathroom?
How many times during the day does he urinate?
How much does he urinate each time he goes?
Does he have any difficulty in initiating or stopping the stream?
Does he ever complain of dribbling or burning while urinating?
Does he complain of feeling like he did not completely empty his
bladder?
Does he have any problems associated with bowel movements?
Has this problem affected you and your family?
Is there anything you've tried so far to deal with this problem?

Has he been diagnosed with any other medical conditions such as
diabetes or sickle cell disease?
Past medical Has he had urinary tract infections?
history Has he had any surgeries or injury to his nervous system?
Did he have any problems at birth?
Were there any complications during pregnancy or delivery?

Does he take any medications (prescription and over­the­counter)?
Medications/
Is he allergic to any medications?
allergies
If so, what reaction does he have?

Do you or his father have any medical conditions?
Family history How old are his brothers and sisters?
Do they have any medical conditions?

Who does he live with?
Children often wet the bed as a result of some stressful event in their
Social history lives. Are you aware of any incident that could be causing these
symptoms in your son?
Would you describe your son as playful and social or shy and quiet?
Conclusion

Summarize history and findings.
Counsel patient on the following:
Emphasize that bedwetting is a common problem, especially in boys.  They usually
do not have any control over it.
Bedwetting does not indicate mental problems, but it can be extremely stressful
and embarrassing for children if they continue to wet the bed beyond the age of 3.
As a result, they may lose self confidence and avoid social situations like
sleepovers at their friends' houses.
Your role as a parent is to be supportive and help your child deal with the
problem.
Here's what you can do:
Monitor your child's fluid intake during the day.
Limit the amount of fluid intake 2 hours before bedtime (ideally it should not
be more than 2 ounces).
Encourage your child to go to the bathroom before going to bed.
Have your child change his pajamas and the bed sheets if he wets them at
night.  It is also helpful to have rubber­flannel sheets handy to keep the bed
dry.
Another helpful step is to set an alarm clock and have him wake up 2­3
hours after going to bed.  He can then get up and go to the bathroom.
Bed­wetting alarms are excellent options for children more than 5 years old.
An alarm is attached to the child's underwear and is triggered by the first
drop of urine.
The child then wakes up and goes to the bathroom to finish urinating.
With time, the child learns to wake up to go to the bathroom when his
bladder is full.
Motivate your child to be involved and reward him when he's “successful.”
Differential diagnosis
Primary enuresis
Secondary enuresis
Investigations
Urinalysis
Urine culture

This copy of the material is licensed to Mohammed  Mirza.
Copyright © 2015. All Rights Reserved http://www.uworld.com
Forgetfulness Case  Print

Doorway information about patient
The patient is a 70­year­old woman who comes to the clinic
complaining of episodes of forgetfulness.

Vital signs

Temperature, 97.0 F (36.1 C)
Blood pressure, 150/85 mm Hg
Heart rate, 76/min
Respiratory rate, 16/min

History

Please tell me more about your memory problems?
Do you have any problems sleeping?
Do you have any trouble eating or preparing meals for yourself?
Do you have difficulty walking?
Are you able to bathe yourself without help?
Do have any problems getting your shopping and housekeeping done?
Are you able to find your way through your house?
HPI Do you have any problems driving to the grocery store?
Do you have any difficulty managing your accounts?
Have you felt sad or lonely?
Do you have any dizzy spells or falls?
Do you feel cold if everyone else in the room feels comfortable?
Do you have any problems urinating?
Have you had any diarrhea or constipation?
Have you noticed any recent weight loss?

Have you had a similar problem before?
When was it diagnosed and treated?
Past medical
Have you been diagnosed with any other medical conditions by a
history
doctor (eg, diabetes, HTN)?
Have you had any surgeries?

Do you take any medications (prescription and over­the­counter)?
Medications/
Are you allergic to any medications?
allergies
If so, what reaction do you have?

How old are your father and mother?
Do they have any medical conditions?
Family history
How old are your brothers and sisters?
Do they have any medical conditions?

Who do you live with?  Do you have someone to take care of you in
case of an emergency?
What kind of work do you do?
Social history Do you smoke?  At what age did you start and how much do you
smoke?
Do you drink alcohol, including beer?  How much and how often?
Have you use recreational drugs (IV drugs)?

Physical examination

Wash your hands.
General Perform proper draping techniques.
Examine without the gown, not through the gown.

HEENT Examine fundi.

Cardiac Auscultate for murmurs, rubs, or gallops.

Pulmonary Auscultate for breath sounds.

Perform mini mental status examination.
Do the “get up and go” test.
Neurologic
Check for power and reflexes.
Check for sensations in legs (position and vibration).

Conclusion

Summarize history and findings.
Counsel patient on the following:
Explain physical examination findings.
Explain differential diagnosis.
Explain the next steps in workup.
Discuss Alzheimer's disease if you suspect it.
Stress the importance of a structured home
environment and precautions for avoiding falls.
Explain the necessity of taking medications
regularly.
Ask about social support and offer help if needed.

Differential diagnosis
Alzheimer's disease
Vascular dementia
Normal­pressure hydrocephalus
Vitamin B12 deficiency
Hypothyroidism
Masked depression
Chronic subdural hematoma

Investigations
CBC with differential
CT scan of the head
Serum TSH and Vitamin B12 level
Basic metabolic panel (Na, K, Cl, CO2, BUN,
creatinine, calcium)
Syphilis serology (Serum RPR)

This copy of the material is licensed to Mohammed  Mirza.
Copyright © 2015. All Rights Reserved http://www.uworld.com
Headache Case  Print

Doorway information about patient
The patient is a 40­year­old woman who comes to the clinic
complaining of episodic headaches.

Vital signs

Temperature, 98.6 F (37 C)
Blood pressure, 120/70 mm Hg
Heart rate, 80/min
Respiratory rate, 19/min

History

Please describe what your headache feels like.
How long have you had the headache?
Did the headache come on gradually or suddenly?
Is the headache constant or intermittent?
Please show me exactly where the pain is.
Does it hurt anywhere else, such as your jaw or the back of your
neck?
What were you doing when the headache started?
On a scale of 1 to 10, with 10 being the worst headache of your life,
which number best describes your pain?
Does anything relieve the pain?
Does anything make the headache worse?
Do you have nausea or vomiting?
HPI
Have you ever had this type of pain before?  If so, how long did it
last?
Do you have any warning signs (eg, blurry vision, double vision, or
flashes) before the headache starts?
How are these headaches affecting your daily life?
Do you have any fever or chills?
Have you had neck stiffness?
Have you had a runny nose, watery eyes, or scratchy throat?
Have you noticed any ear discharge?
Have you had any head trauma?
Have you had any weakness in your arms or legs?
Have you had any tingling or numbness in your hands or legs?
Have you lost or gained weight recently?

Have you had a similar problem before?
When was it diagnosed and treated?
Past medical
Have you been diagnosed with any other medical conditions by a
history
doctor (eg, diabetes, HTN)?
Have you had any surgeries?

Do you take any medications (prescription and over­the­counter)?
Medications/
Are you allergic to any medications?
allergies
If so, what reaction do you have?
How old are your father and mother?
Do they have any medical conditions?
Family history How old are your brothers and sisters?
Do they have any medical conditions?

What kind of work do you do?
Do you smoke?  At what age did you start and how much do you
Social history smoke?
Do you drink alcohol, including beer?  How much and how often?
Have you use recreational drugs (IV drugs)?

Physical examination

Wash your hands.
General Perform proper draping techniques.
Examine without the gown, not through the gown.

Examine fundi.
HEENT Palpate scalp.
Examine ears to visualize tympanic membrane.

Neck Auscultate for carotid bruits.

Musculoskeletal Palpate muscles in neck, cervical spine, and shoulder for tenderness.

Examine cranial nerve function.
Evaluate motor and sensory function in arms and legs.
Neurologic Perform Romberg test.
Check dysmetria and dysdiadochokinesia (cerebellar tests).
Assess gait.

Conclusion

Summarize history and findings.
Counsel patient on the following:
Explain physical examination findings.
Explain differential diagnosis.
Explain the next steps in workup.

Differential diagnosis
Migraine
Tension headache
Cluster headache
Sinus headache
Brain tumor
Subarachnoid hemorrhage/CVA
Meningitis/encephalitis/infections
Temporal arteritis (in elderly patients)
Medications/drugs

Investigations
CBC with differential
Sinus x­ray
CT head without contrast

This copy of the material is licensed to Mohammed  Mirza.
Copyright © 2015. All Rights Reserved http://www.uworld.com
Heel pain Case  Print

Doorway information about patient
The patient is a 35­year­old man who comes to the clinic
complaining of 1 week of left heel pain.

Vital signs

Temperature, 98.6 F (37 C)
Blood pressure, 120/70 mm Hg
Heart rate, 80/min
Respiratory rate, 12/min

History

Please tell me more about your pain?
Where exactly do you feel the pain?
On a scale of 1 to 10, with 10 being the worst, how would you rate
the severity of your pain?
How would you describe the quality of the pain?
When did this pain begin?
When you have the heel/foot pain, do you feel pain in any other part
of your body?
What makes the pain worse?
Does anything make the pain better?
How long does an episode last and how often do episodes occur?
HPI Have you had any accidents/trauma involving your foot/heel?
Do you stand for long periods during the day?
Do you walk a lot during the day?
Do you have fever?
Do you have joint pain?  Where?
Do you have any joint/back stiffness in the morning that improves as
the day goes on?
Have you had diarrhea or other recent illness (for possible reactive
arthritis)?
Have you had any discharge from your penis?
Have you had any eye problems lately?
Have you had any rashes (for psoriatic arthritis)?

Have you had a similar problem before?
When was it diagnosed and treated?
Past medical
Have you been diagnosed with any other medical conditions by a
history
doctor (eg, diabetes, HTN)?
Have you had any surgeries?

Do you take any medications (prescription and over­the­counter)?
Medications/
Are you allergic to any medications?
allergies
If so, what reaction do you have?

How old are your father and mother?
Do they have any medical conditions?
Family history How old are your brothers and sisters?
Do they have any medical conditions?

What kind of work do you do?
Do you smoke?  At what age did you start and how much do you
Social history smoke?
Do you drink alcohol, including beer?  How much and how often?
Have you used recreational drugs (IV drugs)?

Physical examination

Wash your hands.
General Perform proper draping techniques.
Examine without the gown, not through the gown.

HEENT Examine conjunctiva.

Examine entire foot.
Palpate foot for any tenderness.
Evaluate range of motion and have patient dorsiflex and plantar flex.
Musculoskeletal
Palpate tendons during flexion to evaluate for tendinitis.
Examine spine also for range of motion and any tenderness in
sacroiliac joints.

Conclusion

Summarize history and findings.
Counsel patient on the following:
Explain physical examination findings.
Explain differential diagnosis.
Explain the next steps in workup.
Can recommend the following:
Rest foot for 2­3 days.
Apply ice to area for 30 minutes 3­4
times/day.
Use soft heel pads or padded foot splint.
Avoid excess weight on heel.
Try over­the­counter ibuprofen for pain
relief.

Differential diagnosis
Plantar fasciitis
Calcaneal periostitis
Calcaneal spurs
Painful heel pad syndrome
Bone tumors
Reactive arthritis and other spondyloarthritides

Investigations
ESR
X­ray of foot and ankle, 3 views (if needed)

This copy of the material is licensed to Mohammed  Mirza.
Copyright © 2015. All Rights Reserved http://www.uworld.com
Hemoptysis Case  Print

Doorway information about patient
The patient is a 45­year­old man who comes to the clinic
complaining of cough with blood in his sputum.

Vital signs

Temperature, 98.6 F (37 C)
Blood pressure, 120/70 mm Hg
Heart rate, 80/min
Respiratory rate, 12/min

History

When did you first start coughing up blood?
How often are you coughing up blood?
Was it bright­red blood or streaked with sputum? Or rust­colored
sputum?
Are you coughing up a lot or a small amount of sputum?
Is the sputum foul­smelling?
Do you have any breathing problems?
HPI Have you had any chest pain?
Have you had fevers or chills?
Have you had sweats at night that soak your clothes?
Have you lost any weight?
How is your appetite?
Have you been exposed to anyone with tuberculosis?
Have you traveled recently within the US or outside the country?
Have you had any other sick contacts recently?

Have you had a similar problem before?
When was it diagnosed and treated?
Past medical
Have you been diagnosed with any other medical conditions by a
history
doctor (eg, diabetes, HTN)?
Have you had any surgeries?

Do you take any medications (prescription and over­the­counter)?
Medications/
Are you allergic to any medications?
allergies
If so, what reaction do you have?

How old are your father and mother?
Do they have any medical conditions?
Family history
How old are your brothers and sisters?
Do they have any medical conditions?

What kind of work do you do?
Do you smoke?  At what age did you start and how much do you
Social history smoke?
Do you drink alcohol, including beer?  How much and how often?
Have you used recreational drugs (IV drugs)?

Physical examination

Wash your hands.
General Perform proper draping techniques.
Examine without the gown, not through the gown.

HEENT Examine oropharynx.

Neck Examine for cervical lymphadenopathy.

Palpate for symmetrical fremitus.
Lungs Percuss to evaluate for dullness.
Auscultate for breath sounds and adventitious sounds.

Heart Auscultate for murmurs, gallops, and rubs.

Examine for clubbing and cyanosis.
Extremities
Examine extremities for rashes.

Conclusion

Summarize history and findings.
Counsel patient on the following:
Explain physical examination findings.
Explain differential diagnosis.
Explain the next steps in workup.

Differential diagnosis
Bronchiectasis
Acute or chronic bronchitis
Pneumonia
Bronchogenic carcinoma
Lung abscess
Tuberculosis
Connective tissue diseases (GPA, Goodpasture's
disease, lupus)
Pulmonary embolism
Pseudohemoptysis (hematemesis)

Investigations
CBC with differential
Sputum for AFB and Gram stain
Chest x­ray
Urinalysis
Basic metabolic panel
CT of the chest pending above results
This copy of the material is licensed to Mohammed  Mirza.
Copyright © 2015. All Rights Reserved http://www.uworld.com
Insomnia Case  Print

Doorway information about patient
The patient is a 45­year­old man who comes to the clinic
complaining of insomnia.

Vital signs

Temperature, 98.6 F (37 C)
Blood pressure, 120/70 mm Hg
Heart rate, 80/min
Respiratory rate, 14/min

Insomnia is a very common problem with numerous and often concurrent etiologies such as
medical conditions, medications, psychiatric disorders, sleep apnea, and poor sleep hygiene. 
The evaluation usually requires a detailed history to narrow down the differential diagnosis.

History

Please tell me more about your sleeping problem.
How long have you been having problems with sleep?
Do you have problems falling asleep?
Do you have any problem staying asleep?
Do you feel tired in the morning when you wake up?
Do you fall asleep during the day?
When do you usually go to bed?
How much time does it take you to fall asleep?
What do you do before you go to bed (eg, exercise)?
Do you drink caffeine before going to bed?
HPI Do you watch television while lying in bed?
Do you wake up with shortness of breath during the night?
Do you wake up at night to urinate?
Do you have any cough at night?
Have you been under more stress recently?
Has there been any recent traumatic event such as a death in the
family?
How is your mood?  (If the SP appears depressed, you have to ask
all questions regarding depression.)
Has anyone noticed if you gasp for air or move your legs while
sleeping?

Have you had a similar problem before?
When was it diagnosed and treated?
Past medical
Have you been diagnosed with any other medical conditions by a
history
doctor (eg, diabetes, HTN)?
Have you had any surgeries?

Do you take any medications (prescription and over­the­counter)?
Medications/
Are you allergic to any medications?
allergies
If so, what reaction do you have?
How old are your father and mother?
Do they have any medical conditions?
Family history
How old are your brothers and sisters?
Do they have any medical conditions?

What kind of work do you do?
Do you smoke?  At what age did you start and how much do you
smoke?
Social history
Do you drink alcohol, including beer?  How much and how often?  Do
you drink any alcohol before you go to bed?
Have you use recreational drugs (IV drugs)?

Physical examination

Wash your hands.
General Perform proper draping techniques.
Examine without the gown, not through the gown.

Examine for thyromegaly.
Neck
Check oral cavity.

Lungs Auscultate for breath sounds and adventitious sounds.

Heart Auscultate for murmurs, gallops, and rubs.

Adomen Palpate for any masses/tenderness.

Extremities Look for edema.

Conclusion

Summarize history and findings.
Counsel patient on the following:
Explain physical examination findings.
Explain differential diagnosis.
Explain the next steps in workup.
Possible suggestions
Maintain sleep diary to record sleep patterns.
Keep regular bedtimes and wake times, even on weekends and days off
from work.
Limit or stop the use of nicotine, caffeine, and alcohol.
Exercise regularly, but no later than late afternoon or early evening.
Do not use the bed as a place to worry, especially about not sleeping.
Write down all worries and concerns before going to bed and place list on
the dresser to examine the next morning.
Use the bedroom only for sleep.  Don't read, watch television, eat, or do
other activities in bed.
Try to avoid daytime naps.  But if you must nap, do so in the early afternoon
and for no longer than 30 minutes per day.
Eat a light snack before bedtime if hungry.
Get regular exposure to outdoor sunlight, especially in the late afternoon.

Differential diagnosis
Circadian rhythm sleep disorder
Mood disorder (depression or anxiety)
Periodic leg movements during sleep
Obstructive sleep apnea
Caffeine or alcohol intake at night
Medications
Hyperthyroidism
BPH (if urinary problems exist)

Investigations
TSH and free T4
Complete blood count
Basic metabolic panel
Polysomnography

This copy of the material is licensed to Mohammed  Mirza.
Copyright © 2015. All Rights Reserved http://www.uworld.com
Menopause Case  Print

Doorway information about patient
The patient is a 52­year­old woman who comes to the clinic
complaining of hot flashes.

Vital signs

Temperature, 98.6 F (37 C)
Blood pressure, 130/80 mm Hg
Heart rate, 80/min
Respiratory rate, 12/min

History

Please tell me when the hot flashes began.
How often do they occur?
Do you feel anything else when these flashes occur?
Can you feel the symptoms coming on before they actually begin?
When did you have your last menstrual period?
Have you noticed anything that makes these flashes get worse? (eg,
food intake)
HPI
Do you have any diarrhea or constipation?
Do you feel any burning or pain when urinating?
Have you had any pain with sexual intercourse?
How has your mood been recently?
Have you had any fever or chills?
Do you have any headache?
Have you had any weight loss?

Have you had a similar problem before?
When was it diagnosed and treated?
Past medical
Have you been diagnosed with any other medical conditions by a
history
doctor (eg, diabetes, HTN)?
Have you had any surgeries?

Do you take any medications (prescription and over­the­counter)?
Medications/
Are you allergic to any medications?
allergies
If so, what reaction do you have?

How old are your father and mother?
Do they have any medical conditions?
Family history
How old are your brothers and sisters?
Do they have any medical conditions?

Who do you live with?
What kind of work do you do?
Do you smoke?  At what age did you start and how much do you
Social history
smoke?
Do you drink alcohol, including beer?  How much and how often?
Have you used recreational drugs (IV drugs)?

Physical examination

Wash your hands.
General Perform proper draping techniques.
Examine without the gown, not through the gown.

Examine for cervical lymphadenopathy.
Neck
Examine for thyromegaly.

Lungs Auscultate for breath sounds and adventitious sounds.

Heart Auscultate for murmurs, gallops, and rubs.

Abdomen Palpate for masses and tenderness.

Neurologic Evaluate deep tendon reflexes.

Conclusion

Summarize history and findings.
Counsel patient on the following:
Explain physical examination findings.
Explain differential diagnosis.
Explain the next steps in workup.

Differential diagnosis
Menopause
Hyperthyroidism
Medication­induced (eg, vasodilators)
Carcinoid syndrome
Pheochromocytoma

Investigations
CBC with differential
Serum TSH
Serum FSH and LH (only in some doubtful cases)

This copy of the material is licensed to Mohammed  Mirza.
Copyright © 2015. All Rights Reserved http://www.uworld.com
Miscellaneous Cases  Print

Other Important Cases

1. Upper extremity pain
In the history, just follow LIQOR AAA and PAM HUGS FOSS. Consider these issues during
your history taking:
Carpal tunnel syndrome (ask about the occupation)
Cervical spondylitis
Herniated cervical disc
Thoracic outlet syndrome (ask whether the symptoms worsen with the above head
activities, like combing)
Tenosynovitis
Trauma
Referred pain from coronary ischemia

Physical Examination:
Wash your hands.
Check the thyroid gland.
Check the neck movements and the range of motion.
Do thoracic outlet test. (Adson's test)
Ask him to take a deep breath. Extend the neck and turn the chin towards the
opposite side.
Repeat the test with the chin on the opposite side.
In the presence of thoracic outlet syndrome, the radial pulse will disappear.
Do Phalen's test (for carpal tunnel syndrome)
Hold the patient's wrists in acute flexion for 30­60 seconds.
Patient will complain of pain, numbness, and tingling over the distribution of
medial nerve, if the test is positive.
You can also elicit Tinel's sign, if you want.
With your finger, percuss over the course of the medial nerve in the carpal
tunnel.
Patient will complain of pain, numbness, and tingling over the distribution of
the medial nerve, if the test is positive.
Check sensations, muscle strength, and reflexes of both upper extremities.

Investigations:
CBC with differential, ESR
EMG and nerve conduction studies
X­ray of the cervical and thoracic spine
ECG
MRI of the spine

2. A 34 yo F who came for a bronchial asthma drug refill
This case is not that important, but there is always the possibility that you may
encounter this in the step 2 CS, so just take a look.
Start with a formal greeting. Ask open­ended questions like, "What brought you in
today?"
The things that you need to ask specifically for this case are:
"Can you please tell me more about your asthma? When were you diagnosed
for the first time? How have you been doing since then?"
"Can you please tell me about your current medications?" or "What
medications are you on?"
"Did you notice any problems or side effects with your medications?"
"Do you have any trouble breathing during the day or night with regular
activity?"
"How often does this occur on a weekly basis?"
"Do you have any trouble breathing with exercise?"
"How often does this occur on a weekly basis?"
"Do you have episodes of excessive coughing during the day or night time?"
"How often does this occur on a weekly basis?"
"Have you ever been admitted to the hospital for an acute or severe attack?"
"Tell me, what do you think about the severity of your asthma? Do you think
it is getting better or worse?"
"Do you know what precipitates your asthma?"
"Are you taking any precautions to avoid those?"
After this you, will just have to follow PAM HUGS FOSS. Make sure you ask about
her smoking history, and talk about the importance of smoking cessation.

Physical Examination:
The PE basically requires:
an HEENT exam to look for any sinus tenderness (sinusitis), or signs of upper
respiratory tract infection, which can aggravate or precipitate asthma.
a complete lung examination
looking for JVD and pedal edema (for signs of cor pulmonale, even though it
is a very rare complication of asthma)

Investigations:
Spirometry or pulmonary function tests are usually not required, unless the patient
is elderly and having persistent asthma.
For chronic, persistent, and refractory asthma, request:
CBC with differential
Aspergillus serology
Chest x­ray
X­ray of paranasal sinuses
24 hour pH for GERD
Skin tests

Differential diagnosis for chronic, persistent asthma in a smoker includes:
Bronchial asthma
Chronic obstructive pulmonary disease
Bronchopulmonary aspergillosis
Sinusitis
Atypical GERD

This copy of the material is licensed to Mohammed  Mirza.
Copyright © 2015. All Rights Reserved http://www.uworld.com
Nightsweats Case  Print

Doorway information about patient
The patient is a 22­year­old man who comes to the clinic
complaining of night sweats.

Vital signs

Temperature, 98.6 F (37 C)
Blood pressure, 130/80 mm Hg
Heart rate, 80/min
Respiratory rate, 12/min

History

How long have you had night sweats?
Have you had any fevers or chills?
Have you lost any weight unintentionally?
How is your appetite?
Do you have any weakness or fatigue?
Do you have itching (pruritus)?
Do you have pain anywhere (back pain and fever suggest
osteomyelitis)?
HPI
Do you have a cough?
Do you have any breathing problems?
Have you had any headaches?
Have you had any palpitations (racing or pounding heart)?
Have you had any diarrhea?
Do you feel warm if everyone else in the room feels comfortable?
Have you traveled anywhere recently?
Have you been exposed to any sick contacts recently?

Have you had a similar problem before?
When was it diagnosed and treated?
Past medical
Have you been diagnosed with any other medical conditions by a
history
doctor (eg, diabetes, HTN)?
Have you had any surgeries?

Do you take any medications (prescription and over­the­counter)?
Medications/
Are you allergic to any medications?
allergies
If so, what reaction do you have?

How old are your father and mother?
Do they have any medical conditions?
Family history
How old are your brothers and sisters?
Do they have any medical conditions?

Who do you live with?
What kind of work do you do?
Do you smoke?  At what age did you start and how much do you
Social history smoke?
Do you drink alcohol, including beer?  How much and how often?
Have you used recreational drugs (IV drugs)?
Are you sexually active?
Do you have sex with men, women, or both?

Physical examination

Wash your hands.
General Perform proper draping techniques.
Examine without the gown, not through the gown.

Examine for lid lag/exophthalmos.
HEENT
Examine oropharynx for thrush.

Examine for cervical lymphadenopathy.
Neck
Examine for thyromegaly.

Lungs Auscultate for breath sounds and adventitious sounds.

Heart Auscultate for murmurs, gallops, and rubs.

Abdomen Palpate for hepatosplenomegaly.

Evaluate deep tendon reflexes.
Neurologic
Examine for hand tremor.

Skin Examine for rashes.

Conclusion

Summarize history and findings.
Counsel patient on the following:
Explain physical examination findings.
Explain differential diagnosis.
Explain the next steps in workup.

Differential diagnosis
Malignancy (lymphoma, solid tumors)
Infections (tuberculosis, HIV, endocarditis)
Endocrine disorders (hyperthyroidism,
pheochromocytoma)
Medications (antidepressants, cholinergic
agonists, hypoglycemic agents)

Investigations
CBC with differential
ESR
Blood cultures
Chest x­ray and PPD
TSH
ELISA for HIV
CT scan of chest and abdomen for lymphoma

This copy of the material is licensed to Mohammed  Mirza.
Copyright © 2015. All Rights Reserved http://www.uworld.com
Obesity Case  Print

Doorway information about patient
The patient is a 40­year­old woman who comes to the clinic
complaining of weight gain.

Vital signs

Temperature, 98 F (36.7 C)
Blood pressure, 150/90 mm Hg
Heart rate, 68/min
Respiratory rate, 16/min

History

I see that you are concerned about weight gain.  How long have you
noticed the weight gain?
How many pounds have you gained?
Can you think of any reasons for this weight gain?
How is your appetite?
If increased, how long have you had this increased appetite?
How many meals do you eat in a day?
Give me examples of what you eat at each meal (breakfast, lunch,
and dinner).
Do you eat any snacks in between?
HPI How often do you eat out?
How much food (portion size) do you eat during your meals?
How is your mood? Are you feeling okay?
Do you have any problems with your breathing, especially at night?
Are you urinating more often? If so, do you urinate more than usual?
Are you feeling thirstier than usual?
Have you had any diarrhea or constipation?
Do you feel cold if everyone else in the room feels comfortable?
Do you have any joint pain?
When was your last period?
Have your periods been irregular at all?

Has a doctor diagnosed you with any other medical conditions (eg,
Past medical diabetes, HTN)?
history Have you had your cholesterol checked recently?
Have you had any surgeries?

Do you take any medications (prescription and over­the­counter)?
Medications/ Are you allergic to any medications?
allergies If so, what reaction do you have?
Have you ever taken steroids?

How old are your father and mother?
Do they have any medical conditions?
Family history
How old are your brothers and sisters?
Do they have any medical conditions?
What kind of work do you do?
Do you smoke?  At what age did you start and how much do you
Social history smoke?
Do you drink alcohol, including beer?  How much and how often?
Have you used recreational drugs (IV drugs)?

Physical examination

Wash your hands.
General Perform proper draping techniques.
Examine without the gown, not through the gown.

Examine for lid lag.
HEENT
Examine for exophthalmos.

Neck Examine for thyromegaly.

Lungs Auscultate for breath sounds and adventitious sounds.

Heart Auscultate for murmurs, gallops, and rubs.

Extremities Examine for peripheral edema.

Conclusion

Summarize history and findings.
Counsel patient on the following:
Explain physical examination findings.
Explain differential diagnosis.
Explain the next steps in workup.

Differential diagnosis
Obesity
Hypothyroidism
Cushing's syndrome

Investigations
CBC with differential
Fasting blood sugar
Serum TSH
Urine cortisol levels
Fasting lipid profile

This copy of the material is licensed to Mohammed  Mirza.
Copyright © 2015. All Rights Reserved http://www.uworld.com
Palpitations Case  Print

Doorway information about patient
The patient is a 28­year­old man who comes to the clinic
complaining of episodes of palpitations.

Vital signs

Temperature, 98.6 F (37 C)
Blood pressure, 130/80 mm Hg
Heart rate, 80/min
Respiratory rate, 12/min

History

Please describe an episode of what you mean by palpitations.
Do you have any symptoms other than the palpitations?
When did these episodes begin?
How often are they happening?
How long do they last?
Do you feel that your heart is skipping any beats?
What are you doing when you have these episodes?
Does anything seem to trigger them?
Do you drink any caffeine? If so, how much?
What makes the symptoms better?
HPI Have you had any chest pain?
Have you had any breathing problems?
Do you feel any dizziness or light­headedness?
Have you ever passed out?
Do you have a fever?
Do you get tremors in your hands?
Do you sweat excessively?
Do you get headaches with these episodes?
Have you noticed any swelling in your legs?
Do you check your blood pressure during these episodes? If so, is it
high?

Have you had a similar problem before?
When was it diagnosed and treated?
Past medical
Have you been diagnosed with any other medical conditions by a
history
doctor (eg, diabetes, HTN)?
Have you had any surgeries?

Do you take any medications (prescription and over­the­counter)?
Medications/
Are you allergic to any medications?
allergies
If so, what reaction do you have?

How old are your father and mother?
Do they have any medical conditions?
Family history
How old are your brothers and sisters?
Do they have any medical conditions?
What kind of work do you do?
Do you smoke?  At what age did you start and how much do you
Social history smoke?
Do you drink alcohol, including beer?  How much and how often?
Have you used recreational drugs (IV drugs)?

Physical examination

Wash your hands.
General Perform proper draping techniques.
Examine without the gown, not through the gown.

Examine conjunctiva for pallor.
HEENT
Examine for exophthalmos.

Examine for cervical lymphadenopathy.
Neck
Examine for thyromegaly.

Lungs Auscultate for breath sounds and adventitious sounds.

Heart Auscultate for murmurs, gallops, and rubs.

Abdomen Palpate for hepatosplenomegaly.

Evaluate deep tendon reflexes.
Neurologic
Examine for hand tremor.

Extremities Examine for any peripheral edema.

Conclusion

Summarize history and findings.
Counsel patient on the following:
Explain physical examination findings.
Explain differential diagnosis.
Explain the next steps in workup.

Differential diagnosis
Cardiac arrhythmias
Valvular heart disease
Hyperthyroidism
Pheochromocytoma
Anxiety/panic attacks
Anemia
Hypoglycemia (diabetic patients)

Investigations
CBC with differential
12­lead EKG
Serum TSH and free T4
Basic metabolic panel
Event monitor
Transthoracic echocardiogram

This copy of the material is licensed to Mohammed  Mirza.
Copyright © 2015. All Rights Reserved http://www.uworld.com
Pre­emp Checkup  Print

Case of a 25 yo M who came for Pre­employment Check­up

You may get these kinds of cases as either "Pre­employment check­up" or as an
"Insurance check­up". They will tell you everything you have to do. Some may ask
you to fill out a form (It will be provided for you.). If they want you to do any
specific examination, do that, but do all the things that are mentioned in the form
first.
You can fill out the form after you leave the room. If the SP asks about the form,
tell him that you will mail it to his home.
After you finish the examination, ask, "Do you have any questions?" Answer any
questions, and then take a relevant history.
In case you were not provided with a form, you will have to take a simple general
history. Ask cardinal symptoms of each system.
"Do you have any cough?" ("No.")
"Do you have any problems with breathing?" ("No.")
"Do you have any chest pain?" ("No.")
"Do you have headaches?" ("No.")
"Do you have a fever?" ("No.")
"Do you have any pain?" ("No.")
"Do you have weakness in the extremities?" ("No.")
"How is your bowel habit?" ("Pretty good.")
"How is your bladder function?" ("Good.")
Then you must ask PAM HUGS FOSS.
Please do not forget to ask about allergy, smoking, alcohol, and sexual history.
Here is the sample of the form that you might get. It may not be exactly like this.
You may get some of the components of this form.
If they ask you to measure blood pressure (on the form/doorway information), you
have to measure it. This is different from all other cases where you don't need to
check blood pressure.
After finishing the case, you just have to fill out this form. You don't need to write
any history, or things that they have not asked you. All you have to do is fill out
the form.

Height  

Weight  
Blood  
pressure
Pulse rate  
Lung
 
auscultation

Heart
 
auscultation

CNS
 
reflexes

Abdomen  

Spine
 
examination

This copy of the material is licensed to Mohammed  Mirza.
Copyright © 2015. All Rights Reserved http://www.uworld.com
Shoulder pain Case  Print

Doorway information about patient
The patient is a 56­year­old man who comes to the clinic
complaining of shoulder pain.

Vital signs

Temperature, 98.6 F (37 C)
Blood pressure, 130/80 mm Hg
Heart rate, 80/min
Respiratory rate, 12/min

History

Be careful to shake hands because the patient may be supporting the
painful hand with the opposite hand. (Try not to cause the patient
unnecessary pain.)
When did this pain begin?
Did you fall or have any trauma?
Describe the pain.
Does the pain move anywhere?
How severe is the pain on scale of 1 to 10?
Show me where the pain is with one finger?
HPI What makes it worse?
What makes it better?
Is it constant or intermittent?
Do you have any numbness or tingling in your arms or hands?
Have you noticed any redness or swelling in the shoulder?
Do you have pain in any other part of your body?
Do you have weakness in any other part of your body?
Do you have any other symptoms? (cough, shortness of breath)
Have you taken any medications for this? If so, did they help?
How is this affecting your daily activities?

Have you had a similar problem before?
When was it diagnosed and treated?
Past medical
Have you been diagnosed with any other medical conditions by a
history
doctor (eg, diabetes, HTN)?
Have you had any surgeries?

Do you take any medications (prescription and over­the­counter)?
Medications/
Are you allergic to any medications?
allergies
If so, what reaction do you have?

How old are your father and mother?
Do they have any medical conditions?
Family history
How old are your brothers and sisters?
Do they have any medical conditions?
What kind of work do you do?
Do you smoke?  At what age did you start and how much do you
Social history smoke?
Do you drink alcohol, including beer?  How much and how often?
Have you used recreational drugs (IV drugs)?

Physical examination

Wash your hands.
General Perform proper draping techniques.
Examine without the gown, not through the gown.

Inspect painful joint for swelling or redness.
Palpate joint for warmth or tenderness.
Compare to other arm.
Musculoskeletal
Check range of motion (passive and active) of shoulder, arm, and
hand, including flexion, extension, abduction, and adduction.
Always adduct arm across chest (crossover test).

Check reflexes.
Neurologic Check motor function and gross sensory examination in hand and
arm.

Conclusion

Summarize history and findings.
Counsel patient on the following:
Explain physical examination findings.
Explain differential diagnosis.
Explain the next steps in workup.

Differential diagnosis
Rotator cuff injury
Subacromial bursitis
Frozen shoulder (adhesive capsulitis)
Shoulder dislocation/ fracture
Referred pain

Investigations
ESR
X­ray of shoulder joint: 2 views, including elbow
Chest x­ray in a smoker to rule out lung cancer
involving brachial plexus
MRI of shoulder pending above results

This copy of the material is licensed to Mohammed  Mirza.
Copyright © 2015. All Rights Reserved http://www.uworld.com
Smoking Cessation  Print

Doorway information about patient
The patient is a 55­year­old woman who comes to the clinic
for counseling on smoking cessation.

Vital signs

Temperature, 98.0 F (36.7 C)
Blood pressure, 122/80 mm Hg
Heart rate, 70/min
Respiratory rate, 14/min

History

When did you start smoking?
As you know, some people smoke and some don't.  What were the
circumstances that caused you to begin smoking?
Do you enjoy smoking?
HPI How many cigarettes do you smoke a day?
Are you concerned about your health?
Is stress or depression a reason for your smoking?
Have you ever had any smoking­related problems like a cough or
shortness of breath?

Past medical Do you have any other medical problems (especially cancer or recent
history infection)?

Do you take any medications?
Medications/
Are you allergic to any medications?
allergies
If so, what reaction do you have?

How old are your father and mother?
Do they have any medical conditions?
Family history
How old are your brothers and sisters?
Do they have any medical conditions?

What kind of work do you do?
Social history Do you drink alcohol, including beer?  How much and how often?
Have you used recreational drugs (IV drugs)?

Conclusion/counseling

Don't address the patient negatively, such as saying,
“You must stop smoking; you're killing yourself.”
Always start in a nonjudgmental manner.
Most patients I've counseled have tried to stop
smoking at some point. Have you ever tried to
quit?
What happened?
Stopping smoking can decrease your risk of
cancer, heart attack, and lung disease.
Your breathing will improve and you will have
more energy.
Have you ever tried a nicotine patch or gum?
If the patient does not want to quit smoking you can
say, “It seems you really don't want to give up
smoking right now.  I wonder if you could cut back
from 2 packs a day to 1.”
Always praise the patient using positive language
("Excellent" or “That's great”).
Finally, repeat sentences such as, “I appreciate your
motivation to quit smoking.”

This copy of the material is licensed to Mohammed  Mirza.
Copyright © 2015. All Rights Reserved http://www.uworld.com
Spells Case  Print

Doorway information about patient
The patient is a 55­year­old man who comes to the clinic
complaining of passing out.

Vital signs

Temperature, 98.6 F (37 C)
Blood pressure, 130/80 mm Hg
Heart rate, 80/min
Respiratory rate, 12/min

History

Please describe what happened.
What were you doing when this happened?
Has this happened before?
Did someone see you pass out?
How long were you out?
Did you have any jerky movements of your hands or legs?
Did you bite your tongue?
Did you lose control of your urine?
Did you lose control of your bowels?
HPI
Were you feeling fine before the event and between spells?
Did you have any nausea or vomiting before the spell?
Have you had any chest pain?
Have you had any breathing problems?
Have you had any palpitations?
Have you had any weakness in your legs and arms?
Have you noticed tingling and numbness anywhere?
Have you had any blurry vision or changes in your vision?
Have you had any recent trauma or injury?

Have you been diagnosed with any other medical conditions by a
Past medical
doctor (eg, diabetes, HTN)?
history
Have you had any surgeries?

Do you take any medications (prescription and over­the­counter)?
Medications/
Are you allergic to any medications?
allergies
If so, what reaction do you have?

How old are your father and mother?
Do they have any medical conditions?
Family history
How old are your brothers and sisters?
Do they have any medical conditions?

What kind of work do you do?
Do you smoke?  At what age did you start and how much do you
Social history smoke?
Do you drink alcohol, including beer?  How much and how often?
Have you used recreational drugs (IV drugs)?

Physical examination

Wash your hands.
General Perform proper draping techniques.
Examine without the gown, not through the gown.

Vital signs Check for orthostatic hypotension if not given in vital signs already.

Neck Auscultate for carotid bruits.

Heart Auscultate for murmurs, gallops, and rubs.

Evaluate motor strength in all areas.
Perform gross sensory examination in extremities.
Neurologic
Check reflexes.
Evaluate gait.

Conclusion

Summarize history and findings.
Counsel patient on the following:
Explain physical examination findings.
Explain differential diagnosis.
Explain the next steps in workup.

Differential diagnosis
Cardiac arrhythmias
Vasovagal syncope (neurocardiogenic)
Orthostatic hypotension
Structural heart disease (aortic disease)
Seizures
TIA/stroke

Investigations
Complete blood count
Basic metabolic panel
ECG
Event monitor
Echocardiogram
Upright tilt table test (neurocardiogenic syncope)
CT or MRI of brain
EEG pending above results

This copy of the material is licensed to Mohammed  Mirza.
Copyright © 2015. All Rights Reserved http://www.uworld.com
Telephone Encounter  Print

Doorway information about patient
The patient is a 5­year­old boy who has been vomiting for 1
day.  His mother has left a phone message with the clinic. 
You will talk with the patient's mother on the phone.

History

I'd like to get a few details from you about your son.
How old is he?
How long has he been vomiting?
How many times has he vomited since yesterday?
Has he been vomiting large amounts?
What does the vomitus contain?
Did you notice any blood in the vomitus?
Does he have any pain in his belly?
HPI Is he having any diarrhea or constipation?
Does he have a fever?
How is his appetite?
Did he eat out recently, at a party or restaurant?
Does he have any headaches?
Is he usually a healthy child or does he frequently get sick?
Has he been exposed to any sick contacts?
Has he traveled anywhere recently?
Has he had all of his recommended vaccinations?

Has he had a similar problem before?
When was it diagnosed and treated?
Past medical
Has a doctor diagnosed him with any other medical conditions?
history
Were there any problems at birth or during the pregnancy?
Has he had any surgeries?

Does he take any medications (prescription and over­the­counter)?
Medications/
Is he allergic to any medications?
allergies
If so, what reaction does he have?

How old are you and his father?
Do either of you have any medical conditions?
Family history
How old are his brothers and sisters?
Do they have any medical conditions?

Who does he live with?
Social history What grade is he in school?
Is he doing OK in school?

Conclusion

Summarize history and findings.
Counsel patient on the following:
Explain differential diagnosis.
Explain the next steps in workup (bring child to
clinic or hospital).
If the child is very sick, he needs to be seen for
proper assessment of volume status.

Differential diagnosis
Gastroenteritis
Gastroparesis
Intussusception
Adrenal insufficiency
Intracranial hypertension
Meningitis
Medications/toxins

This copy of the material is licensed to Mohammed  Mirza.
Copyright © 2015. All Rights Reserved http://www.uworld.com
Terminal cancer Case  Print

Doorway information about patient
The patient is a 69­year­old man with a history of terminal
lung cancer who comes to the clinic requesting pain
medication.

Vital signs

Temperature, 98.6 F (37 C)
Blood pressure, 130/80 mm Hg
Heart rate, 80/min
Respiratory rate, 12/min

History

I understand that you have been diagnosed with cancer.  How can I
help you today?
Please tell me more about your cancer.
I am very sorry to hear that and I am sure that it has been difficult
for you.  Tell me more about your pain.
How severe is the pain, on a scale of 1 to 10, with 10 being the
worst?
Does anything improve your pain?
What makes your pain worse?
Do you have pain anywhere else?
HPI
Are you using any medication, such as morphine, for your pain?
Do you have any complaints other than pain?
How is your appetite?
Have you lost any weight?
Have you had fever?
Have you had diarrhea or constipation?
Do you have any problems urinating?
Have you been able to sleep well?
How is your mood?
Have you thought about hurting yourself at all?

Have you been diagnosed you with any other medical conditions by a
Past medical
doctor (eg, diabetes, HTN)?
history
Have you had any surgeries?

Do you take any medications (prescription and over­the­counter)?
Medications/
Are you allergic to any medications?
allergies
If so, what reaction do you have?

Who do you live with?
Do you have other family members or friends who provide emotional
Family history/ support for you?
Social history Do you smoke?  At what age did you start and how much do you
smoke?
Do you drink alcohol, including beer?  How much and how often?
Physical examination

Wash your hands.
General Perform proper draping techniques.
Examine without the gown, not through the gown.

HEENT Examine conjunctiva for pallor or jaundice.

Heart Auscultate for murmurs, gallops, and rubs.

Lungs Auscultate for breath sounds and adventitious sounds.

Abdomen Palpate for masses and tenderness.

Extremities Evaluate for any edema.

Conclusion

Summarize history and findings.
Counsel patient on the following:
Show support, be sensitive, and convey
necessary information.
Discuss pain medications such as narcotics, etc.
Discuss the patient's living situation (home,
assisted living).
Discuss hospice care
Can allow patient to live pain­free with
comfort care.
Can provide social, spiritual, and physical
support for patient and family.
All hospice care is under professional
medical supervision.
Discuss advance directives.

This copy of the material is licensed to Mohammed  Mirza.
Copyright © 2015. All Rights Reserved http://www.uworld.com
Vaginal Bleeding  Print

Case of a 20 yo F Complaining of Vaginal Bleeding 

Vital Signs:
BP: 110/70 mm Hg
Pulse: 80/min
RR: 16/min
T: 36.7C(98F)

History Taking:
When did the bleeding start?
Was the onset gradual or sudden?
Can you describe the bleeding?
For example, is it bright red or clotted blood?
Is the blood pure or does it contain tissue like substance? (A molar
pregnancy would have grape like tissue.)
Has it been a continuous flow or spotting?
What were you doing when it started? Were you sleeping or having sex?
Do you have any other symptoms besides bleeding? Did you have abdominal pain?
Fever? Vomiting?
Were you ever involved in any accident/trauma?
Have you ever been pregnant?
When was your last menstrual period (LMP)?
Can you describe more about your menstrual cycle?
How heavy is the flow?
How many pads do you use per day?
How long are your periods?
Are your periods regular or irregular?
Have you had any abortions?
Have you ever been tested for STDs?

Past Medical History:
Have you ever had any history of bleeding? Were you ever hospitalized for
bleeding?

Medications:
Are you currently taking any medications?
What medications have you recently taken?

Family history:
Do you have a family history of bleeding disorders?
Does anyone in your family have a history of multiple abortions?

Sexual History:
Are you married?
If yes:
Do you have any other sexual partners? (yes)
Do you use any means of contraception?
If no:
Do you have any other sexual partners? (yes)
Do you use any means of contraception?
When was your last sexual contact?

Social History:
Do you smoke?
Do you drink alcohol?
Do you use illicit drugs? (Cocaine may cause bleeding.)

Physical Examination:
Wash your hands.
Perform the proper draping technique.
Look for other sites of bleeding, i.e. nose or gums
Check for orthostatic hypotension.
Auscultate the abdomen.
Percuss the abdomen for liver span.
Palpate the abdomen superficially.
Palpate the abdomen deeply.
Check for rebound tenderness.
Ask to perform a pelvic exam.
Examine without the gown, not through the gown.

Differential Diagnosis:
Regular menses
Abortion
Pregnancy
Ectopic pregnancy
Hydatiform mole

Investigations:
Pelvic examination
Pregnancy test
CBC with differential
Transvaginal ultrasound
Serum R­HCG levels
Serum TSH

This copy of the material is licensed to Mohammed  Mirza.
Copyright © 2015. All Rights Reserved http://www.uworld.com
Vomiting Case  Print

Doorway information about patient
The patient is a 25­year­old woman who comes to the clinic
complaining of vomiting.

Vital signs

Temperature, 98 F (36.7 C)
Blood pressure, 110/70 mm Hg
Heart rate, 80/min
Respiratory rate, 14/min

History

When did the vomiting start?
Was the vomiting projectile (forceful)?
Was there any blood in the vomitus?
How often are you vomiting?
Did you eat at a restaurant?  Is anyone else who ate there sick?
Have you been exposed to any sick contacts?
Do you have any abdominal pain?
HPI Do you have any back pain?
Do you have diarrhea?
Do you have constipation?
Do you have fever and chills?
Have you had any headaches?
Do you have any burning with urination?
When was your last menstrual period?
Is there a chance you could be pregnant?

Have you had a similar problem before?
When was it diagnosed and treated?
Past medical
Have you been diagnosed with any other medical conditions by a
history
doctor (eg, diabetes, HTN)?
Have you had any surgeries?

Do you take any medications (prescription and over­the­counter)?
Medications/
Are you allergic to any medications?
allergies
If so, what reaction do you have?

How old are your father and mother?
Do they have any medical conditions?
Family history
How old are your brothers and sisters?
Do they have any medical conditions?

What kind of work do you do?
Do you smoke?  At what age did you start and how much do you
Social history smoke?
Do you drink alcohol, including beer?  How much and how often?
Have you used recreational drugs (IV drugs)?

Physical examination

Wash your hands.
General Perform proper draping techniques.
Examine without the gown, not through the gown.

Vital signs Check for orthostatic hypotension if not given in vital signs already.

Examine oropharynx for dry mucous membranes.
HEENT
Examine fundi if suspecting increased intracranial pressure.

Heart Auscultate for murmurs, gallops, and rubs.

Inspect for abdominal distention.
Auscultate for bowel sounds.
Abdomen Percuss for bowel gas pattern.
Palpate for tenderness and rebound tenderness.
Examine for CVA tenderness.

Conclusion

Summarize history and findings.
Counsel patient on the following:
Explain physical examination findings.
Explain differential diagnosis.
Explain the next steps in workup.

Differential diagnosis
Gastroenteritis
Intestinal obstruction
Pregnancy
Medications/toxins
Meningitis
Pancreatitis
Hepatitis
Cholecystitis
Pyelonephritis

Investigations
CBC with differential
Serum electrolytes
Pregnancy test
Urinalysis and urine culture
Abdominal x­ray after negative pregnancy test
Serum amylase and lipase
Liver function panel
Abdominal ultrasound
Stool studies

This copy of the material is licensed to Mohammed  Mirza.
Copyright © 2015. All Rights Reserved http://www.uworld.com
case1 Scenario  Print

30 Yr. O/F Complaining of Abdominal Pain
Vitals

Pulse­­98/min
B.P­­120/75 mm of Hg
Temp­101.3
R.rate­­22/min

Make a mental checklist of Differential Diagnosis

Pelvic inflammatory disease
Pelvic abscess
Endometriosis
Urinary tract infection
Appendicitis
Rupture/torsion of ovarian cyst
Acute cholecystitis
Renal colic
Ectopic pregnancy
Abortion
Acute gastroenteritis
Inflammatory bowel disease

This copy of the material is licensed to Mohammed  Mirza.
Copyright © 2015. All Rights Reserved http://www.uworld.com
case1 SP  Print

If the doctor asks you anything other than these just say 'no' (or) say things that
are normal in daily routine life.

You are Mrs. Mary, age: 30yrs
Have abdominal pain since 12 hrs
Started slowly, progressively increasing
6­7/10 in severity
Right below the umbilicus
It's a type of sharp pain
All over your lower abdomen
Began after eating a large meal
Moving around makes it worse
No alleviating factors
Associated with nausea and vomiting
Passing urine more number of times and have burning urination
No bowel problems
Last menstrual period was 3 weeks ago
No discharge from vagina/no bleeding from vagina
Have fever since yesterday associated with chills and rigors
Have one episode of urinary tract infections (UTI) in the past
No allergies
Once hospitalized for evaluation of UTI
Have multiple sexual partners
Using oral contraceptive pills
Familie's health is normal
Smoking ­ No
Alcohol ­ No
Recreational drugs­ No
Occupation: Working as a receptionist
Appetite and wt is normal
No illicit drug intake

Ask this qt ­ Doc is it an appendicitis?

This copy of the material is licensed to Mohammed  Mirza.
Copyright © 2015. All Rights Reserved http://www.uworld.com
case1 Pt Notes  Print

Clinical Skills Evaluation
Case 1 Patient Note

History:  Describe the history you just obtained from this patient.  Include only
information (pertinent positives and negatives) relevant to this patient’s problem(s).

30 YO female with 12 hours of abdominal pain and previously well.
Ate a heavy meal at a local barbecue restaurant one day ago.
Developed acute onset of periumbilical abdominal pain 12 hours later described as
sharp, lasting a few minutes at a time, radiates diffusely in the abdomen, made
worse with movement, and relieved with rest.
Also notes dysuria, increased urinary frequency, fever to 100.5 F, chills, nausea,
and one episode of non­bloody and non­bilious vomiting.

ROS:  No chest pain, shortness of breath, diarrhea, constipation
PMHx:  Previous UTI with possible pyelonephritis
PSHx:  none
FHx:  Noncontributory
SHx:  Has had 2 sexual partners with unprotected intercourse in the past month

Physical Examination:  Describe any positive and negative findings relevant to this
patient’s problem(s).  Be careful to include only those parts of examination performed in
this encounter.

Vital signs:  temperature of 101.3 F, blood pressure of 120/75, heart rate of
98/min, and respiratory rate of 22/min
Abdominal exam shows tenderness in periumbilical, RLQ, and LLQ regions.
No abdominal distention or guarding
Normoactive bowel sounds
Negative Murphy’s sign, Rovsing’s sign, psoas sign, and obturator sign
No CVA tenderness
Lungs are clear to auscultation.  Heart sounds are normal and there are no
murmurs.

Data interpretation:  Based on what you have learned from the history and physical
examination, list up to 3 diagnoses that might explain this patient’s complaint(s).  List
your diagnoses from most to least likely.  For some cases, fewer than 3 diagnoses will be
appropriate.  Then, enter the positive or negative findings from the history and the
physical examination (if present) that support each diagnosis.  Lastly, list initial
diagnostic studies (if any) you would order for each listed diagnosis (e.g., restricted
physical examination maneuvers, laboratory tests, imaging, ECG, etc.).

Diagnosis #1:  Appendicitis

History finding(s) Physical Exam finding(s)
Diffuse abdominal pain Diffuse abdominal tenderness
Fever, chills Fever
Nausea, vomiting  
Diagnosis #2:  UTI

History finding(s) Physical Exam finding(s)
Dysuria Fever
Increased urinary frequency  
Fever  

Diagnosis #3:  Pelvic Inflammatory Disease

History finding(s) Physical Exam finding(s)
Multiple sexual partners with unprotected
Diffuse abdominal tenderness
intercourse
Nausea, vomiting Fever

Diagnostic Studies
Pregnancy test
Urinalysis with urine culture
CT scan of abdomen

This copy of the material is licensed to Mohammed  Mirza.
Copyright © 2015. All Rights Reserved http://www.uworld.com
case2 Scenario  Print

27 Yr. O/F complaining of rash

Vitals

Pulse­­78/min
B.P­­120/75 mm of Hg
Temp­98.3 F
R.rate­­22/min

Make a mental checklist of Differential Diagnosis

Infections
Insect borne diseases
SLE
Photo dermatitis
Drug induced
Occupational exposure
Rheumatoid arthritis
Other autoimmune diseases

This copy of the material is licensed to Mohammed  Mirza.
Copyright © 2015. All Rights Reserved http://www.uworld.com
case2 SP  Print

If the doctor asks you anything other than these just say 'no' (or) say things that
are normal in daily routine life.

You are a 27­year­old female who presents with 1 week of rash.

History of Present Illness:
You are a 27­year­old female with no past medical history who presents with 1 week of
rash.  You were well until 1 week ago when you worked in the garden for 3 hours.  A few
hours later, you developed a flat rash on the face and neck with no itching, numbness,
burning sensation, or tenderness.

The rash is getting larger, but it is not affecting any new areas of the body.  Also, the
rash has remained flat without any blisters.  It seems to be worse with exposure to the
sun and nothing seems to improve the rash.  You also noted 4 days of joint pain and
stiffness in the early morning that lasts about 1 hour and seems to improve afterwards.

You also have subjective fever but did not check your temperature.  You have no chills,
eye redness, chest pain, shortness of breath, nausea, vomiting, diarrhea, constipation, or
urinary symptoms.  You also have not had any recent travel or sick contacts.

Past medical history:
You have had joint pains in the past on and off that seem to go away on their own.
You have had no pregnancies in the past

Past surgical history:  None

Medications:  None, except for aspirin 7 days ago for mild headache

Family history:
Father is 55 and healthy
Mother is 54 with “rheumatism”
Sister is 28 with hypothyroidism

Social history:
You work in a chemical manufacturing company and live alone.  You have had 1 sexual
partner in the last month and use condoms regularly.  You don't smoke, drink, or do any
illegal drugs.  You have not had transfusions or tattoos.  Last menstrual period was 2
weeks ago.

Physical Exam:
The following are your temperature (T), blood pressure (BP), heart rate (HR), and
respiratory rate (RR).

Vital signs:  T 98.3 F, BP 120/75, HR 78/min, RR 22/min

Skin:  Multiple well­circumscribed lesions on face and neck without vesicles; no
tenderness to touch

The student needs to do a skin examination at the minimum, and you should not have
any pain around the rash.

The student may perform a heart and/or lung exam, but these will be normal.

This copy of the material is licensed to Mohammed  Mirza.
Copyright © 2015. All Rights Reserved http://www.uworld.com
case2 Pt Notes  Print

Clinical Skills Evaluation
Case 2 Patient Note

History:  Describe the history you just obtained from this patient.  Include only
information (pertinent positives and negatives) relevant to this patient’s problem(s).

27 YO female with 1 week of rash; she was previously well
Did 3 hours of gardening 1 week ago and developed a rash on her face and neck a
few hours later
Rash has increased in size but does not involve other areas of body
Rash is worse with sun exposure and not improved with anything
Also notes morning joint pains and subjective fever

ROS:  No chest pain, shortness of breath, diarrhea, constipation, sick contacts, recent
travel
PMHx:  Previous joint pain and stiffness episodes in the past with spontaneous resolution
PSHx:  none
FHx:  Mother has possible rheumatoid arthritis
SHx:  Has 1 sexual partner and uses condoms

Physical Examination:  Describe any positive and negative findings relevant to this
patient’s problem(s).  Be careful to include only those parts of examination performed in
this encounter.

Vital signs:  temperature of 98.3 F, blood pressure of 120/75, heart rate of 78/min,
and respiratory rate of 22/min
Face and neck have multiple, well­circumscribed, erythematous macules without
tenderness to palpation
No scaliness, vesicles, or cysts noted in rash area
Normal range of motion in all joints without tenderness, edema, or erythema
HEENT with no pallor, jaundice, or eye lesions
Lungs:  CTA bilaterally
Heart:  RRR without M/G/R

Data interpretation:  Based on what you have learned from the history and physical
examination, list up to 3 diagnoses that might explain this patient’s complaint(s).  List
your diagnoses from most to least likely.  For some cases, fewer than 3 diagnoses will be
appropriate.  Then, enter the positive or negative findings from the history and the
physical examination (if present) that support each diagnosis.  Lastly, list initial
diagnostic studies (if any) you would order for each listed diagnosis (e.g., restricted
physical exam maneuvers, laboratory tests, imaging, ECG, etc.).

Diagnosis #1:  SLE

History finding(s) Physical Exam finding(s)
Cutaneous photosensitive rash Discoid lupus rash
Fever  
Joint pain  
Diagnosis #2:  Rheumatoid arthritis

History finding(s) Physical Exam finding(s)
Joint pain and stiffness  
Fever  
Family history  

Diagnosis #3:  Photodermatitis

History finding(s) Physical Exam finding(s)
History of sun exposure followed by rash Photosensitive rash on face and neck
  No progression of rash to other areas

Diagnostic Studies
ANA and anti­ds DNA
Rheumatoid factor and ESR
Skin biopsy

This copy of the material is licensed to Mohammed  Mirza.
Copyright © 2015. All Rights Reserved http://www.uworld.com
case3 Scenario  Print

65­year­old female complaining of arm and leg weakness

Vitals:
PR: 78/min
BP: 160/90 mmHg
Temp: 98.3 F (36.7 C)
RR: 16/min

Make a mental checklist of Differential Diagnosis:
Stroke
Transient Ischemic Attack (TIA)
Hypoglycemia
Subarachnoid hemorrhage
Subdural hematoma
Intracranial mass
Guillain Barrn syndrome
Complex migraine
Conversion disorder

This copy of the material is licensed to Mohammed  Mirza.
Copyright © 2015. All Rights Reserved http://www.uworld.com
case3 SP  Print

If the student asks you anything other than these, just say 'no,' (or) say things
that are normal in daily routine life.

You are a 65­year­old female who presents with 1 hour of right arm and leg weakness.

History of Present Illness:
You are a 65­year­old woman who presents with 1 hour of weakness in the right arm and
leg that gradually worsened to involve numbness in the entire right arm and right leg. 
You also noticed a mild headache that is diffuse and is a 5/10 in severity and nausea
without vomiting.

You have not had these symptoms in the past and also have no slurred speech, fever,
blurry or double vision, problems with swallowing, chest pain, urinary symptoms,
diarrhea, constipation, seizures, or palpitations.  You have not passed out or fallen down
recently.

Past medical history:
High blood pressure for 25 years
Heart attack 6 years ago
High cholesterol

Past medical history:
Heart bypass surgery 6 years ago

Medications:
Simvastatin 20 mg a day
Aspirin 81 mg a day
Atenolol 50 mg a day

Allergies:  None

Family history:
Mother died at age 68 with a heart attack and had high blood pressure
Father died at age 65 with a heart attack and had high blood pressure
Brother age 70 with high blood pressure and high cholesterol

Social history:
You are a widow whose husband died 8 years ago.  You live alone.  Your neighbor Steve,
who is like a son to you, brought you to the hospital.  You smoked 2 PPD for 35 years
and quit 6 years ago.  You drink alcohol once a month and have never used illegal drugs
or had transfusions or tattoos.

Ask this question, if the student does not address about the stroke:  “Doctor, is it a
stroke?”

Physical Exam:
The following are your temperature (T), blood pressure (BP), heart rate (HR), and
respiratory rate (RR).

Vital signs:  T 98.3 F, BP 160/90, HR 78/min, RR 16/min

Neurological:
Cranial nerves 2­12 intact
Motor 5/5 in left upper and lower extremity but 3/5 in right upper and lower extremity,
DTR 3+ on right and 2+ on left
Upgoing toe on right and downgoing toe on left
Unable to perform gait due to inability to stand

The student needs to do a neurological exam at the minimum, and you should be weaker
on the right side of the body.

You should be unable to lift your leg or arm without help from the student.  You also
cannot stand up if the student asks you to.  In addition, your reflexes on the right side
should be more hyper than the left.  The sensation is the same on both sides of the body.

The student may perform a heart and/or lung exam, but these will be normal.

This copy of the material is licensed to Mohammed  Mirza.
Copyright © 2015. All Rights Reserved http://www.uworld.com
case3 Pt Notes  Print

Clinical Skills Evaluation
Case 3 Patient Note

History:  Describe the history you just obtained from this patient.  Include only
information (pertinent positives and negatives) relevant to this patient’s problem(s).

65 YO female with 1 hour of acute onset right­sided weakness and headache
The symptoms have progressed gradually over the past hour
She also has nausea without vomiting
No history of fall or syncope

ROS:  No fever, chest pain, shortness of breath, vision changes, dysarthria, seizures
PMHx:  HTN, hypercholesterolemia, CAD
PSHx:  CABG 6 years ago
Meds:  Aspirin, simvastatin, atenolol
FHx:  Mother died of MI and had HTN, father died of MI and had HTN, brother with HTN
and hypercholesterolemia
SHx:  Smoked 2 PPD for 35 years but quit 6 years ago

Physical Examination:  Describe any positive and negative findings relevant to this
patient’s problem(s).  Be careful to include only those parts of examination performed in
this encounter.

Vital signs:  temperature of 98.3 F (36.8 C), blood pressure of 160/90, heart rate of
78/min, and respiratory rate of 16/min
Lungs:  CTA bilaterally
Heart:  RRR without M/G/R
Neuro:  A&O x 3, CN 2­12 intact, Motor 5/5 on LUE and LLE but 3/5 in RUE and RLE,
sensory grossly intact, DTR 2+ on left but 3+ in RUE and RLE, upgoing toes on right
and downgoing on left, gait unable to be assessed

Data interpretation:  Based on what you have learned from the history and physical
examination, list up to 3 diagnoses that might explain this patient’s complaint(s).  List
your diagnoses from most to least likely.  For some cases, fewer than 3 diagnoses will be
appropriate.  Then, enter the positive or negative findings from the history and the
physical examination (if present) that support each diagnosis.  Lastly, list initial
diagnostic studies (if any) you would order for each listed diagnosis (e.g., restricted
physical examination maneuvers, laboratory tests, imaging, ECG, etc.).

Diagnosis #1:  Evolving stroke

History finding(s) Physical Exam finding(s)
Acute onset weakness Right hemiparesis
Gradually progressing symptoms Right­sided hyperreflexia
  Right Babinski present

Diagnosis #2:  TIA or reversible ischemic neurological deficit
History finding(s) Physical Exam finding(s)
Acute onset weakness Right hemiparesis
  Right­sided hyperreflexia
  Right Babinski present

Diagnosis #3:  Subarachnoid hemorrhage

History finding(s) Physical Exam finding(s)
Headache Right hemiparesis
Nausea Right­sided hyperreflexia
Acute onset weakness Right Babinski present

Diagnostic Studies
CT scan of head without contrast
Transesophageal echocardiogram
Carotid Doppler
CBC with differential

This copy of the material is licensed to Mohammed  Mirza.
Copyright © 2015. All Rights Reserved http://www.uworld.com
case4 Scenario  Print

29 Yr. O/F known sickle cell anemia pt c/o chest pain
Vitals:

P.R: 98/min
B.P: 120/75 mm of Hg
Temp: 101.3F
R.R: 22/min

Make a mental checklist of Differential Diagnosis:

Chest syndrome due to sickle cell anemia
Pneumonia
Costochondritis
Pericarditis
Pulmonary thromboembolism
Salmonella Osteomyelitis
Panic attacks

This copy of the material is licensed to Mohammed  Mirza.
Copyright © 2015. All Rights Reserved http://www.uworld.com
case4 SP  Print

If the student asks you anything other than these, just say 'no,' (or) say things
that are normal in daily routine life.

You are a 29­year­old female who presents with 12 hours of chest pain.

History of Present Illness:
You are a 29­year­old woman with sickle cell anemia presenting with 12 hours of chest
pain in the middle of the chest.  The pain is sharp and started slowly, but has been
increasing over the past 12 hours and is now a 7­8/10 in severity.  The pain does not
move anywhere, but it is worse with any movement and breathing.  You took Tylenol,
which helped reduce the pain a little.

You also have had mild shortness of breath, fever to 101 F for the past 3 days, cough
with green sputum, and chills with rigors.  You have not had chest trauma, nausea,
vomiting, blood in the sputum, leg pain or swelling, abdominal pain, back pain, urinary
symptoms, or diarrhea.

Past medical history:
Sickle cell anemia diagnosed in childhood­ you had mild pain episodes as a child and
have done relatively well in adulthood.  You were admitted once for abdominal pain 5
years ago that resolved with supportive care in the hospital.
You have had no pregnancies in the past.

Past surgical history:  None

Medications:
Birth control pills
Tylenol as needed

Allergies:  None

Family history:
Mother 53 with sickle cell anemia
Father 55 healthy
Brother 25 healthy

Social history:
You are a teacher and live alone.  You are sexually active with your boyfriend and use
condoms.  You have not smoked, drank alcohol, or used illegal drugs.  You had a
transfusion as a child.  Last menstrual period was 2 weeks ago.

Physical Exam:
The following are your temperature (T), blood pressure (BP), heart rate (HR), and
respiratory rate (RR).

Vital signs:  T 101.3 F, BP 120/75, HR 98/min, RR 22/min

Head and Neck:  No redness or exudates in the mouth, no enlarged lymph nodes

Chest/Lungs:  No tenderness to palpation of the chest wall, clear to auscultation
bilaterally

Heart:  Regular rate and rhythm

Abdomen:  Non­tender, non­distended, normoactive bowel sounds throughout; tympanic
to percussion; no hepatosplenomegaly
Extremities:  No cyanosis, clubbing, edema, or tenderness in the legs

The student needs to do the above exams at the minimum, and you should not have any
chest pain when it is palpated.  You should also try to breathe a little faster to a rate
near 20/minute.  The rest of the examination is normal.

This copy of the material is licensed to Mohammed  Mirza.
Copyright © 2015. All Rights Reserved http://www.uworld.com
case4 Pt Notes  Print

Clinical Skills Evaluation
Case 4 Patient Note

History:  Describe the history you just obtained from this patient.  Include only
information (pertinent positives and negatives) relevant to this patient’s problem(s).

29 YO female with history of sickle cell anemia with 12 hours of acute onset chest
pain
The pain has gradually increased to 7­8/10 intensity over the past 12 hours
Pain is worse with movement and respiration, improved with Tylenol
3 days of fever to 101 F, productive cough of green sputum, and mild shortness of
breath

ROS:  No nausea, vomiting, trauma to chest, dysuria, diarrhea, leg swelling, or leg pain
PMHx:  Admitted once for abdominal pain that resolved 5 years ago
PSHx:  None
Meds:  Birth control pills, Tylenol PRN
FHx:  Mother has sickle cell anemia
SHx:  Denies smoking, alcohol, illegal drugs.  She had a blood transfusion as a child.

Physical Examination:  Describe any positive and negative findings relevant to this
patient’s problem(s).  Be careful to include only those parts of examination performed in
this encounter.

Vital signs:  temperature of 101.3 F, blood pressure of 120/75, heart rate of
98/min, and respiratory rate of 22/min
HEENT:  Oropharynx without erythema or exudates
Neck:  No cervical lymphadenopathy, trachea midline
Lungs:  CTA bilaterally, no chest tenderness to palpation
Heart:  RRR without M/G/R
Abdomen:  Non­tender, non­distended, normoactive bowel sounds throughout;
tympanic to percussion; no hepatosplenomegaly
Extremities:  No cyanosis, clubbing, or edema.  No swelling or tenderness in the
legs.

Data interpretation:  Based on what you have learned from the history and physical
examination, list up to 3 diagnoses that might explain this patient’s complaint(s).  List
your diagnoses from most to least likely.  For some cases, fewer than 3 diagnoses will be
appropriate.  Then, enter the positive or negative findings from the history and the
physical examination (if present) that support each diagnosis.  Lastly, list initial
diagnostic studies (if any) you would order for each listed diagnosis (e.g., restricted
physical exam maneuvers, laboratory tests, imaging, ECG, etc.).

Diagnosis #1:  Acute chest syndrome

History finding(s) Physical Exam finding(s)
Acute onset chest pain No chest pain to palpation
Shortness of breath Fever
History of sickle cell anemia  

Diagnosis #2:  Pneumonia

History finding(s) Physical Exam finding(s)
Fever Fever
Productive cough of green sputum Tachypnea
Shortness of breath  

Diagnosis #3:  Pulmonary embolism

History finding(s) Physical Exam finding(s)
Pleuritic chest pain Fever
Fever No chest tenderness to palpation
Shortness of breath  
Birth control pills  

Diagnostic Studies
Chest x­ray
ABG
Blood cultures
CBC with differential

This copy of the material is licensed to Mohammed  Mirza.
Copyright © 2015. All Rights Reserved http://www.uworld.com
case5 Scenario  Print

35­year­old male with recent onset cough

Vitals:
PR:        98/min, regular
BP:        120/75 mmHg
Temp:    38.3 C (101.0 F)
RR:        20/min

Differential diagnosis for recent onset Cough:
Common cold
Acute sinusitis
Allergic rhinitis
Acute bronchitis
Pneumonia
Pertussis
Pulmonary embolism
Drugs (ACE inhibitors)
Asthma

In elderly patients also consider:
Congestive heart failure
Chronic obstructive pulmonary disease (COPD) exacerbation

This copy of the material is licensed to Mohammed  Mirza.
Copyright © 2015. All Rights Reserved http://www.uworld.com
case5 SP  Print

If the doctor asks you anything other than these, just say 'no,' or say things that
are normal in daily routine life.

You are a 35­year­old male who presents with 5 days of cough.

History of Present Illness:
You are a 35­year­old male with history of seasonal allergies and mild asthma who
presents with 5 days of cough productive of yellow sputum.  You were well until 7 days
ago when you noted a sore throat, sinus congestion, runny nose, mild frontal headache,
and subjective fever to 100.2 degrees.  The symptoms persisted and you then developed
a worsening cough that is keeping you awake at night.  You are coughing up a
teaspoonful of sputum each time.

You are also still having a mild frontal headache that is slightly worse with lying down. 
There is no blood in the sputum, chills, night sweats, chest pain, shortness of breath,
wheezing, or abdominal pain.  There was some slight relief with Tylenol and Robitussin
DM.  Your 8­year­old son was also recently sick.

Past medical history:
Asthma ­ had it since childhood with mild intermittent symptoms.
Seasonal allergies­ mainly in the Fall and Spring.

Past surgical history:  None

Medications:  Albuterol inhaler, 1­2 puffs as needed

Allergies:  No medications, but you are allergic to cats.

Family history:
Father is 60 years old and has asthma
Mother is 59 and is healthy
You have no other siblings

Social history:
You live with your husband and son.  You currently work as a paramedic.  You have
smoked 1 PPD for the past 10 years and drink occasionally.  You have never used illegal
drugs and also have not had any blood transfusions.

Physical Exam:
The following are your temperature (T), blood pressure (BP), heart rate (HR), and
respiratory rate (RR).

Vital signs:  T 101.0 F, BP 120/75, HR 98/min, RR 20/min

Head and Neck:  No redness or exudates in the mouth, no sinus tenderness to
percussion, no enlarged lymph nodes

Chest/Lungs:  Clear to auscultation bilaterally

Heart:  Regular rate and rhythm

The student needs to do the above exams at the minimum, and you should not have
significant sinus pain when they are palpated.  You should also try to breathe a little
faster to a rate near 20/minute.  The rest of the examination is normal.

This copy of the material is licensed to Mohammed  Mirza.
Copyright © 2015. All Rights Reserved http://www.uworld.com
case5 Pt Notes  Print

Clinical Skills Evaluation
Case 5 Patient Note

History:  Describe the history you just obtained from this patient.  Include only
information (pertinent positives and negatives) relevant to this patient’s problem(s).

35 YO male with 5 days of worsening cough
He initially developed sore throat, fever, rhinorrhea, and frontal headache 7 days
ago
Cough is productive of yellow sputum without hemoptysis, about 1 teaspoonful
Recent sick contact­ 8 YO son

ROS:  No chills, night sweats, chest pain, shortness of breath, wheezing, or abdominal
pain.
PMHx:  Mild asthma, seasonal allergic rhinitis
PSHx:  None
Meds:  Albuterol MDI PRN
Allergies:  Cats
FHx:  Father has asthma
SHx:  Smokes 1 PPD for past 10 years and drinks occasionally.

Physical Examination:  Describe any positive and negative findings relevant to this
patient’s problem(s).  Be careful to include only those parts of examination performed in
this encounter.

Vital signs:  temperature of 101.0 F, blood pressure of 120/75, heart rate of
98/min, and respiratory rate of 20/min
HEENT:  Oropharynx without erythema or exudates, no sinus tenderness to
percussion
Neck:  No cervical lymphadenopathy, trachea midline
Lungs:  CTA bilaterally
Heart:  RRR without M/G/R

Data interpretation:  Based on what you have learned from the history and physical
examination, list up to 3 diagnoses that might explain this patient’s complaint(s).  List
your diagnoses from most to least likely.  For some cases, fewer than 3 diagnoses will be
appropriate.  Then, enter the positive or negative findings from the history and the
physical examination (if present) that support each diagnosis.  Lastly, list initial
diagnostic studies (if any) you would order for each listed diagnosis (e.g., restricted
physical exam maneuvers, laboratory tests, imaging, ECG, etc.).

Diagnosis #1:  Acute sinusitis

History finding(s) Physical Exam finding(s)
History of allergic rhinitis Fever
Frontal headache  
Fever  
Diagnosis #2:  Pneumonia

History finding(s) Physical Exam finding(s)
Fever Fever
Productive cough of yellow sputum Tachypnea
Smoking history  

Diagnosis #3:  Acute bronchitis

History finding(s) Physical Exam finding(s)
Cough Fever
Fever  
Smoking history  

Diagnostic Studies
Chest x­ray
Sputum Gram stain and cultures
CBC with differential

This copy of the material is licensed to Mohammed  Mirza.
Copyright © 2015. All Rights Reserved http://www.uworld.com
case6 Scenario  Print

50­year­old male complaining of fatigue and loss of weight

Vitals:
PR: 78/min
BP: 120/76 mmHg
Temp: 98.0 F (36.7 C)
RR: 18/min

Make a mental checklist of DD for weight loss:
Malignancy
Diabetes mellitus
Hyperthyroidism
Depression
Infections like TB, HIV
Malabsorption
Addison's disease

This copy of the material is licensed to Mohammed  Mirza.
Copyright © 2015. All Rights Reserved http://www.uworld.com
case6 Pt Notes  Print

Clinical Skills Evaluation
Case 6 Patient Note

History:  Describe the history you just obtained from this patient.  Include only
information (pertinent positives and negatives) relevant to this patient’s problem(s).

50 YO male with 5 months of increased fatigue
Spouse died 3 months ago and he has had worsened symptoms
Has had a 30­pound weight loss, decreased appetite, peri­umbilical abdominal pain,
early satiety
He has loss of interest in activities and terminal insomnia, but is not suicidal

ROS:  No dysphagia, fever, chills, night sweats, chest pain, shortness of breath, or
cough.
PMHx:  None
PSHx:  None
Meds:  None
Allergies:  None
FHx:  Mother died at age 60 from pancreatic cancer, father died at age 55 from heart
attack
SHx:  Denies smoking

Physical Examination:  Describe any positive and negative findings relevant to this
patient’s problem(s).  Be careful to include only those parts of examination performed in
this encounter.

Vital signs:  temperature of 98.0 F, blood pressure of 120/76, heart rate of 78/min,
and respiratory rate of 18/min
Head and Neck:  No redness or exudates in the mouth, no enlarged lymph nodes,
no jaundice, no thyromegaly
Chest/Lungs:  No tenderness to palpation of the chest wall, clear to auscultation
bilaterally
Heart:  Regular rate and rhythm
Abdomen:  Non­tender, non­distended, normoactive bowel sounds throughout;
tympanic to percussion; no hepatosplenomegaly
Extremities:  No cyanosis, clubbing, or edema
Neurological:  Motor 5/5 throughout, DTR 2+ bilaterally

Data interpretation:  Based on what you have learned from the history and physical
examination, list up to 3 diagnoses that might explain this patient’s complaint(s).  List
your diagnoses from most to least likely.  For some cases, fewer than 3 diagnoses will be
appropriate.  Then, enter the positive or negative findings from the history and the
physical examination (if present) that support each diagnosis.  Lastly, list initial
diagnostic studies (if any) you would order for each listed diagnosis (e.g., restricted
physical exam maneuvers, laboratory tests, imaging, ECG, etc.).

Diagnosis #1:  GI malignancy (ex:  colon cancer, gastric cancer)

History finding(s) Physical Exam finding(s)
Fatigue None
Weight loss  
Early satiety  

Diagnosis #2:  Depression

History finding(s) Physical Exam finding(s)
Fatigue None
Weight loss  
Death of spouse  
Terminal insomnia  

Diagnosis #3:  Hyperthyroidism

History finding(s) Physical Exam finding(s)
Weight loss None
Fatigue  

Diagnostic Studies
TSH and T4
CBC with differential
Rectal examination with FOBT
Colonoscopy
CT scan of the abdomen

This copy of the material is licensed to Mohammed  Mirza.
Copyright © 2015. All Rights Reserved http://www.uworld.com
case6 SP  Print

If the doctor asks you anything other than these just say 'no,' (or) say things that
are normal in daily routine life.

You are a 50­year­old male who presents with 5 months of fatigue.

History of Present Illness:
You are a 50­year­old male who presents with 5 months of generalized weakness and
fatigue.  It initially started as a mild decrease in energy but has been getting worse over
the past 3 months.  You also notice abdominal pain around the belly button described as
a gas feeling that has been happening on and off during this time.  You seem to get full
easier after eating less food.

(You need to have the student ask the following and only volunteer if asked)

Your symptoms got worse after your spouse died 3 months ago.  You started having
decreased appetite, a 30­pound weight loss, increasing fatigue, decreased interest in
activities, and difficulty falling asleep at night.  You also have been getting up at night
and have been unable to get back to sleep during this time.  You have not thought about
suicide, but you do have feelings of guilt.  You also have difficulty concentrating on tasks.

You do not have fever, chills, nausea, vomiting, chest pain, shortness of breath, jaundice,
diarrhea, constipation, or tremors.  You think that your family does not understand what
you are going through and you feel isolated from many of your friends.

Past medical history:  None

Past surgical history:  None

Medications:  None

Allergies:  None

Family history:
Mother died at age 60 of pancreatic cancer
Father died at age 55 of heart attack
You have no siblings

Social history:
You currently live alone and are a widow.  You have 2 children who are ages 28 and 25. 
Your spouse died 3 months ago and you work as a manager in a local restaurant (not
stressful).  You do not smoke but drink occasionally.  You have not used any illegal
drugs.  You have not had transfusions or tattoos.

Physical Exam:
The following are your temperature (T), blood pressure (BP), heart rate (HR), and
respiratory rate (RR).

Vital signs:  T 98.0 F, BP 120/76, HR 78/min, RR 18/min

Head and Neck:  No redness or exudates in the mouth, no enlarged lymph nodes, no
jaundice, no thyromegaly

Chest/Lungs:  No tenderness to palpation of the chest wall, clear to auscultation
bilaterally

Heart:  Regular rate and rhythm
Abdomen:  Non­tender, non­distended, normoactive bowel sounds throughout; tympanic
to percussion; no hepatosplenomegaly

Extremities:  No cyanosis, clubbing, edema

Neurological:  Motor 5/5 throughout, DTR 2+ bilaterally

The student needs to do the above exams at the minimum, and are all normal.

This copy of the material is licensed to Mohammed  Mirza.
Copyright © 2015. All Rights Reserved http://www.uworld.com
case7 Scenario  Print

35­year­old male with acute onset diarrhea

Vitals:
PR: 100/min, regular
BP: 110/65 mmHg
RR: 18/min
Temp: 98.0 F (36.7 C)

Make a mental checklist of DD for acute onset diarrhea:
Viral gastroenteritis
Bacterial gastroenteritis
Medication induced
Clostridium difficle colitis
Inflammatory bowel disease
Irritable bowel disease
Malabsorption
HIV

This copy of the material is licensed to Mohammed  Mirza.
Copyright © 2015. All Rights Reserved http://www.uworld.com
case7 Pt Notes  Print

Clinical Skills Evaluation
Case 7 Patient Note

History:  Describe the history you just obtained from this patient.  Include only
information (pertinent positives and negatives) relevant to this patient’s problem(s).

35 YO male with 1 day of diarrhea with 6­7 loose BM/day without blood or mucus
Ate seafood and salad at a local restaurant 6­8 hours before symptom onset
Recent sinus infection treated with amoxicillin, last dose 2 days ago
Developed diffuse crampy abdominal pain, nausea, vomiting, decreased PO intake
Two other friends that ate at the restaurant have the same symptoms

ROS:  No fever, chills, chest pain, shortness of breath, burning with urination, recent
travel, or back pain.
PMHx:  None
PSHx:  None
Meds:  None
Allergies:  None
FHx:  Noncontributory
SHx:  Denies smoking and alcohol use

Physical Examination:  Describe any positive and negative findings relevant to this
patient’s problem(s).  Be careful to include only those parts of examination performed in
this encounter.

Vital signs:  temperature of 98.0 F, blood pressure of 110/65, heart rate of
100/min, and respiratory rate of 18/min
Head and Neck:  Oropharynx with dry mucous membranes but no erythema or
exudates, no enlarged lymph nodes, no jaundice
Chest/Lungs:  Clear to auscultation bilaterally
Heart:  Regular rate and rhythm without M/G/R
Abdomen:  Non­tender, non­distended, normoactive bowel sounds throughout;
tympanic to percussion; no hepatosplenomegaly

Data interpretation:  Based on what you have learned from the history and physical
examination, list up to 3 diagnoses that might explain this patient’s complaint(s).  List
your diagnoses from most to least likely.  For some cases, fewer than 3 diagnoses will be
appropriate.  Then, enter the positive or negative findings from the history and the
physical examination (if present) that support each diagnosis.  Lastly, list initial
diagnostic studies (if any) you would order for each listed diagnosis (e.g., restricted
physical exam maneuvers, laboratory tests, imaging, ECG, etc.).

Diagnosis #1:  Viral gastroenteritis

History finding(s) Physical Exam finding(s)
Diarrhea without fever No fever
No blood in stool  
Other sick friends who also ate at same place  
Diagnosis #2:  Bacterial gastroenteritis

History finding(s) Physical Exam finding(s)
Symptom onset 6­8 hours after eating at
None
restaurant
Nausea, vomiting  

Diagnosis #3:  Clostridium difficile colitis

History finding(s) Physical Exam finding(s)
Symptoms started 2 days after recent
None
amoxicillin use
No blood in stool  

Diagnostic Studies
Stool for leukocytes
Stool for Clostridium difficile
Rectal examination with FOBT

This copy of the material is licensed to Mohammed  Mirza.
Copyright © 2015. All Rights Reserved http://www.uworld.com
case7 SP  Print

If the doctor asks you anything other than these, just say 'no,' or say things that
are normal in daily routine life.

You are a 35­year­old male who presents with 1 day of diarrhea.

History of Present Illness:
You are a 35­year­old male who presents with 1 day of diarrhea.  You ate at a local
seafood restaurant one day ago and had salad and some shrimp.  You started having
diarrhea 6­8 hours later.  You have had 6­7 bowel movements a day described as loose,
watery, and without blood or mucus.  Nothing seems to worsen the diarrhea, and you
have not taken any medications to improve it.

You also have diffuse, crampy abdominal discomfort.  You later started having nausea,
vomiting, and difficulty keeping liquids and solids down.  You have not had fever, chills,
chest pain, shortness of breath, urinary problems, or back pain.  A couple of your friends
who ate with you also have had similar symptoms.  You recently had a sinus infection 2
weeks ago treated with amoxicillin; you took your last dose 2 days ago.

You do not have fever, chills, chest pain, shortness of breath, burning with urination, or
back pain.  You have not had any recent travel.

Past medical history:  None

Past surgical history:  None

Medications:  None

Allergies:  None

Family history:  Both of your parents and your siblings are healthy.

Social history:
You live alone and work as a computer programmer.  You do not smoke, drink, or do
illegal drugs.  You have not had transfusions or tattoos.

Physical Exam:
The following are your temperature (T), blood pressure (BP), heart rate (HR), and
respiratory rate (RR).

Vital signs:  T 98.0 F, BP 110/65, HR 100/min, RR 18/min

Head and Neck:  No redness or exudates in the mouth, dry mucous membranes, no
enlarged lymph nodes, no jaundice

Chest/Lungs:  Clear to auscultation bilaterally

Heart:  Regular rhythm without murmurs, gallops, or rubs

Abdomen:  Non­tender, non­distended, normoactive bowel sounds throughout; tympanic
to percussion; no hepatosplenomegaly

The doctor needs to do the above exams at the minimum, and the dry mucous
membranes are the main finding indicating dehydration.

This copy of the material is licensed to Mohammed  Mirza.
Copyright © 2015. All Rights Reserved http://www.uworld.com
case8 Scenario  Print

25­year­old female complains of sore throat

Vitals:
PR: 90/min, regular
BP: 120/70 mmHg
RR: 16/min
Temp: 101.0 F (38.3 C)

Make a mental checklist of DD for sore throat:
Viral pharyngitis (rhino virus and influenza)
Bacterial pharyngitis
Group A Streptococcal pharyngitis
Mycoplasma pneumonia
Neisseria gonorrhea
EBV mononucleosis
Postnasal drip secondary to rhinitis
Chronic tonsillitis
Primary HIV

This copy of the material is licensed to Mohammed  Mirza.
Copyright © 2015. All Rights Reserved http://www.uworld.com
case08 Pt Notes  Print

Clinical Skills Evaluation
Case 8 Patient Note

History:  Describe the history you just obtained from this patient. Include only
information (pertinent positives and negatives) relevant to this patient’s problem(s).

25 YO female with 3 days of sore throat
Initially started with sore throat, sinus congestion, runny nose, fever, and dry
cough
Sore throat has worsened with pain when swallowing
Sick contact (boyfriend) had similar symptoms that resolved

ROS:  No chills, chest pain, shortness of breath, nausea, vomiting, or abdominal pain
PMHx:  None
PSHx:  None
Meds:  None
Allergies:  None
FHx:  Noncontributory
SHx:  Denies smokingand alcohol use

Physical Examination:  Describe any positive and negative findings relevant to this
patient’s problem(s).  Be careful to include only those parts of examination performed in
this encounter.

Vital signs:  temperature of 101.0 F (38.3 C), blood pressure of 120/70, heart rate
of 90/min, and respiratory rate of 16/min
HEENT:  Oropharynx withtonsillar exudates, TM clear bilaterally
Neck:  No enlarged lymph nodes
Chest/Lungs:  Clear to auscultation bilaterally
Heart:  Regular rate and rhythm without M/G/R
Abdomen:  Non­tender, non­distended, normoactive bowel sounds throughout;
tympanic to percussion; no hepatosplenomegaly

Data interpretation:  Based on what you have learned from the history and physical
examination, list up to 3 diagnoses that might explain this patient’s complaint(s).  List
your diagnoses from most to least likely.  For some cases, fewer than 3 diagnoses will be
appropriate.  Then, enter the positive or negative findings from the history and the
physical examination (if present) that support each diagnosis.  Lastly, list initial
diagnostic studies (if any) you would order for each listed diagnosis (e.g., restricted
physical exam maneuvers, laboratory tests, imaging, ECG, etc.).

Diagnosis #1:  Bacterial pharyngitis

History finding(s) Physical Exam finding(s)
Sore throat Fever
Fever Tonsillar exudates

Diagnosis #2:  Viral pharyngitis
History finding(s) Physical Exam finding(s)
Fever Fever
Sore throat  
Boyfriend with similar illness that resolved  

Diagnostic Studies
Rapid strep test
Throat culture if rapid strep negative

This copy of the material is licensed to Mohammed  Mirza.
Copyright © 2015. All Rights Reserved http://www.uworld.com
case8 SP  Print

If the doctor asks you anything other than these, just say 'no,' or say things that
are normal in daily routine life.

You are a 25­year­old female who presents with sore throat.

History of Present Illness:
You are a 25­year­old female who presents with 3 days of sore throat.  You initially had
headache, nasal congestion, sore throat, dry cough, fever to 100.5, and some pain with
swallowing food.  You then started having some body aches and tried Tylenol with mild
relief.  Your boyfriend also had similar symptoms 2 weeks ago and is better now.

You have no abdominal pain, pelvic pain, rash, chills, chest pain, or shortness of breath. 
You had frequent sore throat episodes as a child, but have only had this twice in the past
2 years.

Past medical history:
You had frequent episodes of tonsillitis in childhood.  You have never been pregnant.

Past surgical history:  None

Medications:  None

Allergies:  None

Family history:  Your parents and sister are healthy

Social history:  You are a college student.  You do not smoke, drink, or do illegal drugs. 
You have not had transfusions or tattoos.  You are sexually active with your boyfriend
and use condoms.  Your last menstrual period was 2 weeks ago.

Physical Exam:
The following are your temperature (T), blood pressure (BP), heart rate (HR), and
respiratory rate (RR).

Vital signs:  T 101.0 F, BP 120/70, HR 90/min, RR 16/min

Head and Neck:  Oropharynx with tonsillar exudates, tympanic membranes clear
bilaterally, no sinus tenderness to palpation, no enlarged lymph nodes, no jaundice.

Chest/Lungs:  Clear to auscultation bilaterally.

Heart:  Regular rhythm without murmurs, gallops, or rubs.

Abdomen:  Non­tender, non­distended, normoactive bowel sounds throughout; tympanic
to percussion; no hepatosplenomegaly.

The doctor needs to do the above exams at the minimum, and should note exudates in
the mouth.  They must also look in the ears.  The rest of the examination is normal.

This copy of the material is licensed to Mohammed  Mirza.
Copyright © 2015. All Rights Reserved http://www.uworld.com
case9 Scenario  Print

56­year­old male for BP Check and refill of the medications

Vitals:
PR: 80/min
BP: 150/90 mmHg
Temp: 97.0 F (36.1 C)
RR: 16/min

Make a metal checklist of complications of BP:
Diastolic congestive cardiac failure
Coronary artery disease (angina)
Peripheral vascular disease
Retinopathy
Side affects of the medications

This copy of the material is licensed to Mohammed  Mirza.
Copyright © 2015. All Rights Reserved http://www.uworld.com
case09 Pt Notes  Print

Clinical Skills Evaluation

Case 9 Patient Notes

History:  Describe the history you just obtained from this patient.  Include only
information (pertinent positives and negatives) relevant to this patient’s problem(s).

56­YO man here for BP check and medication refill
Noticed BP ranges from 140­150 systolic and 80­90 diastolic at home 3x/week in
morning and evening
Is not compliant with low­salt diet and does not exercise regularly
Compliant with medications
Developed cough with ACE inhibitor, ED with beta blockers

ROS:  No headaches, palpitations, blurry vision, chest pain, shortness of breath, nose
bleeds, dizziness, or leg swelling
PMHx:  Hypertension and hypercholesterolemia for past 10 years
PSHx:  None
Meds:  HCTZ 50 mg a day, simvastatin 20 mg a day
Allergies:  None
FHx:  Father with hypertension, mother with diabetes
SHx:  Smokes 1 PPD for 30 years, drinks occasionally

Physical examination:  Describe any positive and negative findings relevant to this
patient’s problem(s). Be careful to include only those parts of examination performed in
this encounter.

Vital signs:  Temperature, 98.6 F (37.0 C); blood pressure, 150/90 mm Hg in both
arms; heart rate, 90/min; and respiratory rate, 16/min
HEENT:  PERRLA, EOMI.  Fundi without papilledema, exudates, or AV nicking
Neck:  No enlarged lymph nodes, no bruits
Chest/lungs:  Clear to auscultation bilaterally
Heart:  Regular rate and rhythm without M/G/R
Extremities:  No cyanosis, clubbing, or edema.  Pulses 2+ bilaterally in carotid,
radial, posterior tibialis

Data interpretation:  Based on what you have learned from the history and physical
examination, list up to 3 diagnoses that might explain this patient’s complaint(s).  List
your diagnoses from most to least likely.  For some cases, fewer than 3 diagnoses will be
appropriate.  Then enter the positive or negative findings from the history and the
physical examination (if present) that support each diagnosis.  Finally, list initial
diagnostic studies (if any) you would order for each listed diagnosis (eg, restricted
physical examination maneuvers, laboratory tests, imaging, ECG)

Diagnosis #1:  Essential hypertension, suboptimally controlled

History finding(s) Physical examination finding(s)
Hypertension Elevated blood pressure to 150/90
Poor dietary compliance No neck bruits
No regular exercise Symmetrical peripheral pulses
  Normal funduscopic examination

Diagnostic studies
Serum electrolytes to monitor creatinine
Lipid panel

This copy of the material is licensed to Mohammed  Mirza.
Copyright © 2015. All Rights Reserved http://www.uworld.com
case9 SP  Print

If the doctor asks you any questions other than these, just say “no” or “things are
normal in my daily life.”

You are a 56­year­old man who needs a blood pressure recheck and medication refill.

History of present illness:
You are a 56­year­old man who comes to the physician for a blood pressure check and
medication refill.  You were diagnosed with high blood pressure and high cholesterol 10
years ago and have been compliant with your medications.  However, you are not very
compliant with dietary sodium restrictions and do not exercise regularly.  You check your
blood pressure 3 times a week in the morning and evening.  It ranges from 140­150
systolic and 80­90 diastolic.

You have no headaches, palpitations, blurry vision, chest pain, shortness of breath, nose
bleeds, dizziness, or leg swelling.

Past medical history:
Hypertension for the past 10 years.  You tried taking lisinopril before and developed a
cough and had sexual problems with atenolol.  (The doctor needs to ask about this; do
not volunteer this information.)
Hypercholesterolemia for the past 10 years.  You had normal numbers a year ago on
your medication.

Past surgical history:  None

Medications:
Hydrochlorothiazide 50 mg once a day
Simvastatin 20 mg at bedtime daily

Allergies:  None

Family history:
Your father has high blood pressure and your mother has diabetes.

Social history:
You are an accountant.  You have smoked 1 pack of cigarettes a day for the past 30
years.  You drink 1 glass of wine occasionally.  You do not use illegal drugs.  You have
had no transfusions or tattoos.

Physical examination:
The following are your temperature (T), blood pressure (BP), heart rate (HR), and
respiratory rate (RR).

Vital signs:  T 98.6 F (37 C), BP 150/90 mm Hg in both arms, HR 90/min, RR 16/min

HEENT:  Pupils are equally round and reactive to light and accommodation; extraocular
movements are intact.  Funduscopic examination is without papilledema, exudates, or AV
nicking.

Neck:  No enlarged lymph nodes, no bruits, carotid pulse 2+ bilaterally

Chest/lungs:  Clear to auscultation bilaterally

Heart:  Regular rhythm without murmurs, gallops, or rubs

Extremities:  No cyanosis, clubbing, or edema.  Peripheral pulses 2+ at radial and
posterior tibialis
The doctor needs to do the above minimum examinations but does not note any
abnormal findings.

This copy of the material is licensed to Mohammed  Mirza.
Copyright © 2015. All Rights Reserved http://www.uworld.com
case10 Scenario  Print

66­year­old male complaining of constipation

Vitals:
PR: 70/min, regular
BP: 120/70 mmHg
RR: 16/min
Temp: 98.0 F (36.7 C)

Make mental checklist for constipation:
Functional constipation
Obstructive lesions (bowel obstruction, carcinoma of colon)
Metabolic disturbances (hypothyroidism, diabetes mellitus, hypercalcemia)
Neurologic dysfunction (stroke, autonomic neuropathy, spinal cord trauma, multiple
sclerosis, and Parkinson's)
Medication induced (iron preparations, opiates, anticholinergics)

This copy of the material is licensed to Mohammed  Mirza.
Copyright © 2015. All Rights Reserved http://www.uworld.com
Case10 Pt Notes  Print

Clinical Skills Evaluation

Case 10 Patient Notes

History:  Describe the history you just obtained from this patient.  Include only
information (pertinent positives and negatives) relevant to this patient’s problem(s).

66­YO man here for constipation for 5 months with worsening over past 2 months
Fatigue, 10­pound weight loss
Change in bowel movement from 1/day to 1 every other day
Occasional black stools

ROS:  No diarrhea, abdominal pain, nausea, vomiting, fever, chills, night sweats, or
urinary problems
PMHx:  Hashimoto’s thyroiditis, severe DJD of right knee
PSHx:  None
Meds:  Levothyroxine, Lortab (started 2 months ago)
Allergies:  None
FHx:  Father died at age 67 of colon cancer
SHx:  No smoking but drinks 1­3 glasses of wine/week

Physical examination:  Describe any positive and negative findings relevant to this
patient’s problem(s).  Be careful to include only those parts of examination performed in
this encounter.

Vital signs:  Temperature, 36.7 C (98.0 F); blood pressure, 120/70 in both arms;
heart rate, 70/min; and respirations, 16/min
HEENT:  No pallor or jaundice
Neck:  No enlarged lymph nodes
Chest/lungs:  Clear to auscultation bilaterally
Heart:  Regular rate and rhythm without M/G/R
Abdomen:  Nontender, nondistended, hypoactive bowel sounds throughout, no
hepatosplenomegaly
Neurologic:  Muscle strength 5/5 throughout, DTR 2+ bilaterally and symmetrical

Data interpretation:  Based on what you have learned from the history and physical
examination, list up to 3 diagnoses that might explain this patient’s complaint(s).  List
your diagnoses from most to least likely.  For some cases, fewer than 3 diagnoses will be
appropriate.  Then enter the positive or negative findings from the history and the
physical examination (if present) that support each diagnosis.  Finally, list initial
diagnostic studies (if any) you would order for each listed diagnosis (eg, restricted
physical examination maneuvers, laboratory tests, imaging, ECG)

Diagnosis #1:  Colon cancer

History finding(s) Physical examination finding(s)
Worsening constipation Hypoactive bowel sounds
Fatigue  
Weight loss  
Black stools  

Diagnosis #2:  Functional constipation from medications

History finding(s) Physical examination finding(s)
New drug (Lortab) started with worsening of
Hypoactive bowel sounds
constipation

Diagnosis #3: Hypothyroidism with suboptimal control

History finding(s) Physical examination finding(s)
Fatigue  
Constipation  

Diagnostic studies
Rectal examination with stool for occult blood
CBC with differential
Serum TSH
Colonoscopy
Serum calcium

This copy of the material is licensed to Mohammed  Mirza.
Copyright © 2015. All Rights Reserved http://www.uworld.com
case10 SP  Print

If the doctor asks you any questions other than these, just say “no” or “things are
normal in my daily life.”

You are a 66­year­old man with constipation.

History of present illness:
You are a 66­year­old man with 5 months of constipation that has worsened over the
past 2 months.  You previously had 1 bowel movement a day that was well formed but
started becoming harder 5 months ago.  You started straining more in the last 2 months
and had increased difficulty passing the stool, a sense of incomplete evacuation, and
decreased frequency to 1 bowel movement every other day.  You have also noticed
occasional black stools without any blood in the past month.  You are tired and have lost
10 pounds in the past 2 months.  You have not changed your diet and eat plenty of fruits
and vegetables.

You have no diarrhea, abdominal pain, nausea, vomiting, fever, chills, night sweats, or
urinary problems.

Past medical history:
Arthritis of the right knee
Hashimoto’s thyroiditis­ you had normal blood work 1 year ago.

Past surgical history: None

Medications:
Levothyroxine 100 µg once a day
Hydrocodone/acetaminophen (Lortab) 5 mg/650 mg 1 pill 3 times/day (started 2 months
ago for knee pain)

Allergies: None

Family history:
Your father died of colon cancer at age 67 and your mother is healthy.

Social history:
You are a supervisor in a pharmaceutical company.  You do not smoke.  You drink 2­3
glasses of wine a week.  You do not use illegal drugs.  You have had no transfusions or
tattoos.

Health maintenance:
You have not had a colonoscopy before but had a normal rectal examination 2 years ago.

Physical examination:
The following are your temperature (T), blood pressure (BP), heart rate (HR), and
respiratory rate (RR).

Vital signs:  T 98.0 F (36.7 C), BP 120/70 mm Hg, HR 70/min, RR 16/min

HEENT:  No pallor or icterus

Neck:  No enlarged lymph nodes

Chest/lungs:  Clear to auscultation bilaterally

Heart:  Regular rhythm without murmurs, gallops, or rubs

Abdomen:  Nontender, nondistended, hypoactive bowel sounds throughout, no
hepatosplenomegaly

Neurologic:  Muscle strength 5/5 throughout, DTR 2+ bilaterally and symmetrical

The doctor needs to do the above minimum examinations.

This copy of the material is licensed to Mohammed  Mirza.
Copyright © 2015. All Rights Reserved http://www.uworld.com
case11 Scenario  Print

50­year­old male complaining of impotence

Vitals:
PR: 80/min, regular
BP: 150/80 mmHg
RR: 16/min
Temp: 98.0 F (36.7 C)

Make mental checklist for Impotence:
Diabetic neuropathy
Atherosclerotic vascular disease
Anxiety and other psychiatric disorders
Medications, like antihypertensives
Chronic alcoholism
Pituitary dysfunction
Spinal cord dysfunction

This copy of the material is licensed to Mohammed  Mirza.
Copyright © 2015. All Rights Reserved http://www.uworld.com
case11 Pt Notes  Print

Clinical Skills Evaluation

Case 11 Patient Notes

History: Describe the history you just obtained from this patient. Include only information
(pertinent positives and negatives) relevant to this patient’s problem(s).

50­YO man here with 4 months of worsening erectile dysfunction
Started new blood pressure medication 4 months ago (atenolol)
Increased stress for past 6 months, mild fatigue
Poorly controlled diabetes with sugars near 200 most of the day

ROS:  No headaches, leg pain, visual disturbances, nausea, vomiting, or abdominal pain
PMHx:  Diabetes, hypertension, anxiety
PSHx:  None
Meds:  Metformin, glyburide, fluoxetine, atenolol (started 4 months ago)
Allergies:  None
FHx:  Parents and siblings are healthy.
SHx:  Occasional smoking but drinks 2­3 beers/day for 25 years

Physical examination:  Describe any positive and negative findings relevant to this patient’s
problem(s).  Be careful to include only those parts of examination performed in this encounter.

Vital signs:  Temperature, 98.0 F (36.7 C); blood pressure, 150/80 mm Hg; heart rate,
80/min; and respiratory rate, 16/min
HEENT:  PERRLA, EOMI
Abdomen:  Nontender, nondistended, normoactive bowel sounds throughout, no
hepatosplenomegaly, no bruits
Extremities:  Pulses 2+ in bilateral lower extremities

Data interpretation: Based on what you have learned from the history and physical
examination, list up to 3 diagnoses that might explain this patient’s complaint(s). List your
diagnoses from most to least likely.  For some cases, fewer than 3 diagnoses will be
appropriate. Then, enter the positive or negative findings from the history and the physical
examination (if present) that support each diagnosis. Finally, list initial diagnostic studies (if
any) you would order for each listed diagnosis (e.g. restricted physical exam maneuvers,
laboratory tests, imaging, ECG, etc.)

Diagnosis #1:  Medication­induced ED

History finding(s) Physical examination finding(s)
Started beta blocker 4 months ago  
No nighttime erection  
Difficulty having daytime erection  

Diagnosis #2:  Testosterone deficiency

History finding(s) Physical examination finding(s)

Fatigue  
Erectile dysfunction  
Diagnosis #3:  Anxiety

History finding(s) Physical examination finding(s)
History of anxiety No focal findings on examination
Increased stress over past 6 months  

Diagnostic studies
Serum glucose and hemoglobin A1C
Serum testosterone and TSH

This copy of the material is licensed to Mohammed  Mirza.
Copyright © 2015. All Rights Reserved http://www.uworld.com
case11 SP  Print

If the doctor asks you any questions other than these, just say “no” or “things are
normal in my daily life.”

You are a 50­year­old man who has erectile dysfunction.

History of present illness:
You are a 50­year­old man who has had 3­4 months of gradually increasing difficulty
getting an erection.  You have sexual desire but have been unable to get or maintain an
erection during this time.  You have been under more stress over the past 6 months due
to financial difficulties and have not had sexual problems before.  You have diabetes and
try to follow a good exercise and diet regimen.  Your sugars range from 150­200
throughout the day.  On further questioning, you say that you also have minimal
nighttime erections.  You also mention mild fatigue during this time.

You have no headaches, leg pain, visual disturbances, nausea, vomiting, or abdominal
pain.

Past medical history:
Diabetes for 10 years
Hypertension
Generalized anxiety disorder

Past surgical history:  None

Medications:
Atenolol 50 mg a day (started 4 months ago)
Metformin 500 mg a day
Glyburide 10 mg a day
Fluoxetine 20 mg a day

Allergies:  None

Family history:  Parents and siblings are healthy.

Social history:  You are a truck driver and live with your wife.  You smoke 1­2 cigarettes
a week.  You have consumed 2­3 beers/day for 25 years.  You do not do illegal drugs.
 You have had no transfusions or tattoos.

Physical examination:
The following are your temperature (T), blood pressure (BP), heart rate (HR), and
respiratory rate (RR).

Vital signs:  T 98.0 F (36.7 C), BP 150/80 mm Hg, HR 80/min, RR 16/min

HEENT:  PERRLA, EOMI

Abdomen:  Nontender, nondistended, normoactive bowel sounds throughout, no
hepatosplenomegaly, no bruits

Extremities:  Pulses 2+ in bilateral lower extremities

The doctor needs to do the above minimum examinations.

This copy of the material is licensed to Mohammed  Mirza.
Copyright © 2015. All Rights Reserved http://www.uworld.com
case12 Scenario  Print

Mother of 1 Yr. O/Baby With Fever
Make a mental note of differential diagnosis of fever

Respiratory tract infections
Ear infections
Exanthematous diseases
Meningitis
Urinary tract infections
Gastroenteritis

This copy of the material is licensed to Mohammed  Mirza.
Copyright © 2015. All Rights Reserved http://www.uworld.com
case12 SP  Print

If the doctor asks you any questions other than these, just say “no,” or “things are
normal in my daily life.” This is a telephone encounter with the doctor asking you
questions on the phone about the child.

You are a 28­year­old woman who calls for evaluation of fever in your 1­year­old child.

History of present illness:
You are a 28­year­old woman whose 1­year­old child has had fever for 2 days.  The child
was previously well but has developed fever to 102 F and dry cough.  The child has not
been feeding well for the past 2 days but does not appear drowsy or lethargic. 
Acetaminophen decreases the fever to 99 F.  The child passes yellow urine but cries
while urinating.  The child also has had 2 episodes of vomiting ingested food without
diarrhea. The child also has been picking at the right ear more often during this time.

You also noticed 1 episode in which the child shook with rhythmic jerking on both sides
and lost urine.  After the episode, the child became silent, was irritable for a few
minutes, and then slept.  You have bathed the child using tepid sponges with cold water
with some relief.  Your child's 3­year­old sibling has no complaints.  The child stays at
home with you.  All of the child's immunizations are up to date.  The most recent well
child visit at 9 months had placed the child at 75%ile for height and 60%ile for weight.

There is no recent travel, rashes, sick contacts, or other complaints.

Past medical history/birth history:
The child was full term without complications at delivery.  He/she was breast­fed until 2
months and then started on formula.  The child is able to stand, hold objects in the
hands, and say “mama” and “dada.” The child has no other medical conditions and no
hospitalizations.

Past surgical history:  None

Medications:  Tylenol as needed

Allergies:  None

Family history:  Parents and siblings are healthy.

This copy of the material is licensed to Mohammed  Mirza.
Copyright © 2015. All Rights Reserved http://www.uworld.com
Case12 Pt Notes  Print

Clinical Skills Evaluation

Case 12 Patient Notes

History:  Describe the history you just obtained from this patient.  Include only
information (pertinent positives and negatives) relevant to this patient's problem(s).

Telephone encounter with mother of 1­YO child
1­YO child with 2 days of fever to 102 F, cough, and picking at the right ear
Two episdoes of vomiting but no diarrhoea
Also had 1 episode of rhythmic total body shaking with passing urine
Crying when passing yellow urine
No lethargy
No previous infections or hospitalizations
No sick contacts; sibling is without symptoms
Birth history unremarkable with normal developmental milestones
Immunizations up to date

ROS:  No recent travel or rashes
PMHx:  None
PSHx:  None
Meds:  Acetaminophen
Allergies:  None
FHx:  Parents and siblings are healthy.

Physical examination:  Describe any positive and negative findings relevant to this
patient's problem(s).  Be careful to include only those parts of examination performed in
this encounter.

Not done as this is a telephone encounter

Data interpretation:  Based on what you have learned from the history and physical
examination, list up to 3 diagnoses that might explain this patient's complaint(s).  List
your diagnoses from most to least likely.  For some cases, fewer than 3 diagnoses will be
appropriate.  Then enter the positive or negative findings from the history and the
physical examination (if present) that support each diagnosis.  Finally, list initial
diagnostic studies (if any) you would order for each listed diagnosis (eg, restricted
physical examination maneuvers, laboratory tests, imaging, ECG)

Diagnosis #1:  Acute otitis media

History finding(s) Physical examination finding(s)
Fever  
Picking at right ear  

Diagnosis #2:  Urinary tract infection

History finding(s) Physical examination finding(s)
Fever  
Pain with urination  

Diagnosis #3:  Febrile seizure

History finding(s) Physical examination finding(s)
Fever  
Rhythmic tremor with fever that subsides  
Passing urine with shaking movements  

Diagnostic studies
Office visit for ear examination
Urinalysis
CBC with differential

This copy of the material is licensed to Mohammed  Mirza.
Copyright © 2015. All Rights Reserved http://www.uworld.com
case13 Scenario  Print

45­year­old female complaining of acute, right upper quadrant abdominal pain

Vital signs:
PR: 100/min
Temp: 101.0 F (38.3 C)
RR: 20/min
BP: 130/80 mmHg

Make a mental checklist of DD for RUQ abdominal pain:
Acute cholecystitis
Biliary colic
Acute hepatitis
Perforation of peptic ulcer
Acute pancreatitis (biliary pain)
Right lower lobe pneumonia  
Myocardial infarction
Congestive hepatomegaly
Hepatic abscess
Retrocecal appendicitis

This copy of the material is licensed to Mohammed  Mirza.
Copyright © 2015. All Rights Reserved http://www.uworld.com
case13 SP  Print

If the doctor asks you any questions other than these, just say “no” or “things are
normal in my daily life.”

You are a 45­year­old woman complaining of abdominal pain.

History of present illness:
You are a 45­year­old woman who has had acute abdominal pain for 2 hours.  The pain
started suddenly in the right upper part of the abdomen and has been gradually
worsening.  The pain is 8­9/10 in severity and feels like a stabbing sensation that
radiates to the back of the right shoulder.  It started 30 minutes after you ate a meal
today.  You also have occasional nausea and vomiting without blood or bile.  The pain is
worsened with deep breathing and is not relieved with antacids.

You have had previous episodes of similar pain approximately 3­4 times a month starting
5 months ago.  The episodes usually begin 2­3 hours after meals and sometimes improve
with antacids.  You eat a lot of fast food for lunch and dinner because you are very busy
with work and have no time to cook.  You also feel warm but did not check your
temperature.

You have no jaundice, cough, shortness of breath, itching, chest pain, or diarrhea.

Past medical history:  None

Past surgical history:  C­section 20 years ago

Medications:  Over­the­counter antacids as needed

Allergies:  None

Family history:  Parents and siblings are healthy.

Social history:  You are an accountant and live with your husband.  You have smoked 1
pack of cigarettes a day for 25 years.  You have consumed 2­3 beers a day for 15 years.
 You do not take illegal drugs.  You have had no transfusions or tattoos.

Physical examination:
The following are your temperature (T), blood pressure (BP), heart rate (HR), and
respiratory rate (RR).

Vital signs:  T 101.0 F (38.3 C), BP 130/80 mm Hg, HR 100/min, RR 20/min

HEENT:  PERRLA, EOMI, no jaundice

Abdomen:  RUQ discomfort with deep palpation and slightly worse with deep breath;
nondistended, normoactive bowel sounds throughout; no hepatosplenomegaly; no bruits

The doctor needs to do the above minimum examinations.

This copy of the material is licensed to Mohammed  Mirza.
Copyright © 2015. All Rights Reserved http://www.uworld.com
case13 Pt Notes  Print

Clinical Skills Evaluation

Case 13 Patient Notes

History:  Describe the history you just obtained from this patient.  Include only
information (pertinent positives and negatives) relevant to this patient’s problem(s).

45­YO female with 2 hours of worsening RUQ abdominal pain radiating to the right
scapula
Has had 5 months of intermittent similar episodes 3­4 times a month that resolved
with antacids
Stabbing pain starting 30 minutes after food with nausea and non­bilious and non­
bloody vomitus
Pain is worsened with deep breathing and not improved with antacids

ROS:  No jaundice, cough, shortness of breath, itching, chest pain, or diarrhea
PMHx:  None
PSHx:  C­section 20 years ago
Meds:  OTC antacids PRN
Allergies:  None
FHx:  Parents and siblings are healthy.
SHx:  1 PPD smoker for 25 years and drinks 2­3 beers/day for 15 years

Physical examination:  Describe any positive and negative findings relevant to this
patient’s problem(s).  Be careful to include only those parts of examination performed in
this encounter.

Vital signs:  Temperature, 101.0 F (38.3 C); blood pressure,  130/80 mm Hg; heart
rate, 100/min; and respiratory rate, 20/min
HEENT:  PERRLA, EOMI, no jaundice
Abdomen:  RUQ discomfort with deep palpation; nondistended, normoactive bowel
sounds throughout; no hepatosplenomegaly; no bruits

Data interpretation:  Based on what you have learned from the history and physical
examination, list up to 3 diagnoses that might explain this patient’s complaint(s).  List
your diagnoses from most to least likely.  For some cases, fewer than 3 diagnoses will be
appropriate.  Then enter the positive or negative findings from the history and the
physical examination (if present) that support each diagnosis.  Finally, list initial
diagnostic studies (if any) you would order for each listed diagnosis (eg, restricted
physical examination maneuvers, laboratory tests, imaging, ECG)

Diagnosis #1:  Acute cholecystitis

History finding(s) Physical examination finding(s)
RUQ abdominal pain Fever
5 months of similar episodes RUQ tenderness
Pain radiating to right shoulder Positive Murphy’s sign
Pain worsened with deep breathing  
Diagnosis #2:  Acute pancreatitis

History finding(s) Physical examination finding(s)
RUQ pain RUQ tenderness
Nausea and vomiting Fever
Alcohol use  

Diagnosis #3:  Peptic ulcer

History finding(s) Physical examination finding(s)
Nausea and vomiting Fever
RUQ pain RUQ tenderness
Alcohol/Tobacco use  

Diagnostic studies
Ultrasound RUQ abdomen
Serum amylase and lipase
Liver function tests
CBC with differential

This copy of the material is licensed to Mohammed  Mirza.
Copyright © 2015. All Rights Reserved http://www.uworld.com
case14 Scenario  Print

24 Yr. O/F Came for prenatal visit for the first time
Vitals

Pulse­­78/min
B.P­­120/75 mm of Hg
Temp­98.8F
R.rate­­20/min

This copy of the material is licensed to Mohammed  Mirza.
Copyright © 2015. All Rights Reserved http://www.uworld.com
case14 SP  Print

If the doctor asks you any questions other than these, just say “no” or “things are
normal in my daily life.”

You are a 25­year­old woman here for your first prenatal visit.

History of present illness:
You are a 25­year­old woman G1P0 who comes to the physician for her first prenatal
visit.  Your last menstrual period was 12 weeks ago and you confirmed your pregnancy 6
weeks ago with a home pregnancy test.  You have not yet felt any fetal movements and
this is your first pregnancy.  You have no history of miscarriage or abortion.  You had
menarche at age 13 and have had regular periods for 4­5 days each month.

You have no morning sickness, vomiting, abdominal pain, vaginal bleeding, fever, rash,
breathing problems, sleep disturbances, or swelling in the feet.

Past medical history:  None

Past surgical history:  None

Medications:  None

Allergies:  None

Family history:  Parents and siblings are healthy.

Social history:  You are currently not working and live with your husband.  You have
smoked 1 pack of cigarettes a day for 5 years.  You have consumed 1­3 beers a week for
3 years.  You do not use illegal drugs.  You have had no transfusions or tattoos.

Health maintenance:  You are up to date on all immunizations

Physical examination:
The following are your temperature (T), blood pressure (BP), heart rate (HR), and
respiratory rate (RR).

Vital signs:  T 98.8 F (37.1 C), BP 120/75 mm Hg, HR 78/min, RR 20/min

HEENT:  PERRLA, EOMI, no jaundice, oropharynx clear

Lungs:  Clear to auscultation bilaterally

Heart:  Regular rate and rhythm without murmurs, gallops, or rubs

Abdomen:  Nontender, nondistended, normoactive bowel sounds throughout, no
hepatosplenomegaly, and no bruits

Extremities:  No cyanosis or edema

The doctor needs to do the above minimum examinations.

This copy of the material is licensed to Mohammed  Mirza.
Copyright © 2015. All Rights Reserved http://www.uworld.com
case14 Pt Notes  Print

Clinical Skills Evaluation

Case 14 Patient Notes

History:  Describe the history you just obtained from this patient.  Include only
information (pertinent positives and negatives) relevant to this patient’s problem(s).

24­YO woman G1P0A0 and LMP 12 weeks ago who presents for her first prenatal
visit
Has no fetal movements yet

ROS:  No morning sickness, vomiting, abdominal pain, vaginal bleeding, fever, rash,
breathing problems, sleep disturbances, or swelling in the feet
PMHx:  None
PSHx:  None
Meds:  None
Allergies:  None
FHx:  Parents and siblings are healthy.
SHx:  1 PPD smoker for 5 years and drinks 1­3 beers/week for 3 years

Physical examination:  Describe any positive and negative findings relevant to this
patient’s problem(s).  Be careful to include only those parts of examination performed in
this encounter.

Vital signs:  Temperature, 98.8 F (37.1 C); blood pressure, 120/75 mm Hg; heart
rate, 78/min; and respiratory rate, 20/min
HEENT:  PERRLA, EOMI, no jaundice, oropharynx clear
Lungs:  Clear to auscultation bilaterally
Heart:  Regular rate and rhythm without murmurs, gallops, or rubs
Abdomen:  Nontender, nondistended, normoactive bowel sounds throughout; no
hepatosplenomegaly; and no bruits
Extremities:  No cyanosis or edema

Data interpretation:  Based on what you have learned from the history and physical
examination, list up to 3 diagnoses that might explain this patient’s complaint(s).  List
your diagnoses from most to least likely.  For some cases, fewer than 3 diagnoses will be
appropriate.  Then enter the positive or negative findings from the history and the
physical examination (if present) that support each diagnosis.  Finally, list initial
diagnostic studies (if any) you would order for each listed diagnosis (eg, restricted
physical examination maneuvers, laboratory tests, imaging, ECG)

Diagnosis #1:  Normal pregnancy

History finding(s) Physical examination finding(s)
LMP 12 weeks ago Nondistended abdomen
Home pregnancy test positive 6 weeks ago No edema

Diagnosis #2:
History finding(s) Physical examination finding(s)

Diagnosis #3:

History finding(s) Physical examination finding(s)

Diagnostic studies
Urine culture
CBC with differential
Rhesus type and red cell antibody screen
Rubella and varicella titters
Serum RPR, HIV, Hepatitis B antigen

This copy of the material is licensed to Mohammed  Mirza.
Copyright © 2015. All Rights Reserved http://www.uworld.com
case15 Scenario  Print

60­year­old male complaining of acute shortness of breath

Vitals:
PR: 90/min
BP: 110/70 mmHg
Temp: 98 F (36.7 C)
RR: 26/min

Make a mental checklist for acute shortness of breath:
Pulmonary embolism
Congestive heart failure
Chronic obstructive pulmonary disease exacerbation
Pneumonia
Spontaneous pneumothorax
Bronchial asthma
Anxiety and panic attacks

This copy of the material is licensed to Mohammed  Mirza.
Copyright © 2015. All Rights Reserved http://www.uworld.com
case15 SP  Print

If the doctor asks you any questions other than these, just say “no” or “things are
normal in my daily life.”

You are a 60­year­old man complaining of shortness of breath.

History of present illness:
You are a 60­year­old man who comes to the physician with shortness of breath for 2
days.  You were doing well until 1 year ago when you had onset of shortness of breath
with strenuous work that slowly worsened during the year.  However, during the last 2
days you have had an acute worsening of the symptoms with trouble breathing at night.
 You need 2 pillows when you sleep.  Your breathing is worse when lying down and
improves with sitting up.  You also have awakened with shortness of breath during the
night and take 20 minutes to get back to sleep.

You have dyspnea with mild exertion ­ walking 1 block ­ that improves with rest.  You
also have occasional dry cough.  You have no swelling in legs.  You had spinal fusion
surgery 2 weeks ago for spinal stenosis that required 1 week in the hospital
postoperatively.

You have no chest pain, wheezing, fatigue, palpitations, syncope, fever, or chills.

Past medical history:  Hypertension ­ diagnosed 20 years ago

Past surgical history:  Spinal stenosis in the low back requiring spinal fusion surgery as
noted above

Medications:  Hydrochlorothiazide 25 mg once daily

Allergies:  None

Family history:  Father died at age 55 of heart attack; mother has hypertension; no
siblings

Social history:  You are a computer software analyst and live with your wife.  You have
smoked 1 pack of cigarettes a day for 40 years.  You have consumed 1­2 glasses of
wine/day for 35 years.  You have never used illegal drugs.  You have had no transfusions
or tattoos.

Physical examination:
The following are your temperature (T), blood pressure (BP), heart rate (HR), and
respiratory rate (RR).

Vital signs:  T 98.0 F (36.7 C), BP 110/70 mm Hg, HR 90/min, RR 26/min

HEENT:  PERRLA, EOMI, no jaundice

Neck:  Supple without lymphadenopathy, no JVD, no thyromegaly

Lungs:  Clear to auscultation

Heart:  Regular rate and rhythm without murmurs, gallops, rubs

Extremities:  No edema, pulses 2+ bilaterally, no cyanosis or clubbing, no lower
extremity tenderness to palpation

The doctor needs to do the above minimum examinations.
This copy of the material is licensed to Mohammed  Mirza.
Copyright © 2015. All Rights Reserved http://www.uworld.com
case15 Pt Notes  Print

Clinical Skills Evaluation

Case 15 Patient Notes

History:  Describe the history you just obtained from this patient.  Include only
information (pertinent positives and negatives) relevant to this patient’s problem(s).

60­YO man with 2 days of worsening shortness of breath
Has had 1 year of gradually worsening shortness of breath that acutely worsened 2
days ago
New 2­pillow orthopnea, PND
Spinal fusion surgery 2 weeks ago
Occasional dry cough

ROS:  No chest pain, wheezing, fatigue, palpitations, leg swelling, syncope, fever, or
chills.
PMHx:  HTN
PSHx:  Spinal fusion surgery
Meds:  Hydrochlorothiazide 25 mg a day
Allergies:  None
FHx:  Father died of MI at age 55; mother has HTN
SHx:  1 PPD smoker for 40 years and drinks 1­2 glasses of wine/day for 35 years

Physical examination:  Describe any positive and negative findings relevant to this
patient’s problem(s).  Be careful to include only those parts of examination performed in
this encounter.

Vital signs:  Temperature, 98.0 F (36.7 C); blood pressure, 110/70 mm Hg; heart
rate, 90/min;  and respiratory rate, 26/min
HEENT:  PERRLA, EOMI, no jaundice
Neck:  Supple without lymphadenopathy, no JVD, no thyromegaly
Lungs:  Clear to auscultation
Heart:  Regular rate and rhythm without murmurs, gallops, rubs
Extremities:  No edema, pulses 2+ bilaterally, no cyanosis or clubbing, no lower­
extremity tenderness to palpation

Data interpretation:  Based on what you have learned from the history and physical
examination, list up to 3 diagnoses that might explain this patient’s complaint(s).  List
your diagnoses from most to least likely.  For some cases, fewer than 3 diagnoses will be
appropriate.  Then enter the positive or negative findings from the history and the
physical examination (if present) that support each diagnosis.  Finally, list initial
diagnostic studies (if any) you would order for each listed diagnosis (eg, restricted
physical examination maneuvers, laboratory tests, imaging, ECG)

Diagnosis #1:  Pulmonary embolism

History finding(s) Physical examination finding(s)
Acute worsening of dyspnea  
Recent spinal fusion surgery  
Diagnosis #2:  Congestive heart failure

History finding(s) Physical examination finding(s)
1 year of gradually worsening dyspnea  
Dyspnea worse with exertion  
Orthopnea and PND  

Diagnosis #3:  COPD

History finding(s) Physical examination finding(s)
History of smoking  
1 Year of gradually worsening dyspnea  
Cough  

Diagnostic studies
Chest x­ray
CT angiogram of chest
CBC with differential
Basic metabolic panel
EKG, Echocardiogram
 

This copy of the material is licensed to Mohammed  Mirza.
Copyright © 2015. All Rights Reserved http://www.uworld.com
case16 Scenario  Print

40­year­old female with increased urination

Vitals:
PR: 86/min,regular
BP: 110/70 mmHg
RR: 16/min
Temp: 98.0 F (36.7 C)

Make a mental checklist of DD for polyuria:
Diabetes mellitus
Central diabetes insipidus
Nephrogenic diabetes insipidus
Psychogenic Polydypsia
Cystitis (Urinary Tract Infection)
Diuretics and other drug intake
Hypercalcemia

This copy of the material is licensed to Mohammed  Mirza.
Copyright © 2015. All Rights Reserved http://www.uworld.com
case16 SP  Print

If the doctor asks you any questions other than these, just say “no” or “things are
normal in my daily life.”

You are a 40­year­old woman complaining of increased urinary frequency.

History of present illness:
You are a 40­year­old woman with 2 months of increased urinary frequency.  You have
been urinating 8­10 times during the day and 2­3 times at night with increased urinary
volume.  You have also become increasingly thirsty for the past month and have felt
tired for the past 2 months.  You drink a lot of water and are also eating more but have
lost 10 pounds during the past 2­3 months.

You have no burning with urination, urinary urgency, fever, or chills.

Past medical history:
Bipolar disorder diagnosed 20 years ago
G2P2
Minor head trauma after falling off bicycle 3 months ago; seen in emergency department
and discharged without intervention

Past surgical history:  None

Medications:  Lithium 600 mg two times daily

Allergies:  None

Family history:  Father and mother both have diabetes, no siblings

Social history:  You are a homemaker and live with your husband.  You do not smoke or
drink alcohol.  You have never used illegal drugs.  You have had no transfusions or
tattoos.

Physical examination:
The following are your temperature (T), blood pressure (BP), heart rate (HR), and
respiratory rate (RR).

Vital signs:  T 98.0 F (36.7 C), BP 110/70 mm Hg, HR 86/min, RR 16/min

HEENT:  PERRLA, EOMI, visual fields intact

Heart:  Regular rate and rhythm without murmurs, gallops, rubs

Abdomen:  Nontender without suprapubic tenderness, no CVA tenderness

Neurologic:  Muscle strength 5/5 throughout, sensation grossly intact in bilateral lower
extremities, DTR 2+ bilateral lower extremities

The doctor needs to do the above minimum examinations.

This copy of the material is licensed to Mohammed  Mirza.
Copyright © 2015. All Rights Reserved http://www.uworld.com
case16 Pt Notes  Print

Clinical Skills Evaluation

Case 16 Patient Notes

History:  Describe the history you just obtained from this patient.  Include only
information (pertinent positives and negatives) relevant to this patient’s problem(s).

40­YO woman with 2 months of polyuria, polydipsia, nocturia, and polyphagia
Has had 2­3 months of 10­pound weight loss with fatigue
No dysuria or urinary urgency

ROS:  No fever or chills
PMHx:  Bipolar disorder diagnosed 20 years ago, minor head trauma 3 months ago seen
in emergency department and discharged without any intervention
PSHx:  None
Meds:  Lithium 600 mg 2 times daily
Allergies:  None
FHx:  Father and mother have diabetes.
SHx:  No history of tobacco or alcohol use

Physical examination:  Describe any positive and negative findings relevant to this
patient’s problem(s).  Be careful to include only those parts of examination performed in
this encounter.

Vital signs:  Temperature,  98.0 F (36.7 C); blood pressure, 110/70 mm Hg; heart
rate,  86/min; and respiratory rate,16/min
HEENT:  PERRLA, EOMI, visual fields intact
Heart:  Regular rate and rhythm without murmurs, gallops, rubs
Abdomen:  Nontender without suprapubic tenderness, no CVA tenderness
Neurologic:  Muscle strength 5/5 throughout, sensation grossly intact in bilateral
lower extremities, DTR 2+ bilateral lower extremities

Data interpretation:  Based on what you have learned from the history and physical
examination, list up to 3 diagnoses that might explain this patient’s complaint(s).  List
your diagnoses from most to least likely.  For some cases, fewer than 3 diagnoses will be
appropriate.  Then enter the positive or negative findings from the history and the
physical examination (if present) that support each diagnosis.  Finally, list initial
diagnostic studies (if any) you would order for each listed diagnosis (eg, restricted
physical examination maneuvers, laboratory tests, imaging, ECG)

Diagnosis #1:  Diabetes mellitus

History finding(s) Physical examination finding(s)
Polyuria  
Polydipsia and polyphagia  
Family history of diabetes in father and
 
mother
Weight loss  
Diagnosis #2:  Diabetes insipidus

History finding(s) Physical examination finding(s)
History of bipolar disorder  
Lithium use  
Polyuria  

Diagnosis #3:  Psychogenic polydipsia

History finding(s) Physical examination finding(s)
History of bipolar disorder  
Polyuria  
Polydipsia  

Diagnostic studies
Fasting blood sugar
Hemoglobin A1C
Urinalysis
Serum electrolytes, lithium level
Urine and serum osmolality

This copy of the material is licensed to Mohammed  Mirza.
Copyright © 2015. All Rights Reserved http://www.uworld.com
case17 Scenario  Print

35 Yr. O/F Came for evaluation of jaundice
Vitals

Pulse­­98/min
B.P­­120/75 mm of Hg
Temp­101.3
R.rate­­22/min

Make a mental checklist of Differential Diagnosis for jaundice

Infectious hepatitis
Hemolytic jaundice
Alcoholic hepatitis
Drug induced hepatitis
Primary biliary cirrhosis
Wilson's disease
Hemochromatosis
Malignancy

This copy of the material is licensed to Mohammed  Mirza.
Copyright © 2015. All Rights Reserved http://www.uworld.com
case17 SP  Print

If the doctor asks you any questions other than these, just say “no” or “things are
normal in my daily life.”

You are a 35­year­old woman complaining of jaundice.

History of present illness:
You are a 35­year­old woman who has had jaundice for 2 days.  You were well until 5
days ago when you developed fever.  You noticed dark urine and pale stools 3 days ago,
and then yellow eyes 2 days ago.  You also have had mild abdominal discomfort, nausea,
and 2­3 episodes of non­bilious and non­bloody vomiting.  You returned 3 weeks ago
from a trip to India, where you ate local food and drank bottled water.  You have not had
vaccines for hepatitis.

You have no sore throat, headache, stiff neck, dysuria, weight loss, itching, diarrhea, or
constipation.

Ask the doctor:  “Do you think that this is hepatitis?”

Past medical history:
Car accident 2 years ago requiring hospitalization and blood transfusion
G2P2

Past surgical history:  None

Medications:  None

Allergies:  None

Family history:  Father and mother are healthy; 3 siblings are also healthy.

Social history:  You are a homemaker and live with your husband.  You do not smoke. 
You have consumed 2­3 beers a day for the past 15 years.  You have never used illegal
drugs.  You have no tattoos.

Physical examination:
The following are your temperature (T), blood pressure (BP), heart rate (HR), and
respiratory rate (RR).

Vital signs:  T 101.3 F (38.5 C), BP 120/75 mm Hg, HR 98/min, RR 22/min

HEENT:  PERRLA, EOMI, icterus present, oropharynx clear

Neck:  No lymphadenopathy

Abdomen:  Nontender, nondistended, no hepatosplenomegaly, normoactive bowel
sounds

The doctor needs to do the above minimum examinations.

This copy of the material is licensed to Mohammed  Mirza.
Copyright © 2015. All Rights Reserved http://www.uworld.com
case17 Pt Notes  Print

Clinical Skills Evaluation

Case 17 Patient Notes

History:  Describe the history you just obtained from this patient.  Include only
information (pertinent positives and negatives) relevant to this patient's problem(s).

35­YO woman with 2 days of jaundice associated with fever
Returned 3 weeks ago from trip to India
Dark urine and pale stools
Mild diffuse abdominal discomfort, nausea, vomiting

ROS:  No sore throat, headache, stiff neck, dysuria, weight loss, itching, diarrhea, or
constipation
PMHx:  Car accident 2 years ago requiring hospitalization and blood transfusion
PSHx:  None
Meds:  None
Allergies:  None
FHx:  Father, mother, and siblings are healthy.
SHx:  No history of tobacco use.  She drinks 2­3 beers/day for the past 15 years.

Physical examination:  Describe any positive and negative findings relevant to this
patient's problem(s).  Be careful to include only those parts of examination performed in
this encounter.

Vital signs:  Temperature, 101.3 F (38.5 C); blood pressure, 120/75 mm Hg; heart
rate, 98/min; respiratory rate, 22/min
HEENT:  PERRLA, EOMI, icterus present, oropharynx clear
Neck:  No lymphadenopathy
Abdomen:  Nontender, nondistended, no hepatosplenomegaly, normoactive bowel
sounds

Data interpretation:  Based on what you have learned from the history and physical
examination, list up to 3 diagnoses that might explain this patient's complaint(s).  List
your diagnoses from most to least likely.  For some cases, fewer than 3 diagnoses will be
appropriate.  Then enter the positive or negative findings from the history and the
physical examination (if present) that support each diagnosis.  Finally, list initial
diagnostic studies (if any) you would order for each listed diagnosis (eg, restricted
physical examination maneuvers, laboratory tests, imaging, ECG)

Diagnosis #1:  Infectious hepatitis

History finding(s) Physical examination finding(s)
Travel to India Fever
Jaundice Jaundice
Nausea, vomiting, abdominal discomfort  
Dark urine with pale stools  
Diagnosis #2:  Alcoholic hepatitis

History finding(s) Physical examination finding(s)
Alcohol use Fever
Nausea, vomiting, abdominal discomfort Jaundice

Diagnostic studies
Hepatitis serologies:  A, B, and C
CBC with differential
Liver function tests
Urinalysis

This copy of the material is licensed to Mohammed  Mirza.
Copyright © 2015. All Rights Reserved http://www.uworld.com
case18 Scenario  Print

35 Yr. O/F Complaining of Chest Pain
Vitals

Pulse­­98/min
B.P­­120/75 mm of Hg
Temp­98.3
R.rate­­12/min

Make a mental checklist of Differential Diagnosis.
If a young adult female comes with chest pain  think of non cardiac causes first because
that is the one that you are going to get in the exam.

Pneumonia
Gastro esophageal reflux disease (GERD)
Panic disorder
Hyperthyroidism
Pheochromocytoma
Hyperventilation syndrome
Angina  
Costochondritis

This copy of the material is licensed to Mohammed  Mirza.
Copyright © 2015. All Rights Reserved http://www.uworld.com
case18 SP  Print

If the doctor asks you any questions other than these, just say “no” or “things are
normal in my daily life.”

You are a 35­year­old woman complaining of chest pain.

History of present illness:
You are a 35­year­old woman who has had chest pain for 1 day.  You were well until 1
year ago, when you started having episodes of chest pain described as squeezing
tightness all over the chest without radiation.  These episodes lasted around 20 minutes.
 Each episode is a 4­5/10 in severity without aggravating or relieving factors.  You also
note hyperventilation, increased sweating, headache, palpitations, and a sense of
impending doom.  You had 4­5 episodes for the first 6 months but they have increased to
once a week since then.  Today, you have had 2 episodes.

You were hospitalized 6 months ago with chest pain and were tested, but everything
came back normal.  No cause for the chest pain was determined.  The symptoms seem
to occur when you go out in public.  You have tried antacids without relief.

You have no nausea, vomiting, dysuria, diarrhea, constipation, flushing, sleep
disturbances, or abdominal pain.

Ask the doctor:  “Do you think that this is a heart attack?”

Past medical history:
Car accident 2 years ago requiring hospitalization and blood transfusion
G0P0

Past surgical history:  None

Medications:  None

Allergies:  None

Family history:  Father is healthy.  Mother has hypochondriasis.  Sister has
hyperthyroidism.

Social history:  You live alone and are a secretary for a large law firm.  You have
smoked 1 pack a day for the past 15 years.  You do not drink alcohol.  You have never
used illegal drugs.  You have no tattoos.

Physical examination:
The following are your temperature (T), blood pressure (BP), heart rate (HR), and
respiratory rate (RR).

Vital signs:  T 98.3 F (36.8 C), BP 120/75 mm Hg, HR 98/min, RR 12/min

Neck:  No lymphadenopathy or thyromegaly

Lungs:  Clear to auscultation bilaterally

Heart:  Regular rate and rhythm without murmurs, gallops, or rubs

The doctor needs to do the above minimum examinations.

This copy of the material is licensed to Mohammed  Mirza.
Copyright © 2015. All Rights Reserved http://www.uworld.com
case18 Pt Notes  Print

Clinical Skills Evaluation

Case 18 Patient Note

History:  Describe the history you just obtained from this patient.  Include only
information (pertinent positives and negatives) relevant to this patient’s problem(s).

35­YO woman with 1 year of worsening chest pain episodes
Episode described as diffuse chest tightness associated with headache, palpitations,
diaphoresis, hyperventilation, and sense of impending doom
Has been hospitalized previously with normal testing
Episodes occur more often when out in public and are not relieved with antacids

ROS:  No nausea, vomiting, dysuria, diarrhea, constipation, flushing, sleep disturbances,
or abdominal pain.
PMHx:  None
PSHx:  None
Meds:  None
Allergies:  None
FHx:  Father is healthy, mother has hypochondriasis, and sister has hyperthyroidism.
SHx:  1 PPD smoker for 15 years. She does not drink alcohol or use illegal drugs.

Physical examination:  Describe any positive and negative findings relevant to this
patient’s problem(s).  Be careful to include only those parts of examination performed in
this encounter.

Vital signs:  Temperature, 98.3 F (36.8 C); blood pressure, 120/75 mm Hg; heart
rate, 98/min; and respiratory rate,12/min
Neck:  No lymphadenopathy or thyromegaly
Lungs:  Clear to auscultation bilaterally
Heart:  Regular rate and rhythm without murmurs, gallops, or rubs

Data interpretation:  Based on what you have learned from the history and physical
examination, list up to 3 diagnoses that might explain this patient’s complaint(s).  List
your diagnoses from most to least likely.  For some cases, fewer than 3 diagnoses will be
appropriate.  Then enter the positive or negative findings from the history and the
physical examination (if present) that support each diagnosis.  Finally, list initial
diagnostic studies (if any) you would order for each listed diagnosis (eg, restricted
physical examination maneuvers, laboratory tests, imaging, ECG).

Diagnosis #1:  Panic attack disorder

History finding(s) Physical examination finding(s)
Episodes of chest pain Normal vital signs
Worse when out in public Normal cardiac examination
Normal investigation in the past  

Diagnosis #2:  Hyperthyroidism
History finding(s) Physical examination finding(s)
Family history  
Palpitations  

Diagnosis #3:  Cardiac arrhythmia (eg, SVT)

History finding(s) Physical examination finding(s)
Episodic palpitations  
Chest pain and diaphoresis  

Diagnostic studies
EKG
Thyroid function tests
Event monitor

This copy of the material is licensed to Mohammed  Mirza.
Copyright © 2015. All Rights Reserved http://www.uworld.com
case19 Scenario  Print

45 Yr. O/M complaining of Rt lower abdominal pain
Vitals

Pulse­­100/min
Temp­­98.7
R.Rate­­20/min
B.P­­130/80 mm of Hg.

Make a mental check list for Rt Lower Quadrant abdominal pain in a male pt

Appendicitis  
Meckel's diverticulitis
Perforated viscus 
Intestinal obstruction
Yersinia enterocolitica
Pancreatitis
Urolithiasis
Even acute cholecystitis

This copy of the material is licensed to Mohammed  Mirza.
Copyright © 2015. All Rights Reserved http://www.uworld.com
case19 SP  Print

If the doctor asks you any questions other than these, just say “no” or “things are
normal in my daily life.”

You are a 45­year­old man complaining of abdominal pain.

History of present illness:
You are a 45­year­old man who has had abdominal pain for 2 hours.  The sharp pain
started suddenly behind the belly button and radiated to the right lower part of the
abdomen.  It is an 8­9/10 severity and has been increasing progressively.  The
symptoms started an hour after you ate a large meal and are worse with movement.
 You also have had nausea and 2 episodes of non­bilious and non­bloody vomiting today.
 Nothing improves the pain.  Your last bowel movement was 20 hours ago and you are
passing gas.

You have no fever, chills, dysuria, diarrhea, constipation, or back pain.

Past medical history:  Peptic ulcer disease that was treated 10 years ago with
omeprazole

Past surgical history:  None

Medications:  None

Allergies:  None

Family history:  Father, mother, and 2 siblings are healthy.

Social history:
You live alone and work as a bus driver for a local company.  You have smoked 1 pack a
day for the past 20 years.  You have consumed 3 beers a day for the past 15 years.  You
have never used illegal drugs.  You have no tattoos.

Physical examination:
The following are your temperature (T), blood pressure (BP), heart rate (HR), and
respiratory rate (RR).

Vital signs:  T 98.7 F (37.1 C), BP 130/80, HR 100/min, RR 20/min

Abdomen:  Right lower quadrant tenderness to superficial and deep palpation, rebound
tenderness noted, normoactive bowel sounds throughout, no hepatosplenomegaly, no
CVA tenderness, negative psoas and obturator signs

The doctor needs to do the above minimum examinations.

This copy of the material is licensed to Mohammed  Mirza.
Copyright © 2015. All Rights Reserved http://www.uworld.com
case19 Pt Notes  Print

Clinical Skills Evaluation

Case 19 Patient Notes

History:  Describe the history you just obtained from this patient.  Include only
information (pertinent positives and negatives) relevant to this patient’s problem(s).

45 ­YO man with 2 hours of worsening midepigastric to RLQ abdominal pain
Pain worse with movement but not relieved with anything
Nausea and vomiting (non­bloody, non­bilious)
Last bowel movement 20 hours ago with passage of flatus

ROS:  No fever, chills, dysuria, diarrhea, constipation, or back pain
PMHx:  Peptic ulcer disease treated in the past
PSHx:  None
Meds:  None
Allergies:  None
FHx:  Father, mother, and siblings healthy
SHx:  1 PPD smoker for past 20 years. He drinks 3 beers/day for past 15 years.

Physical examination:  Describe any positive and negative findings relevant to this
patient’s problem(s).  Be careful to include only those parts of examination performed in
this encounter.

Vital signs:  Temperature, 98.7 F (37.1 C); blood pressure, 130/80 mm Hg; heart
rate, 100/min; and respiratory rate, 20/min
Abdomen:  Right lower quadrant tenderness to superficial and deep palpation,
rebound tenderness noted, normoactive bowel sounds throughout, no
hepatosplenomegaly, no CVA tenderness, negative psoas and obturator signs

Data interpretation:  Based on what you have learned from the history and physical
examination, list up to 3 diagnoses that might explain this patient’s complaint(s).  List
your diagnoses from most to least likely.  For some cases, fewer than 3 diagnoses will be
appropriate.  Then enter the positive or negative findings from the history and the
physical examination (if present) that support each diagnosis.  Finally, list initial
diagnostic studies (if any) you would order for each listed diagnosis (eg, restricted
physical examination maneuvers, laboratory tests, imaging, ECG)

Diagnosis #1:  Appendicitis

History finding(s) Physical examination finding(s)
Sudden onset of RLQ abdominal pain RLQ abdominal tenderness
Nausea and vomiting Rebound tenderness in abdomen

Diagnosis #2:  Perforated peptic ulcer

History finding(s) Physical examination finding(s)
History of peptic ulcer Rebound tenderness in abdomen
Abdominal pain  
Alcohol use  

Diagnosis #3:  Intestinal obstruction

History finding(s) Physical examination finding(s)
Abdominal pain Rebound tenderness in abdomen
Last bowel movement 20 hours ago  

Diagnostic studies
CBC with differential
Abdominal x­ray
CT abdomen

This copy of the material is licensed to Mohammed  Mirza.
Copyright © 2015. All Rights Reserved http://www.uworld.com
case20 Scenario  Print

50­year­old male with bilateral leg pain

Vitals:
Pulse: 78/min
Temp: 98.0 F (36.7 C)
RR: 20/min
BP: 140/80 mmHg

Make a mental check list of DD for bilateral leg pain:
Thromboangiitis obliterans
Atherosclerotic vascular disease
Lumbar spinal stenosis
Diabetic polyneuropathy
Radiculopathy due to spinal disease
Medications, such as statins  
Trauma
Deep vein thrombosis (rarely bilateral)

Unilateral pain:
Cellulites/myofascitis
Deep vein thrombosis
Rupture of Bakers cyst
Osteomyelitis
Bleeding into the leg (if the patient is on warfarin/coumadin)
Radiculopathy
Pathological fracture of the bone

This copy of the material is licensed to Mohammed  Mirza.
Copyright © 2015. All Rights Reserved http://www.uworld.com
case20 SP  Print

If the doctor asks you any questions other than these, just say “no” or “things are
normal in my daily life.”

You are a 50­year­old man complaining of bilateral leg pain.

History of present illness:
You are a 50­year­old man who has had bilateral leg pain for 2 months.  The pain started
gradually and has been progressively increasing to a 5­6/10 severity.  The pain is
throbbing and is located mainly over the calf muscles without radiation.  It is made
worse with walking, running, and prolonged standing.  The symptoms improve with rest
and sitting.

You have no pain at rest, fever, trauma, swelling, back pain, weakness, sexual
difficulties, numbness, or tingling in the legs.

Past medical history:
Diabetes for the past 3 years on diet control
High cholesterol

Past surgical history:  None

Medications:  Simvastatin 40 mg daily at bedtime

Allergies:  None

Family history:  Father died at age 65 of a stroke.  Mother and 2 siblings are healthy.

Social history:  You are a postal worker and live with your wife.  You have smoked 2
packs per day for the past 30 years.  You drink a beer occasionally.  You have never
used illegal drugs.  You have no tattoos.

Physical examination:
The following are your temperature (T), blood pressure (BP), heart rate (HR), and
respiratory rate (RR).

Vital signs:  T 98.0 F (36.7 C), BP 140/80 mm Hg, HR 78/min, RR 20/min

Abdomen:  No bruits

Extremities:  Pulses 2+ and symmetrical in bilateral lower extremities

Musculoskeletal:  Negative Homan's sign, no calf tenderness to palpation bilaterally

Neurologic:  Bilateral lower extremities with 5/6 motor, grossly intact sensation, and
DTR 2+

The doctor needs to do the above minimum examinations.

This copy of the material is licensed to Mohammed  Mirza.
Copyright © 2015. All Rights Reserved http://www.uworld.com
case20 Pt Notes  Print

Clinical Skills Evaluation

Case 20 Patient Notes

History:  Describe the history you just obtained from this patient.  Include only
information (pertinent positives and negatives) relevant to this patient's problem(s).

50­YO man with 2 months of worsening bilateral leg pain
Throbbing pain mainly over the calf muscles made worse with walking, running,
and prolonged standing
Symptoms improve with rest and sitting

ROS:  No pain at rest, fever, trauma, swelling, back pain, weakness, sexual difficulties,
numbness, or tingling in legs
PMHx:  Diabetes for 3 years under diet control
          Hypercholesterolemia
PSHx:  None
Meds:  Simvastatin 40 mg daily at bedtime
Allergies:  None
FHx:  Father died at age 65 of stroke. Mother and other 2 siblings are healthy.
SHx:  2 PPD smoker for past 30 years, occasional alcohol use

Physical examination:  Describe any positive and negative findings relevant to this
patient's problem(s).  Be careful to include only those parts of examination performed in
this encounter.

Vital signs:  Temperature, of 98.0 F (36.7 C); blood pressure, 140/80 mm Hg; heart
rate, 78/min; and respiratory rate, 20/min
Abdomen:  No bruits
Extremities:  Pulses 2+ and symmetrical in bilateral lower extremities
Musculoskeletal:  Negative Homan's sign, no calf tenderness to palpation bilaterally
Neurologic:  Bilateral lower extremities with 5/5 motor strength, intact vibratory
sensation and proprioception, and DTR 2+

Data interpretation:  Based on what you have learned from the history and physical
examination, list up to 3 diagnoses that might explain this patient's complaint(s).  List
your diagnoses from most to least likely.  For some cases, fewer than 3 diagnoses will be
appropriate.  Then enter the positive or negative findings from the history and the
physical examination (if present) that support each diagnosis.  Finally, list initial
diagnostic studies (if any) you would order for each listed diagnosis (eg, restricted
physical examination maneuvers, laboratory tests, imaging, ECG)

Diagnosis #1:  Drug­induced (statin) myopathy

History finding(s) Physical examination finding(s)
Proximal muscle pain Normal motor strength in legs
History of statin use No sensory deficits in legs

Diagnosis #2:  Atherosclerotic peripheral vascular disease
History finding(s) Physical examination finding(s)
History of diabetes and high cholesterol  
History of smoking  
Family history of stroke  

Diagnosis #3:  Diabetic polyneuropathy

History finding(s) Physical examination finding(s)
Diet­controlled diabetes  
History of high cholesterol  

Diagnostic studies
Creatinine kinase
Fasting blood sugar and hemoglobin A1C
Lower­extremity arterial Doppler

This copy of the material is licensed to Mohammed  Mirza.
Copyright © 2015. All Rights Reserved http://www.uworld.com
case21 Scenario  Print

40­year­old male with vomiting of blood

Vitals:
PR: 88/min
Temp: 98.0 F (36.7 C)
RR: 20/min
BP: 110/80 mmHg

Make a mental checklist of DD for hematemesis:
Peptic ulcer decease
Gastric erosions  
Esophageal varices
Mallory­Weiss tear
Esophagitis  
Duodenitis
Malignancy (esophageal and gastric)

This copy of the material is licensed to Mohammed  Mirza.
Copyright © 2015. All Rights Reserved http://www.uworld.com
case21 SP  Print

If the doctor asks you any questions other than these, just say “no” or “things are
normal in my daily life.”

You are a 40­year­old man complaining of vomiting up blood.

History of present illness:
You are a 40­year­old man who has vomited up blood for 2 hours.  You were well until 1
day ago when you developed sharp mid­epigastric abdominal pain of 4­5/10 severity and
no radiation.  You also developed nausea that worsened today, throwing up twice with
almost a teaspoonful of blood each time.  You have also had black stools for the past
week.

You have no changes in appetite, weight loss, fever, chills, shortness of breath,
dizziness, or chest pain.

Past medical history:  GERD for past 2 years relieved with antacids as needed; Chronic
back pain.

Past surgical history:  None

Medications:  Ibuprofen as needed

Allergies:  None

Family history:  Father died at age 60 of heart attack.  Mother and 2 siblings are
healthy.

Social history:  You are a postal worker and live with your wife.  You have smoked 1
pack a day for the past 25 years.  You have consumed 4­5 beers daily for the past 20
years.  You have never used illegal drugs.  You have no tattoos.

Physical examination:
The following are your temperature (T), blood pressure (BP), heart rate (HR), and
respiratory rate (RR).

Vital signs:  T 98.0 F (36.7 C), BP 110/80 mm Hg, HR 88/min, RR 20/min

HEENT:  Oropharynx clear

Abdomen:  Nontender, nondistended, normoactive bowel sounds, no
hepatosplenomegaly

The doctor needs to do the above minimum examinations.

This copy of the material is licensed to Mohammed  Mirza.
Copyright © 2015. All Rights Reserved http://www.uworld.com
case21 Pt Notes  Print

Clinical Skills Evaluation

Case 21 Patient Notes

History:  Describe the history you just obtained from this patient.  Include only
information (pertinent positives and negatives) relevant to this patient's problem(s).

40­YO man with 2 hours of hematemesis with 1 teaspoon of blood
History of GERD relieved with as­needed antacids
One day of midepigastric abdominal pain without radiation associated with nausea
One week of melena

ROS:  No changes in appetite, weight loss, fever, chills, shortness of breath, dizziness, or
chest pain
PMHx:  GERD; Chronic back pain
PSHx:  None
Meds:  Ibuprofen, as needed
Allergies:  None
FHx:  Father died at age 60 of heart attack.  Mother and 2 siblings are healthy.
SHx:  1 PPD smoker for past 20 years, 4­5 beers/day for past 20 years

Physical examination:  Describe any positive and negative findings relevant to this
patient's problem(s).  Be careful to include only those parts of examination performed in
this encounter.

Vital signs:  Temperature, 98.0 F (36.7 C); blood pressure,110/80 mm Hg; heart
rate, 88/min; and respiratory rate, 20/min
HEENT:  Oropharynx clear
Abdomen:  Nontender, nondistended, normoactive bowel sounds, no
hepatosplenomegaly

Data interpretation:  Based on what you have learned from the history and physical
examination, list up to 3 diagnoses that might explain this patient's complaint(s).  List
your diagnoses from most to least likely.  For some cases, fewer than 3 diagnoses will be
appropriate.  Then enter the positive or negative findings from the history and the
physical examination (if present) that support each diagnosis.  Finally, list initial
diagnostic studies (if any) you would order for each listed diagnosis (eg, restricted
physical examination maneuvers, laboratory tests, imaging, ECG)

Diagnosis #1:  Bleeding peptic ulcer

History finding(s) Physical examination finding(s)
History of ibuprofen and excessive alcohol use  
Midepigastric pain  
Hematemesis and melena  

Diagnosis #2:  Esophageal variceal bleeding
History finding(s) Physical examination finding(s)
History of GERD and excessive alcohol use  
Midepigastric pain and hematemesis  
Melena  

Diagnosis #3:  Gastritis

History finding(s) Physical examination finding(s)
History of ibuprofen and excessive alcohol use  
Midepigastric pain  
Melena  

Diagnostic studies
Orthostatic BP and HR measurements
CBC with differential, Basic metabolic panel
Liver function tests and PT/INR
Upper GI endoscopy

This copy of the material is licensed to Mohammed  Mirza.
Copyright © 2015. All Rights Reserved http://www.uworld.com
case22 Scenario  Print

55 Yr. O/M Complaining of Chest Pain
Vitals

Pulse­­78/min
Temp­­98.7
R.Rate­­20/min
B.P­­130/80 mm of Hg.

Make a mental check list for Chest pain 

Myocardial Infarction
Unstable Angina
Pulmonary Embolism
Costochondritis
Pleuritis
Pericarditis
Aortic Dissection  
G.E.R.D
Esophageal perforation

This copy of the material is licensed to Mohammed  Mirza.
Copyright © 2015. All Rights Reserved http://www.uworld.com
case22 SP  Print

If the doctor asks you any questions other than these, just say “no” or “things are
normal in my daily life.”

You are a 55­year­old man complaining of chest pain.

History of present illness:
You are a 55­year­old man who comes to the physician with 2 hours of chest pain.  The
pain came on suddenly and has progressively worsened to 8­9/10 severity.  You describe
the pain as tightness and squeezing in the substernal area without radiation.  It is worse
when walking and moving around.  The pain has been fairly constant and is associated
with nausea, 1 episode of vomiting, sweating, and mild shortness of breath.

You have no fever, cough, headache, abdominal pain, diarrhea, constipation, recent
trauma, appetite changes, weight loss, or urinary problems.

Past medical history:
High blood pressure for 20 years
Diabetes for 5 years
Cholesterol tested a year ago was 280 (total).  You are trying to control your cholesterol
with diet.

Past surgical history:  None

Medications:
Atenolol 50 mg a day
Metformin 500 mg a day

Allergies:  None

Family history:  Father died at age 60 of a heart attack.  Mother is living and had a
stroke at age 65.  Your brother had a heart attack at age 58.

Social history:  You live with your wife and work as a lawyer.  You have smoked 1 pack
a day for the past 30 years.  You have consumed 1 glass of wine daily for the past 20
years.  You eat a lot of fast food.  You have never used illegal drugs.  You have no
tattoos.

Physical examination:
The following are your temperature (T), blood pressure (BP), heart rate (HR), and
respiratory rate (RR).

Vital signs:  T 98.7 F (37.1 C), BP 130/80 mm Hg, HR 78/min, RR 20/min

Neck:  Supple without JVD or lymphadenopathy, no thyromegaly

Lungs:  Clear to auscultation bilaterally.  No reproducible chest pain with palpation

Heart:  Regular rate and rhythm without murmurs, gallops, or rubs

The doctor needs to do the above minimum examinations.

This copy of the material is licensed to Mohammed  Mirza.
Copyright © 2015. All Rights Reserved http://www.uworld.com
case22 Pt Notes  Print

Clinical Skills Evaluation

Case 22 Patient Notes

History:  Describe the history you just obtained from this patient.  Include only
information (pertinent positives and negatives) relevant to this patient's problem(s).

55­YO man with 2 hours of chest pain described as substernal tightness and
pressure increased with movement and walking.  The pain is 8­9/10 in severity. 
There is no radiation.
Nausea, vomiting, sweating, and shortness of breath are associated with pain.

ROS:  No fever, cough, headache, abdominal pain, diarrhea, constipation, recent trauma,
appetite changes, weight loss, or urinary problems
PMHx:  Hypertension, diabetes, high cholesterol
PSHx:  None
Meds:  Atenolol 50 mg a day, metformin 500 mg a day
Allergies:  None
FHx:  Father died at age 60 of heart attack.  Mother is living and had a stroke at age 65. 
Brother had a heart attack at age 58.
SHx:  1 PPD smoker for past 30 years, 1 glass of wine/day for past 20 years

Physical examination:  Describe any positive and negative findings relevant to this
patient's problem(s).  Be careful to include only those parts of examination performed in
this encounter.

Vital signs:  Temperature, 98.7 F (37.1 C); blood pressure, 130/80 mm Hg; heart
rate, 78/min; and respiratory rate, 20/min
Neck:  Supple without JVD or lymphadenopathy, no thyromegaly
Lungs:  Clear to auscultation bilaterally.  No reproducible chest pain with palpation
Heart:  Regular rate and rhythm without murmurs, gallops, or rubs

Data interpretation:  Based on what you have learned from the history and physical
examination, list up to 3 diagnoses that might explain this patient's complaint(s).  List
your diagnoses from most to least likely.  For some cases, fewer than 3 diagnoses will be
appropriate.  Then enter the positive or negative findings from the history and the
physical examination (if present) that support each diagnosis.  Finally, list initial
diagnostic studies (if any) you would order for each listed diagnosis (eg, restricted
physical examination maneuvers, laboratory tests, imaging, ECG)

Diagnosis #1:  Acute coronary syndrome

History finding(s) Physical examination finding(s)
Substernal chest pressure No reproducible chest pain with palpation
History of untreated high cholesterol  
Nausea, vomiting, diaphoresis  

Diagnosis #2:  Aortic dissection
History finding(s) Physical examination finding(s)
History of hypertension No reproducible chest pain with palpation
Substernal chest pain  
Sudden onset of symptoms  

Diagnosis #3:  Pulmonary embolism

History finding(s) Physical examination finding(s)
Sudden onset of chest pain No reproducible chest pain with palpation
Shortness of breath  

Diagnostic studies
EKG
Chest x­ray
Cardiac enzymes
Echocardiogram

This copy of the material is licensed to Mohammed  Mirza.
Copyright © 2015. All Rights Reserved http://www.uworld.com
case23 Scenario  Print

70 Yr. O/M complaining of Frequent Falls
Vitals

Pulse­­78/min
Temp­­98.7
R.Rate­­20/min
B.P­­130/80 mm of Hg.

Make a mental checklist of differential diagnosis for falls

Cerebellar disease [Alcoholic/Tumor]
Parkinson's disease [Idiopathic/drug induced]
Diabetic neuropathy [Sensory ataxia]
Brain tumors
Hyperthyroidism
Fractured hip
Seizure
Vertigo

This copy of the material is licensed to Mohammed  Mirza.
Copyright © 2015. All Rights Reserved http://www.uworld.com
case23 SP  Print

If the doctor asks you any questions other than these, just say “no” or “things are
normal in my daily life.”

You are a 70­year­old man complaining of frequent falls.

History of present illness:
You are a 70­year­old man who has fallen frequently for the past 2 months.  You were
well until 2 months ago when you started having problems with your balance while
standing, leading to falls.  These episodes initially occurred once a week but have
increased to twice a week over the last month.  You have not fractured any bones but
are concerned that this may happen in the future.  You have also noticed a tremor in
your hands that makes it difficult to hold things; it worsens when you reach for an
object.  A friend also noticed that your speech has changed.  You also have occasional
headache in the morning.  There is no numbness of tingling in both legs.

You have no dizziness, vertigo, leg weakness, fever, nausea, vomiting, hair loss, chest
pain, abdominal pain, recent trauma, diarrhea, constipation, or urinary problems.

Past medical history:  Diabetes for past 10 years under good control

Past surgical history:  None

Medications:  Metformin 500 mg a day

Allergies:  None

Family history:  Father and mother both died of old age.  You have no siblings.

Social history:  Your wife passed away 5 years ago and you live alone.  You do not
smoke.  You have consumed 2 beers daily for the past 40 years.  You have not used illicit
drugs.

Physical examination:
The following are your temperature (T), blood pressure (BP), heart rate (HR), and
respiratory rate (RR).

Vital signs:  T 98.7 F (37.1 C), BP 130/80 mm Hg, HR 78/min, RR 20/min

Check for orthostatic vitals

HEENT:  Visual acuity testing

Neck:  Supple without JVD or lymphadenopathy, no thyromegaly, no bruits

Lungs:  Clear to auscultation bilaterally

Heart:  Regular rate and rhythm without murmurs, gallops, or rubs

Neurologic:  Motor 5/5 bilaterally, sensory grossly intact bilaterally, resting tremor, mild
dysmetria (finger to nose) present, mild dysdiadochokinesia (alternating movements),
DTR 2+ bilaterally

The doctor needs to do the above minimum examinations.

This copy of the material is licensed to Mohammed  Mirza.
Copyright © 2015. All Rights Reserved http://www.uworld.com
Case23 Pt Notes  Print

Clinical Skills Evaluation

Case 23 Patient Notes

History:  Describe the history you just obtained from this patient.  Include only
information (pertinent positives and negatives) relevant to this patient's problem(s).

70­YO man with 2 months of frequent falls
Has had balance problems while standing up; no syncope
Tremor in hands worse with reaching for objects
Change in speech, occasional morning headache
No sensory symptoms (numbness, tingling) in legs
No dizziness or vertigo

ROS:  No fever, nausea, vomiting, hair loss, chest pain, abdominal pain, recent trauma,
diarrhea, constipation, or urinary problems
PMHx:  Diabetes
PSHx:  None
Meds:  Metformin 500 mg a day
Allergies:  None
FHx:  Father and mother both died of old age.
SHx:  No smoking, 2 beers daily for past 40 years

Physical examination:  Describe any positive and negative findings relevant to this
patient's problem(s).  Be careful to include only those parts of examination performed in
this encounter.

Vital signs:  Temperature, 98.7 F (37.1 C); blood pressure, 130/80 mm Hg; heart
rate, 78/min; and respiratory rate, 20/min
HEENT:  PERRLA, EOMI; Visual acuity ­ normal
Neck:  Supple without JVD or lymphadenopathy, no thyromegaly, no bruits
Lungs:  Clear to auscultation bilaterally
Heart:  Regular rate and rhythm without murmurs, gallops, or rubs
Neurologic:  Motor 5/5 bilaterally, sensory grossly intact bilaterally, resting tremor,
mild dysmetria (finger to nose) present, mild dysdiadochokinesia (alternating
movements), DTR 2+ bilaterally

Data interpretation:  Based on what you have learned from the history and physical
examination, list up to 3 diagnoses that might explain this patient's complaint(s).  List
your diagnoses from most to least likely.  For some cases, fewer than 3 diagnoses will be
appropriate.  Then enter the positive or negative findings from the history and the
physical examination (if present) that support each diagnosis.  Finally, list initial
diagnostic studies (if any) you would order for each listed diagnosis (eg, restricted
physical examination maneuvers, laboratory tests, imaging, ECG)

Diagnosis #1:  Cerebellar disease due to alcohol use

History finding(s) Physical examination finding(s)
History of chronic alcohol use Dysmetria
Difficulty with balance Dysdiadochokinesia

Diagnosis #2:  Parkinson�s disease

History finding(s) Physical examination finding(s)
Tremor Resting tremor
Balance problems  

Diagnosis #3:  Brain tumor

History finding(s) Physical examination finding(s)
Speech difficulties; headache  
Balance problems  
2 Months of symptom onset  

Diagnostic studies
Orthostatic vitals
Brain imaging (CT or MRI)
Basic metabolic panel
Thyroid function tests; Vitamin B 12 levels
Complete blood count

This copy of the material is licensed to Mohammed  Mirza.
Copyright © 2015. All Rights Reserved http://www.uworld.com
case24 Scenario  Print

35 Yr. O/M Complaining of Cough and Chest Pain
Vitals

Pulse­­94/min
Temp ­­101.7
R.Rate ­­24/min
B.P­­130/80 mm of Hg.

Make a mental check list for Cough and Chest pain 

Pneumonia
Pleuritic pain
Pleural effusion
Pulmonary edema
Tuberculosis
Pulmonary embolism
Ca. Bronchus
Infective endocarditis
GERD

This copy of the material is licensed to Mohammed  Mirza.
Copyright © 2015. All Rights Reserved http://www.uworld.com
case24 SP  Print

If the doctor asks you any questions other than these, just say “no” or “things are
normal in my daily life.”

You are a 35­year­old man complaining of cough.

History of present illness:
You are a 35­year­old man who comes to the physician with 2 days of coughing.  You
initially developed malaise followed by productive cough of blood­tinged, yellow sputum. 
You subsequently had sharp pain described as 5­6/10 intensity over the left chest.  The
pain is worsened with moving around and deep inspiration and improved with expiration. 
You have also had fever, chills, sweating, and mild shortness of breath.  One of your
colleagues in the office recently had pneumonia.

You ask:  “Is this pneumonia?”

You have no changes in appetite, weight loss, abdominal pain, recent trauma, diarrhea,
constipation, or urinary problems.

Past medical history:  Hospitalized once for chest pain 5 years ago with negative
testing

Past surgical history:  None

Medications:  None

Allergies:  Penicillin (rash)

Family history:  Father and mother are healthy.  You have no siblings.

Social history:  You live alone and work as an investment advisor for a local firm.  You
have smoked 1 pack a day for the past 15 years.  You have consumed 2 shots of liquor
every weekend with friends for the past 10 years.  You have not used illicit drugs and
have no tattoos.

Physical examination:
The following are your temperature (T), blood pressure (BP), heart rate (HR), and
respiratory rate (RR).

Vital signs:  T 101.7 F (38.7 C), BP 130/80 mm Hg, HR 94/min, RR 24/min

HEENT:  Oropharynx clear

Neck:  Supple without JVD or lymphadenopathy, no thyromegaly, no bruits, no accessory
muscle use

Lungs:  Clear to auscultation bilaterally, fremitus symmetrical bilaterally, resonant to
percussion bilaterally, no bronchophony or egophony

Heart:  Regular rate and rhythm without murmurs, gallops, or rubs

Abdomen:  Nontender, nondistended, no hepatosplenomegaly, normoactive bowel
sounds

The doctor needs to do the above minimum examinations.

This copy of the material is licensed to Mohammed  Mirza.
Copyright © 2015. All Rights Reserved http://www.uworld.com
Case24 Pt Notes  Print

Clinical Skills Evaluation

Case 24 Patient Notes

History:  Describe the history you just obtained from this patient.  Include only
information (pertinent positives and negatives) relevant to this patient's problem(s).

35 YO male with 2 days of productive cough of yellow blood­tinged sputum
2 days of sharp left­sided chest pain worsened with inspiration and improved with
expiration
Patient has fever, chills, sweating, and mild shortness of breath
Sick contact in office

ROS:  No changes in appetite, weight changes, abdominal pain, recent trauma, diarrhea,
constipation, or urinary problems.
PMHx:  None
PSHx:  None
Meds:  None
Allergies:  Penicillin (rash)
FHx:  Father and mother are healthy
SHx:  1 PPD smoker for past 15 years, 2 shots a week for past 10 years

Physical examination:  Describe any positive and negative findings relevant to this
patient's problem(s).  Be careful to include only those parts of examination performed in
this encounter.

Vital signs:  temperature of 101.7 F (38.7 C), blood pressure of 130/80, heart rate
of 94/min, and respiratory rate of 24/min
HEENT:  Oropharynx clear
Neck:  Supple without JVD or lymphadenopathy, no thyromegaly, no bruits, no
accessory muscle use
Lungs:  Clear to auscultation bilaterally, fremitus symmetrical bilaterally, resonant
to percussion bilaterally, no bronchophony or egophony
Heart:  Regular rate and rhythm without murmurs, gallops, or rubs
Abdomen:  Nontender, nondistended, no hepatosplenomegaly, normoactive bowel
sounds

Data interpretation:  Based on what you have learned from the history and physical
examination, list up to 3 diagnoses that might explain this patient's complaint(s).  List
your diagnoses from most to least likely.  For some cases, fewer than 3 diagnoses will be
appropriate.  Then enter the positive or negative findings from the history and the
physical examination (if present) that support each diagnosis.  Finally, list initial
diagnostic studies (if any) you would order for each listed diagnosis (eg, restricted
physical examination maneuvers, laboratory tests, imaging, ECG)

Diagnosis #1:  Bronchitis

History finding(s) Physical examination finding(s)
Smoking history Fever
Fever  
Productive cough of yellow blood­tinged
 
sputum

Diagnosis #2:  Pneumonia

History finding(s) Physical examination finding(s)
Fever and chills Fever
Productive cough of blood­tinged yellow Respiratory rate 24/min
sputum
Pleuritic chest pain  
Sick contact at office  

Diagnosis #3:  Lung cancer

History finding(s) Physical examination finding(s)
Smoking history  
Productive cough of blood­tinged sputum  

Diagnostic studies
Chest x­ray
Sputum gram stain and culture
CBC with differential

This copy of the material is licensed to Mohammed  Mirza.
Copyright © 2015. All Rights Reserved http://www.uworld.com
case25 Scenario  Print

60­year­old male complaining of lower abdominal pain
 

Vitals:
PR: 98/min
BP: 130/84 mmHg
T: 101 F (38.3 C)
RR: 22/min

Make a mental checklist of Differential Diagnosis:
Diverticulitis
Renal colic
Appendicitis
Ischemic colitis
Infectious colitis
Leaking aneurysm
Intestinal obstruction from strangulated or incarcerated hernia

This copy of the material is licensed to Mohammed  Mirza.
Copyright © 2015. All Rights Reserved http://www.uworld.com
case25 SP  Print

If the doctor asks you any questions other than these, just say “no” or “things are
normal in my daily life.”

You are a 60­year­old man complaining of lower abdominal pain.

History of present illness:
You are a 60­year­old man who has had 1 day of lower abdominal pain.  You went to
dinner at a restaurant yesterday with family members and later developed left lower
abdominal pain described as 6/10 severity.  The pain is episodic and crampy and lasts
10­15 minutes at a time.  It started slowly and got progressively worse.  There is no
radiation or aggravating/alleviating factors.  You had nausea with 1 episode of non­
bilious and non­bloody vomiting and a low­ grade subjective fever.  You also had 2­3
episodes of diarrhea with blood but without black stools after the pain started.  Your
other family members have no symptoms and are well.

You have no recent travel, sick contacts, chills, or urinary problems.

Past medical history:
Hospitalized once 10 years ago for kidney stone that passed spontaneously
Hypertension

Past surgical history:  None

Medications:  Hydrochlorothiazide 25 mg daily

Allergies:  None

Family history:  Father died at age 65 of colon cancer.  Mother died at age 70 of breast
cancer.  You have no siblings.

Social history:  You live with your wife and work as a financial planner for a local firm. 
You have never smoked.  You have consumed 1 beer daily for the past 30 years.  You
have not used illicit drugs and have no tattoos.

Physical examination:
The following are your temperature (T), blood pressure (BP), heart rate (HR), and
respiratory rate (RR).

Vital signs:  T 101.0 F (38.3 C), BP 130/84 mm Hg, HR 98/min, RR 22/min

Heart:  Regular rate and rhythm without murmurs, gallops, or rubs

Abdomen:  LLQ tenderness to deep palpation, no rebound tenderness, nondistended, no
hepatosplenomegaly, normoactive bowel sounds, no CVA tenderness

The doctor needs to do the above minimum examinations.

This copy of the material is licensed to Mohammed  Mirza.
Copyright © 2015. All Rights Reserved http://www.uworld.com
Case25 Pt Notes  Print

Clinical Skills Evaluation

Case 25 Patient Notes

History:  Describe the history you just obtained from this patient.  Include only
information (pertinent positives and negatives) relevant to this patient's problem(s).

60­YO man with 1 day of episodic crampy LLQ abdominal pain lasting 10­15
minutes each time without radiation
Pain is 7/10
Ate at a local restaurant with family, but no one else is sick
He has had nausea, 1 episode of vomiting, 2­3 episodes of diarrhea with blood, and
subjective fever

ROS:  No recent travel, sick contacts, chills, or urinary problems
PMHx:  Hypertension, kidney stone in past
PSHx:  None
Meds:  Hydrochlorothiazide 25 mg daily
Allergies:  None
FHx:  Father died at age 65 of colon cancer.  Mother died at age 70 of breast cancer.
SHx:  No smoking, 1 beer daily for past 30 years

Physical examination:  Describe any positive and negative findings relevant to this
patient's problem(s).  Be careful to include only those parts of examination performed in
this encounter.

Vital signs:  Temperature, 101.0 F (38.3 C); blood pressure, 130/84 mm Hg; heart
rate, 98/min; and respiratory rate, 22/min
Heart:  Regular rate and rhythm without murmurs, gallops, or rubs
Abdomen:  LLQ tenderness to deep palpation, no rebound tenderness,
nondistended, no hepatosplenomegaly, normoactive bowel sounds, no CVA
tenderness

Data interpretation:  Based on what you have learned from the history and physical
examination, list up to 3 diagnoses that might explain this patient's complaint(s).  List
your diagnoses from most to least likely.  For some cases, fewer than 3 diagnoses will be
appropriate.  Then enter the positive or negative findings from the history and the
physical examination (if present) that support each diagnosis.  Finally, list initial
diagnostic studies (if any) you would order for each listed diagnosis (eg, restricted
physical examination maneuvers, laboratory tests, imaging, ECG)

Diagnosis #1:  Acute diverticulitis

History finding(s) Physical examination finding(s)
Abdominal pain Fever
Fever LLQ abdominal tenderness to palpation
Diarrhea with blood  
Diagnosis #2:  Infectious colitis

History finding(s) Physical examination finding(s)
Fever Fever
Ate at local restaurant before symptom onset Abdominal tenderness to palpation
Diarrhea with blood  

Diagnosis #3:  Ischemic colitis

History finding(s) Physical examination finding(s)
History of hypertension Fever
Fever  
Abdominal pain  

Diagnostic studies
Rectal examination with stool guaiac
Abdominal imaging (X­ray, CT scan)
CBC with differential
Urinalysis

This copy of the material is licensed to Mohammed  Mirza.
Copyright © 2015. All Rights Reserved http://www.uworld.com
case26 Scenario  Print

35­year­old male complains of fatigue

Vitals:

PR:      82/min
BP:      120/80 mmHg
RR:      16/min
Temp:  96.8 F (36.0 C)

Mental Checklist of DD:

Depression
Anemia
Hypo or Hyperthyroidism
HIV or AIDS
Malingering
Hypochondriasis
Post Traumatic Stress Disorder
Chronic fatigue syndrome

This copy of the material is licensed to Mohammed  Mirza.
Copyright © 2015. All Rights Reserved http://www.uworld.com
case26 SP  Print

If the doctor asks you any questions other than these, just say “no” or “things are
normal in my daily life.”

You are a 35­year­old man complaining of fatigue.

History of present illness:
You are a 35­year­old man who comes to the physician with 2 months of fatigue.  You
were previously well until you were robbed 2 months ago.  You have become unable to
fall asleep at night, get tired easily during the day, have nightmares every night, and
generally feel anxious all the time.  You have been unable to concentrate at work, have
restricted your daily activities, and feel emotionally distant and lonely.  You have also
had 3­4 months of constipation.  You note no increased stress at work or home.

You have no shortness of breath, chest pain, palpitations, sweating, fever, chills, weight
loss, or changes in appetite.

Past medical history:  None

Past surgical history:  None

Medications:  None

Allergies:  None

Family history:  Father, mother, and 3 siblings are healthy.

Social history:  You live with your girlfriend and work as a florist.  You have smoked 1
pack a day for the past 15 years and drink occasionally.  You have not used illicit drugs
and have no tattoos.

Physical examination:
The following are your temperature (T), blood pressure (BP), heart rate (HR), and
respiratory rate (RR).

Vital signs:  T 96.8 F (36.0 C), BP 120/80 mm Hg, HR 82/min, RR 16/min

HEENT:  No pallor, oropharynx clear

Neck:  Supple without lymphadenopathy or thyromegaly

Heart:  Regular rate and rhythm without murmurs, gallops, or rubs

Lungs:  Clear to auscultation

Abdomen:  No masses or tenderness

Psychiatric:  Alert and oriented to person, place, and time.

The doctor needs to do the above minimum examinations.

This copy of the material is licensed to Mohammed  Mirza.
Copyright © 2015. All Rights Reserved http://www.uworld.com
Case26 Pt Notes  Print

Clinical Skills Evaluation

Case 26 Patient Notes

History:  Describe the history you just obtained from this patient.  Include only
information (pertinent positives and negatives) relevant to this patient's problem(s).

35­YO man with 2 months of fatigue after robbery
He has insomnia, increased daytime fatigue, nightmares every night, generalized
anxiety throughout the day, and inability to concentrate at work.
He feels emotionally alone and distant with no increased stress at work or home.
No hallucinations or delusions.
Constipation for 3­4 months.

ROS:  No shortness of breath, chest pain, palpitations, sweating, fever, chills, weight
loss, or changes in appetite
PMHx:  None
PSHx:  None
Meds:  None
Allergies:  None
FHx:  Father, mother, and 3 siblings are healthy.
SHx:  1 PPD smoker for 15 years, occasional alcohol use

Physical examination:  Describe any positive and negative findings relevant to this
patient's problem(s).  Be careful to include only those parts of examination performed in
this encounter.

Vital signs:  Temperature, 96.8 F (36.0 C); blood pressure, 120/80 mm Hg; heart
rate, 82/min; and respiratory rate, 16/min
HEENT:  No pallor, oropharynx clear
Neck:  Supple without lymphadenopathy or thyromegaly
Heart:  Regular rate and rhythm without murmurs, gallops, or rubs
Lungs:  Clear to auscultation
Abdomen:  No masses or tenderness
Psychiatric:  Alert and oriented to person, place, and time.

Data interpretation:  Based on what you have learned from the history and physical
examination, list up to 3 diagnoses that might explain this patient's complaint(s).  List
your diagnoses from most to least likely.  For some cases, fewer than 3 diagnoses will be
appropriate.  Then enter the positive or negative findings from the history and the
physical examination (if present) that support each diagnosis.  Finally, list initial
diagnostic studies (if any) you would order for each listed diagnosis (eg, restricted
physical examination maneuvers, laboratory tests, imaging, ECG)

Diagnosis #1:  Post­traumatic stress disorder

History finding(s) Physical examination finding(s)
Symptom onset after robbery 2 months ago  
Insomnia  
Difficulty concentrating, nightmares  

Diagnosis #2:  Depression

History finding(s) Physical examination finding(s)
Fatigue  
Insomnia  
Emotionally feels alone and distant  

Diagnosis #3:  Hypothyroidism

History finding(s) Physical examination finding(s)
Constipation for 3­4 months  
Fatigue  
Inability to concentrate at work  

Diagnostic studies
Thyroid function tests
CBC with differential
ESR
Basic metabolic panel

This copy of the material is licensed to Mohammed  Mirza.
Copyright © 2015. All Rights Reserved http://www.uworld.com
case27 Scenario  Print

65­year­old female complaining of loss of hearing

Vitals:
PR: 80/min
BP: 130/86 mmHg
T: 98.0 F (36.7 C)
RR: 16/min
 
Make a mental check list of DD for loss of hearing:

Conducting hearing loss:
Cerumen impaction
Otitis media with effusion
Tympanic membrane perforation
Otosclerosis
Foreign body in ear canal
Cholesteatoma
Tympanosclerosis
Tumor of the ear canal or middle ear

Sensorineural hearing loss:
Presbycusis (hearing loss with aging)
Ototoxicity
Noise­induced loss
Meniere's disease
Diabetes
Acoustic neuroma

This copy of the material is licensed to Mohammed  Mirza.
Copyright © 2015. All Rights Reserved http://www.uworld.com
case27 SP  Print

If the doctor asks you any questions other than these, just say “no” or “things are
normal in my daily life.”

You are a 65­year­old woman complaining of hearing loss.

History of present illness:
You are a 65­year­old woman who comes to the physician with 3 months of reduced
hearing in the left ear.  Your hearing was initially decreased in this ear, but it has
progressively worsened.  It is difficult for you to hear people with high­pitched voices.
 You were admitted to the hospital 4 months ago for a kidney infection and treated with
IV antibiotics but can’t remember the name of the drug.  Nothing seems to improve or
worsen your hearing difficulties.

You have no discharge from the ear, ringing in the ear, dizziness, facial muscle
weakness, weakness or numbness in other parts of the body, or earache.

Past medical history:  Hypertension for the past 30 years

Past surgical history:  None

Medications:  Hydrochlorothiazide 50 mg daily

Allergies:  None

Family history:  Father and mother died of old age.  Your 2 siblings are healthy.

Social history:  You live with your husband and work as a supervisor at a steel factory
where you are exposed to loud noises.  You do not smoke but drink occasionally.  You
have not used illicit drugs and have no tattoos.

Physical examination:
The following are your temperature (T), blood pressure (BP), heart rate (HR), and
respiratory rate (RR).

Vital signs:  T 98.0 F (36.7 C), BP 130/86 mm Hg, HR 80/min, RR 16/min

HEENT:  PERRLA, EOMI, oropharynx clear, TM clear bilaterally, Rinne with AC>BC,
Weber localizes to the right ear, oropharynx clear

Neck:  Supple without lymphadenopathy or thyromegaly

Neurologic:  Alert and oriented to person, place, and time.  Cranial nerves 2­12 intact,
except for decreased hearing on the left ear

The doctor needs to do the above minimum examinations.

This copy of the material is licensed to Mohammed  Mirza.
Copyright © 2015. All Rights Reserved http://www.uworld.com
case27 Pt Notes  Print

Clinical Skills Evaluation

Case 27 Patient Notes

History:  Describe the history you just obtained from this patient.  Include only
information (pertinent positives and negatives) relevant to this patient’s problem(s).

65­YO woman with 3 months of high­frequency hearing loss
Hospitalized for pyelonephritis 4 months ago and treated with unknown antibiotic
Exposed to loud noises at work in steel factory

ROS:  No discharge from ear, ringing in the ear, dizziness, facial muscle weakness,
weakness or numbness in other parts of the body, or earache
PMHx:  Hypertension
PSHx:  None
Meds:  Hydrochlorothiazide 50 mg daily
Allergies:  None
FHx:  Father and mother died of old age, healthy siblings
SHx:  No smoking, occasional alcohol use

Physical examination:  Describe any positive and negative findings relevant to this
patient’s problem(s).  Be careful to include only those parts of examination performed in
this encounter.

Vital signs:  Temperature, 98.0 F (36.7 C); blood pressure, 130/86 mm Hg; heart
rate, 80/min; and respiratory rate, 16/min
HEENT:  PERRLA, EOMI, oropharynx clear, TM clear bilaterally, Rinne with AC>BC,
Weber localizes to the right ear, oropharynx clear
Neck:  Supple without lymphadenopathy or thyromegaly
Neurologic:  Alert and oriented to person, place, and time.  Cranial nerves 2­12
intact except for decreased hearing in left ear.

Data interpretation:  Based on what you have learned from the history and physical
examination, list up to 3 diagnoses that might explain this patient’s complaint(s).  List
your diagnoses from most to least likely.  For some cases, fewer than 3 diagnoses will be
appropriate.  Then enter the positive or negative findings from the history and the
physical examination (if present) that support each diagnosis.  Finally, list initial
diagnostic studies (if any) you would order for each listed diagnosis (eg, restricted
physical examination maneuvers, laboratory tests, imaging, ECG)

Diagnosis #1:  Presbycusis (age­related hearing loss)

History finding(s) Physical examination finding(s)
Decreased hearing for 2­3 months Weber localizing to right ear
High­frequency hearing loss  

Diagnosis #2:  Noise­induced hearing loss

History finding(s) Physical examination finding(s)
Decreased hearing Weber localized to right ear
Works at steel factory with loud noises  

Diagnosis #3:  Drug­induced hearing loss

History finding(s) Physical examination finding(s)
History of recent antibiotic use Weber localized to right ear

Diagnostic studies
Audiometric testing
ESR
MRI of brain

This copy of the material is licensed to Mohammed  Mirza.
Copyright © 2015. All Rights Reserved http://www.uworld.com
case28 Scenario  Print

A 53­year­old male with right knee pain and swelling

Vitals:
PR: 80/min, regular
BP: 130/60 mmHg
RR: 18/min
T: 101.0 F (38.3 C)

Mental Checklist of DD:
Osteoarthritis
Septic arthritis
Pseudogout and gout
Reactive arthritis
Traumatic knee injury
Lyme disease
Monoarticular rheumatoid arthritis
Psoriatic arthritis

This copy of the material is licensed to Mohammed  Mirza.
Copyright © 2015. All Rights Reserved http://www.uworld.com
case28 SP  Print

If the doctor asks you any questions other than these, just say “no” or “things are
normal in my daily life.”

You are a 53­year­old man complaining of right knee pain.

History of present illness:
You are a 53­year­old man who has had 2 days of worsening right knee pain.  You were
well until 1 year ago when you developed right and left knee pain made mildly worse
with walking.  Initially you thought that it was due to your heavy weight.  You also have
stiffness in multiple joints every morning for 10­15 minutes that resolves spontaneously.

The pain worsened acutely 2 days ago in the right knee.  You describe it as a throbbing
sensation of 7/10 severity with no radiation.  You took ibuprofen without relief.  There
are no aggravating or relieving factors and no recent trauma.

You have no fever, chills, nausea, vomiting, diarrhea, constipation, rashes, travel
history, sick contacts, insect bites, or urinary complaints.

Past medical history:  Hypertension for the past 10 years

Past surgical history:  None

Medications:
Ibuprofen 600 mg as needed
Hydrochlorothiazide 25 mg a day

Allergies:  None

Family history:  Father has hypertension and mother has pseudogout.  You have no
siblings.

Social history:  You live with your wife and work as a librarian.  You do not smoke but
drink beer occasionally.  You have not used illicit drugs and have no tattoos.

Physical examination:
The following are your temperature (T), blood pressure (BP), heart rate (HR), and
respiratory rate (RR).

Vital signs:  T 101.0 F (38.3 C), BP 130/60 mm Hg, HR 80/min, RR 18/min

HEENT:  PERRLA, EOMI, no conjunctival hemorrhage

Heart:  Regular rate and rhythm without murmurs, gallops, or rubs

Musculoskeletal:  Right knee is tender to palpation.  ROM is decreased.  There is no
redness or warmth.  There are no other joint deformities.

Skin:  No rashes or lesions

The doctor needs to do the above minimum examinations.

This copy of the material is licensed to Mohammed  Mirza.
Copyright © 2015. All Rights Reserved http://www.uworld.com
Case28 Pt Notes  Print

Clinical Skills Evaluation

Case 28 Patient Notes

History:  Describe the history you just obtained from this patient.  Include only
information (pertinent positives and negatives) relevant to this patient's problem(s).

53­YO man with 1 year of bilateral knee pain now with 2 days of worsening right
knee pain described as throbbing and constant
Pain is 7/10 and no relief with ibuprofen
Has 15­20 minutes of morning stiffness in multiple joints each day that resolves
spontaneously
No H/O trauma

ROS:  No fever, chills, nausea, vomiting, diarrhea, constipation, rashes, travel history,
sick contacts, insect bites, or urinary complaints
PMHx:  Hypertension for 10 years
PSHx:  None
Meds:  Hydrochlorothiazide 25 mg daily, ibuprofen 600 mg as needed
Allergies:  None
FHx:  Father has hypertension and mother has pseudogout.
SHx:  No smoking, occasional alcohol use

Physical examination:  Describe any positive and negative findings relevant to this
patient's problem(s).  Be careful to include only those parts of examination performed in
this encounter.

Vital signs:  Temperature, 101.0 F (38.3 C); blood pressure, 130/60 mm Hg; heart
rate, 80/min; and respiratory rate, 18/min
HEENT:  PERRLA, EOMI, no conjunctival hemorrhage
Heart:  Regular rate and rhythm without murmurs, gallops, or rubs
Musculoskeletal:  Right knee is tender.  ROM is decreased.  There is no swelling or
warmth.  There are no other joint deformities.
Skin:  No rashes or lesions.

Data interpretation:  Based on what you have learned from the history and physical
examination, list up to 3 diagnoses that might explain this patient's complaint(s).  List
your diagnoses from most to least likely.  For some cases, fewer than 3 diagnoses will be
appropriate.  Then enter the positive or negative findings from the history and the
physical examination (if present) that support each diagnosis.  Finally, list initial
diagnostic studies (if any) you would order for each listed diagnosis (eg, restricted
physical examination maneuvers, laboratory tests, imaging, ECG)

Diagnosis #1:  Osteoarthritis flare

History finding(s) Physical examination finding(s)
History of chronic knee pain Decreased ROM in right knee
Acute worsening of right knee pain  
Diagnosis #2:  Septic joint

History finding(s) Physical examination finding(s)
Acute onset of right knee pain Fever
  Decreased ROM in right knee

Diagnosis #3:  Acute crystal arthritis (gout or pseudogout)

History finding(s) Physical examination finding(s)
History of diuretic use Fever
Acute­onset right knee pain Right knee with decreased ROM
Family history of pseudogout  

Diagnostic studies
Right knee arthrocentesis
Right knee x­ray
ESR
CBC with differential

This copy of the material is licensed to Mohammed  Mirza.
Copyright © 2015. All Rights Reserved http://www.uworld.com
case29 Scenario  Print

A 50­year­old man with blurred vision

Vitals:
BP: 160/90 mmHg
Temp: 98.0 F (36.7 C)
Pulse: 70/min
RR: 16/min

Make a mental checklist of DD for blurry vision:
Diabetes mellitus
Cataract
Hypertensive retinopathy
Glaucoma
Macular degeneration
Brain lesions
Hyperviscosity syndromes (polycythemia)
Illegal drugs
Temporal arteritis (usually starts unilateral)
Trauma or infections to the eye (if unilateral)

This copy of the material is licensed to Mohammed  Mirza.
Copyright © 2015. All Rights Reserved http://www.uworld.com
case29 SP  Print

If the doctor asks you any questions other than these, just say “no” or “things are
normal in my daily life.”

You are a 50­year­old man complaining of blurry vision.

History of present illness:
You are a 50­year­old man who has had blurry vision in both eyes for 2 months.  You
have also had a 10­pound weight loss, increased urination, excessive thirst, and
increased appetite.  You can still see objects without any halos around them, but the
objects are becoming more blurry.  You bought over­the­counter reading glasses but still
have some difficulty seeing.  You have not seen a doctor for the past 10 years.

You have no nausea, vomiting, headache, arm/leg weakness, eye discharge, eye pain,
dizziness, loss of consciousness, or numbness or tingling in the extremities.

Past medical history:  None

Past surgical history:  None

Medications:  None

Allergies:  None

Family history:  Father has hypertension and mother has diabetes.  You have no siblings.

Social history:  You live with your wife and work as a truck driver.  You have smoked a
pack a day for 30 years and drink occasionally.  You have not used illicit drugs and have
no tattoos.

Physical examination:
The following are your temperature (T), blood pressure (BP), heart rate (HR), and
respiratory rate (RR).

Vital signs:  T 98.0 F (36.7 C), BP 160/90 mm Hg, HR 70/min, RR 16/min

HEENT:  PERRLA, EOMI, fundi without hemorrhages or AV nicking

Neck:  Supple without lymphadenopathy, thyromegaly, or bruits

Heart:  Regular rate and rhythm without murmurs, gallops, or rubs

Extremities:  Pulses 2+ in bilateral lower extremities

Neurologic:  Motor 5/5 bilaterally, sensory grossly intact bilaterally

The doctor needs to do the above minimum examinations.

This copy of the material is licensed to Mohammed  Mirza.
Copyright © 2015. All Rights Reserved http://www.uworld.com
case29 Pt Notes  Print

Clinical Skills Evaluation

Case 29 Patient Notes

History:  Describe the history you just obtained from this patient.  Include only
information (pertinent positives and negatives) relevant to this patient’s problem(s).

50­YO man with 2 months of blurry vision, polyuria, polydipsia, polyphagia, and 10­
pound weight loss
Has not seen a doctor in 10 years
Objects blurry without complete loss of vision or halos around light

ROS:  No nausea, vomiting, headache, arm/leg weakness, eye discharge, eye pain,
dizziness, loss of consciousness, or numbness or tingling in the extremities
PMHx:  None
PSHx:  None
Meds:  None
Allergies:  None
FHx:  Father has hypertension and mother has diabetes.
SHx:  1 PPD smoker for past 30 years, occasional alcohol use

Physical examination:  Describe any positive and negative findings relevant to this
patient’s problem(s).  Be careful to include only those parts of examination performed in
this encounter.

Vital signs:  Temperature, 98.0 F (36.7 C); blood pressure, 160/90 mm Hg; heart
rate, 70/min; and respiratory rate, 16/min
HEENT:  PERRLA, EOMI, fundi without hemorrhages or AV nicking
Neck:  Supple without lymphadenopathy, thyromegaly, or bruits
Heart:  Regular rate and rhythm without murmurs, gallops, or rubs
Extremities:  Pulses 2+ in bilateral lower extremities
Neurologic:  Motor 5/5 bilaterally, sensory grossly intact bilaterally

Data interpretation:  Based on what you have learned from the history and physical
examination, list up to 3 diagnoses that might explain this patient’s complaint(s).  List
your diagnoses from most to least likely.  For some cases, fewer than 3 diagnoses will be
appropriate.  Then enter the positive or negative findings from the history and the
physical examination (if present) that support each diagnosis.  Finally, list initial
diagnostic studies (if any) you would order for each listed diagnosis (eg, restricted
physical examination maneuvers, laboratory tests, imaging, ECG).

Diagnosis #1:  Diabetic retinopathy

History finding(s) Physical examination finding(s)
Polyuria,  polydipsia,  polyphagia  
Weight loss  
Blurry vision  
Diagnosis #2:  Hypertensive retinopathy

History finding(s) Physical examination finding(s)
Blurry vision BP 160/90 mm Hg

Diagnosis #3:  Glaucoma

History finding(s) Physical examination finding(s)
Decreased vision  

Diagnostic studies
Fasting blood sugar and hemoglobin A1C
Eye examination to measure pressure
Lipid profile
CBC with differential

This copy of the material is licensed to Mohammed  Mirza.
Copyright © 2015. All Rights Reserved http://www.uworld.com
case30 Scenario  Print

32­year­old female with multiple bruises

 Vitals:
PR: 90/min
BP: 120/80 mmHg
RR: 16/min
Temp: 99.4 F (37.4 C)

Mental Checklist of DD:
Accident
Physical assault
Spousal abuse
Bleeding disorders
Collagen vascular disorders

This copy of the material is licensed to Mohammed  Mirza.
Copyright © 2015. All Rights Reserved http://www.uworld.com
case30 SP  Print

*If the doctor asks you anything other than these, just say 'no,' or say things
that are normal in daily routine life.

You are a 32­year­old woman
You have a bruise on the right arm between your shoulder and your elbow
You are accompanied by your husband
When the doctor asks you how you sustained the injury, you tell him that, "My
husband told me that I fell down the stairs."
On further questioning by the doctor, you say that you have been hit by your
husband
You have been married for 7 years
Your husband is a truck driver
Your husband hits you whenever he has his rage episodes ­ usually once a week
You have 2 children, a boy (age 6) and a girl (age 5). He does not hit them. He
loves them, but they are afraid to go near him when he has his rage episodes.
Your husband is an alcoholic, and he almost always has a bottle of bourbon by his
side
Both your parents are living in the same town as you are and neither of them is
aware of the abuse that you are subject to
You feel that your husband loves you
You love your husband, but you are always on the edge when he is around
You feel that it is very difficult for you to leave him
You have never reported the matter to any agency
You do not feel safe at home, especially when he is around
You have felt at least on two occasions that he might kill you
You do not have an emergency plan to leave home if the need arises
You do not wish the matter to be reported to the authorities
You have a satisfying sexual relationship with him, and you are monogamous
You do not smoke, drink, or use recreational drugs
There is a shotgun at your house. You think your husband might use it.
When the doctor persuades you that you need not endure such a relationship in
which you are always in mortal fear, you say that you are going to think about
reporting it to the social welfare agencies
If you have been persuaded enough by the doctor, take his phone number and tell
him that you are going to call him if the need arises.

This copy of the material is licensed to Mohammed  Mirza.
Copyright © 2015. All Rights Reserved http://www.uworld.com
case31 Scenario  Print

A 20­year­old woman with burning urination

Vitals:
PR: 82/min
BP: 110/80 mmHg
RR: 16/min
Temp: 101.0 F (38.3 C)

 Mental Checklist of DD:
Cystitis
Pyelonephritis
Urethritis
Vulvovaginitis
Pelvic inflammatory disease 
Noninflammatory dysuria (trauma, irritant, allergy)

This copy of the material is licensed to Mohammed  Mirza.
Copyright © 2015. All Rights Reserved http://www.uworld.com
case31 SP  Print

If the doctor asks you any questions other than these, just say “no” or “things are
normal in my daily life.”

You are a 20­year­old woman who complains of burning with urination.

History of present illness:
You are a 20­year­old woman who comes to the physician with 4 days of burning with
urination.  You were well before but then had onset of fever to 38.3 C (101 F), chills,
rigors, and burning with urination.  You are urinating 10­12 times a day, sometimes with
increased urgency and little urine.  There is also a dull intermittent pain in the lower
pelvic area, greenish vaginal discharge, and occasional blood in the urine.  You had a
similar episode 1 year ago diagnosed as chlamydia and treated with doxycycline.  Your
last menstrual period was 2 weeks ago, and you have had a new sexual partner for the
past 2 months.  You do not use condoms and have no pain during intercourse.

You have no back pain, nausea, vaginal bleeding, vomiting, diarrhea, or constipation.

Past medical history:  None

Past surgical history:  None

Medications:  Birth control pills

Allergies:  None

Family history:  Father and mother are healthy.  You have no siblings.

Social history:  You live alone and are a college student.  You do not smoke but do drink
occasionally.  You have not used illicit drugs and have no tattoos.

Physical examination:
The following are your temperature (T), blood pressure (BP), heart rate (HR), and
respiratory rate (RR).

Vital signs:  T 101 F (38.3 C), BP 110/80 mm Hg, HR 82/min, RR 16/min

Abdomen:  Mild suprapubic discomfort with deep palpation, nondistended, normoactive
bowel sounds, no CVA tenderness

The doctor needs to do the above minimum examinations.

This copy of the material is licensed to Mohammed  Mirza.
Copyright © 2015. All Rights Reserved http://www.uworld.com
Case31 Pt Notes  Print

Clinical Skills Evaluation

Case 31 Patient Notes

History:  Describe the history you just obtained from this patient.  Include only
information (pertinent positives and negatives) relevant to this patient's problem(s).

20­YO woman with 4 days of dysuria, increased urinary frequency and urgency,
fever, chills, and episode of hematuria
New sexual partner 2 months ago without condom use
Treated for chlamydia cervicitis 1 year ago with similar symptoms
Intermittent suprapubic pain with green vaginal discharge

ROS:  No back pain, nausea, vaginal bleeding, pain with intercourse, vomiting, diarrhea,
or constipation.
PMHx:  None
PSHx:  None
Meds:  Birth control pills
Allergies:  None
FHx:  Father and mother are healthy.
SHx:  No smoking, occasional alcohol use

Physical examination:  Describe any positive and negative findings relevant to this
patient's problem(s).  Be careful to include only those parts of examination performed in
this encounter.

Vital signs:  Temperature, 101 F (38.3 C); blood pressure, 110/80 mm Hg; heart
rate, 82/min; and respiratory rate, 16/min
Abdomen:  Mild suprapubic discomfort with deep palpation, nondistended,
normoactive bowel sounds, no CVA tenderness

Data interpretation:  Based on what you have learned from the history and physical
examination, list up to 3 diagnoses that might explain this patient's complaint(s).  List
your diagnoses from most to least likely.  For some cases, fewer than 3 diagnoses will be
appropriate.  Then enter the positive or negative findings from the history and the
physical examination (if present) that support each diagnosis.  Finally, list initial
diagnostic studies (if any) you would order for each listed diagnosis (eg, restricted
physical examination maneuvers, laboratory tests, imaging, ECG)

Diagnosis #1:  Cystitis

History finding(s) Physical examination finding(s)
Increased urinary frequency and urgency Suprapubic discomfort
Fever, chills Fever
Hematuria, dysuria  

Diagnosis #2:  Pyelonephritis
History finding(s) Physical examination finding(s)
Increased urinary frequency and urgency Fever
Fever, chills  
Hematuria, dysuria  

Diagnosis #3:  Cervicitis

History finding(s) Physical examination finding(s)
Fever Fever
Vaginal discharge  
New sexual partner with no condom use  
History of previous cervicitis  

Diagnostic studies
Pelvic examination
Nucleic acid amplification tests for Chlamydia
and Gonorrhea
Urinalysis
Urine culture
CBC with differential

This copy of the material is licensed to Mohammed  Mirza.
Copyright © 2015. All Rights Reserved http://www.uworld.com
case32 Scenario  Print

A 50­year­old male with difficulty swallowing

 Vitals: 
BP: 130/90 mmHg
PR: 85/min
Temp: 98 F (36.7 C)
RR: 16/min

Make a mental checklist of DD:
Oropharyngeal dysphagia:
Neuromuscular (CVA, Parkinsonism, multiple sclerosis)
Mechanical obstruction (Zenker diverticulum, thyromegaly)
Skeletal muscle disorders (myasthenia gravis, muscular dystrophies, polymyositis)
Miscellaneous (medications, radiation)

Esophageal dysphagia:
Mechanical obstruction [carcinoma esophagus, benign strictures, webs and rings
(Schatzki)]
Achalasia cardia (achalasia, scleroderma)
Gastroesophageal reflux disease
Miscellaneous (diabetes, alcoholism)

This copy of the material is licensed to Mohammed  Mirza.
Copyright © 2015. All Rights Reserved http://www.uworld.com
case32 SP  Print

If the doctor asks you any questions other than these, just say “no” or “things are
normal in my daily life.”

You are a 50­year­old man complaining of difficulty swallowing.

History of present illness:
You are a 50­year­old man who has had difficulty swallowing for 3 months.  You were
previously well but began having problems swallowing solids.  Food would get stuck in
the middle of your chest before slowly going down.  This has progressed slowly and you
have had difficulty swallowing liquids for the past 3 weeks.  You could previously gulp
water but are unable to do so now.  Food also regurgitates into your chest 2­3 hours
after eating.  You have no problem chewing or transferring food out of your mouth into
your throat.  You have lost nearly 10 pounds (4.53 kg) in the past 3 months and have had
a decreased appetite for the past 3 weeks.

You have no weakness in the arms or legs, shortness of breath, nausea, vomiting, chest
pain, diarrhea, constipation, or urinary problems.

Past medical history:  Gastroesophageal reflux disease ­ you have had symptoms 2­3
times a week in the past relieved with antacids for the past 25 years.

Past surgical history:  None

Medications:  Over­the­counter antacids as needed

Allergies:  None

Family history:  Father and mother are healthy.  You have 2 healthy siblings.

Social history:  You live with your wife and are a stock broker.  You have smoked 1
pack a day for the past 30 years and drink wine occasionally.  You have not used illicit
drugs and have no tattoos.

Physical examination:
The following are your temperature (T), blood pressure (BP), heart rate (HR), and
respiratory rate (RR).

Vital signs:  T 98 F (36.7 C), BP 130/90 mm Hg, HR 85/min, RR 16/min

HEENT:  Oropharynx clear, difficulty swallowing water

Neck:  Supple without lymphadenopathy

Lymph nodes:  No axillary or supraclavicular adenopathy

Lungs:  Clear to auscultation bilaterally

Heart:  Regular rate and rhythm without murmurs, gallops, or rubs

Abdomen:  Nontender, nondistended, normoactive bowel sounds, no
hepatosplenomegaly, no CVA tenderness

The doctor needs to do the above minimum examinations.

This copy of the material is licensed to Mohammed  Mirza.
Copyright © 2015. All Rights Reserved http://www.uworld.com
case32 Pt Notes  Print

Clinical Skills Evaluation

Case 32 Patient Notes

History:  Describe the history you just obtained from this patient.  Include only
information (pertinent positives and negatives) relevant to this patient's problem(s).

50­YO man with 3 months of dysphagia initially with solids and now with liquids for
the past 3 weeks
No problem with chewing and transferring food to throat, but feels food getting
stuck in middle of chest
Decreased appetite and 10­pound (4.53­kg) weight loss
Food regurgitates 2­3 hours after eating

ROS:  No weakness in the arms or legs, shortness of breath, nausea, vomiting, chest
pain, diarrhea, constipation, or urinary problems
PMHx:  GERD for 25 years relieved with OTC antacids
PSHx:  None
Meds:  OTC antacids
Allergies:  None
FHx:  Father, mother, and 2 siblings are healthy.
SHx:  1 PPD smoker for 30 years, occasional alcohol use

Physical examination:  Describe any positive and negative findings relevant to this
patient's problem(s).  Be careful to include only those parts of examination performed in
this encounter.

Vital signs:  Temperature, 98 F (36.7 C); blood pressure,130/90 mm Hg; heart
rate, 85/min; and respiratory rate, 16/min
HEENT:  Oropharynx clear; patient has difficulty swallowing water
Neck:  Supple without lymphadenopathy
Lymph nodes:  No axillary or supraclavicular adenopathy
Lungs:  Clear to auscultation bilaterally
Heart:  Regular rate and rhythm without murmurs, gallops, or rubs
Abdomen:  Nontender, nondistended, normoactive bowel sounds, no
hepatosplenomegaly, no CVA tenderness

Data interpretation:  Based on what you have learned from the history and physical
examination, list up to 3 diagnoses that might explain this patient's complaint(s).  List
your diagnoses from most to least likely.  For some cases, fewer than 3 diagnoses will be
appropriate.  Then enter the positive or negative findings from the history and the
physical examination (if present) that support each diagnosis.  Finally, list initial
diagnostic studies (if any) you would order for each listed diagnosis (eg, restricted
physical examination maneuvers, laboratory tests, imaging, ECG)

Diagnosis #1:  Esophageal cancer

History finding(s) Physical examination finding(s)
Dysphagia with solids and then liquids  
Weight loss with decreased appetite  
Smoking history  

Diagnosis #2:  Achalasia

History finding(s) Physical examination finding(s)
Dysphagia with solids and liquids  
Weight loss  

Diagnosis #3:  Reflux esophagitis/Stricture

History finding(s) Physical examination finding(s)
History of GERD  
Food regurgitates 2­3 hours after eating  
OTC antacid use  

Diagnostic studies
Chest x­ray
Barium swallow
Upper GI endoscopy (EGD)

This copy of the material is licensed to Mohammed  Mirza.
Copyright © 2015. All Rights Reserved http://www.uworld.com
case33 Scenario  Print

30 Yr. O/M came to HIV drug refill
Vitals

Pulse­78/min
B.P­120/75 mm of Hg
Temp­98.8F
R.rate­16/min

This copy of the material is licensed to Mohammed  Mirza.
Copyright © 2015. All Rights Reserved http://www.uworld.com
case33 SP  Print

If the doctor asks you any questions other than these, just say “no” or “things are
normal in my daily life.”

You are a 30­year­old man requesting a refill for your HIV medications.

History of present illness:
You are a 30­year­old man who comes to the physician for a refill of HIV medications. 
You were diagnosed 1 year ago with HIV and your partner at the time terminated the
relationship.  You initially were sad but have slowly recovered and currently have a
positive outlook.  You have been compliant with your medications for the past 6 months,
and your last CD4 count was 480 with an undetectable viral load 3 months ago.  You have
had normal appetite and no weight loss.

You have no numbness or tingling in the extremities, weakness, chest pain, shortness of
breath, abdominal pain, skin rashes, cough, diarrhea, constipation, genital lesions, fever,
chills, or night sweats.

Past medical history:  HIV screened 1 year ago by ELISA test and confirmed by Western
blot

Past surgical history:  None

Medications:  HAART medications

Allergies:  None

Family history:  Your father and mother are healthy.  You have 1 healthy sister.

Social history:  You live alone and work as a truck driver.  You have had multiple male
sexual partners in the past but have not had a relationship in the past year.  You do not
smoke or drink.  You have not used illicit drugs and have no tattoos.

Physical examination:
The following are your temperature (T), blood pressure (BP), heart rate (HR), and
respiratory rate (RR).

Vital signs:  T 98.8 F (37.1 C), BP 120/75 mm Hg, HR 78/min, RR 16/min

HEENT:  Oropharynx clear, fundi without papilledema or lesions

Neck:  Supple without lymphadenopathy

Lungs:  Clear to auscultation bilaterally

Heart:  Regular rate and rhythm without murmurs, gallops, or rubs

Abdomen:  Nontender, nondistended, normoactive bowel sounds, no
hepatosplenomegaly, no CVA tenderness

Extremities:  No rash or edema

The doctor needs to do the above minimum examinations.

This copy of the material is licensed to Mohammed  Mirza.
Copyright © 2015. All Rights Reserved http://www.uworld.com
Case33 Pt Notes  Print

Clinical Skills Evaluation

Case 33 Patient Notes

History:  Describe the history you just obtained from this patient.  Include only
information (pertinent positives and negatives) relevant to this patient's problem(s).

30­YO man for refill of HIV medications
Diagnosed 1 year ago and has no stable support system
Initially felt sad but now has positive outlook
Compliant with meds
No weight loss; appetite ­normal

ROS:  No numbness or tingling in extremities, weakness, chest pain, shortness of breath,
abdominal pain, skin rashes, cough, diarrhea, constipation, genital lesions, fever, chills,
or night sweats
PMHx:  HIV diagnosed 1 year ago­ last CD4 count 480 with undetectable viral load 3
months ago
PSHx:  None
Meds:  HAART therapy
Allergies:  None
FHx:  Father, mother, and 1 sister are healthy.
SHx:  No smoking or alcohol use

Physical examination:  Describe any positive and negative findings relevant to this
patient's problem(s).  Be careful to include only those parts of examination performed in
this encounter.

Vital signs:  Temperature, 98.8 F (37.1 C); blood pressure, 120/75 mm Hg; heart
rate, 78/min; and respiratory rate, 16/min
HEENT:  Oropharynx clear, fundi without papilledema or lesions
Neck:  Supple without lymphadenopathy
Lungs:  Clear to auscultation bilaterally
Heart:  Regular rate and rhythm without murmurs, gallops, or rubs
Abdomen:  Nontender, nondistended, normoactive bowel sounds, no
hepatosplenomegaly, no CVA tenderness
Extremities:  no rash or edema

Data interpretation:  Based on what you have learned from the history and physical
examination, list up to 3 diagnoses that might explain this patient's complaint(s).  List
your diagnoses from most to least likely.  For some cases, fewer than 3 diagnoses will be
appropriate.  Then enter the positive or negative findings from the history and the
physical examination (if present) that support each diagnosis.  Finally, list initial
diagnostic studies (if any) you would order for each listed diagnosis (eg, restricted
physical examination maneuvers, laboratory tests, imaging, ECG)

Diagnosis #1:  HIV medication refill

History finding(s) Physical examination finding(s)
HIV diagnosis 1 year ago Normal examination findings
Recent stable CD4 and viral load  
Compliant with medications  

Diagnostic studies
CD4 count and viral load
CBC with differential
Liver function tests

This copy of the material is licensed to Mohammed  Mirza.
Copyright © 2015. All Rights Reserved http://www.uworld.com
case34 Scenario  Print

16­year­old female with amenorrhea

Vitals:
PR: 76/min
BP: 120/70 mmHg
RR: 16/min
Temp: 98.0 F (36.7 C)

 Mental checklist of DD:
Primary amenorrhea
Secondary amenorrhea
Pregnancy
Anorexia nervosa
Hyperprolactinemia
Thyroid dysfunction
Polycystic ovarian syndrome
Stress
Post pill amenorrhea
Hypothalamic pituitary ovarian axis problems

 
Note: Both hypothyroidism and hyperthyroidism can present with amenorrhea.

This copy of the material is licensed to Mohammed  Mirza.
Copyright © 2015. All Rights Reserved http://www.uworld.com
case34 SP  Print

If the doctor asks you any questions other than these, just say “no” or “things are
normal in my daily life.”

You are a 16­year­old girl who complains of having no periods for 2 months.

History of present illness:
You are a 16­year­old girl who has had no menstrual period for 2 months.  You started
having periods at age 13.  They were regular and you had no missed periods until the
last 2 months.  Your cycle usually lasts 28­30 days with moderate bleeding for 3­4 days
requiring 3­4 fully soaked pads a day.  You have been under a lot of stress at school for
the past 4 months and are currently studying for upcoming mid­term examinations in 2
weeks.  You have a good appetite but have unintentionally lost 10 pounds (4.53 kg) in the
past 4 months.  You are anxious during the interview.

You have no palpitations, diarrhea, constipation, hair loss, skin changes, breast
tenderness, nipple discharge, vaginal discharge, or urinary problems.

During the interview you ask “Do you think I'm pregnant?  I can't be pregnant right
now.  My parents will not be happy.”

Past medical history:  G0P0

Past surgical history:  None

Medications:  None

Allergies:  None

Family history:  Father and mother are healthy. You have 1 healthy sister.

Social history:  You live with your parents and are currently a junior in high school.  You
are doing well in school academically and participate in many extracurricular activities. 
You have a boyfriend and have been sexually active for the past 6 months but don't
always use condoms.  You do not smoke or drink.  You have not used illicit drugs and
have no tattoos.

Physical examination:
The following are your temperature (T), blood pressure (BP), heart rate (HR), and
respiratory rate (RR).

Vital signs:  T 98 F (36.7 C), BP 120/70 mm Hg, HR 76/min, RR 16/min

HEENT:  PERRLA, EOMI, oropharynx clear

Neck:  Supple without lymphadenopathy or thyromegaly

Abdomen:  Nontender, nondistended, normoactive bowel sounds, no
hepatosplenomegaly

The doctor needs to do the above minimum examinations.

This copy of the material is licensed to Mohammed  Mirza.
Copyright © 2015. All Rights Reserved http://www.uworld.com
Case34 Pt Notes  Print

Clinical Skills Evaluation

Case 34 Patient Notes

History:  Describe the history you just obtained from this patient.  Include only
information (pertinent positives and negatives) relevant to this patient's problem(s).

16­YO girl presents with 2 months of amenorrhea
Menarche at age 13 with regular and no missed periods until 2 months ago
Sexually active with boyfriend and does not always use condoms
Increased stress at school
Unintentional 10­pound (4.53­kg) weight loss in past 4 months with good appetite

ROS:  No palpitations, diarrhea, constipation, hair loss, skin changes, breast tenderness,
nipple discharge, vaginal discharge, or urinary problems
PMHx:  G0P0
PSHx:  None
Meds:  None
Allergies:  None
FHx:  Father, mother, and 1 sister are healthy.
SHx:  No smoking or alcohol use

Physical examination:  Describe any positive and negative findings relevant to this
patient's problem(s).  Be careful to include only those parts of examination performed in
this encounter.

Vital signs:  Temperature, 98 F (36.7 C); blood pressure, 120/70 mm Hg; heart
rate, 76/min; and respiratory rate, 16/min
HEENT:  PERRLA, EOMI, oropharynx clear
Neck:  Supple without lymphadenopathy or thyromegaly
Abdomen:  Nontender, nondistended, normoactive bowel sounds, no
hepatosplenomegaly

Data interpretation:  Based on what you have learned from the history and physical
examination, list up to 3 diagnoses that might explain this patient's complaint(s).  List
your diagnoses from most to least likely.  For some cases, fewer than 3 diagnoses will be
appropriate.  Then enter the positive or negative findings from the history and the
physical examination (if present) that support each diagnosis.  Finally, list initial
diagnostic studies (if any) you would order for each listed diagnosis (eg, restricted
physical examination maneuvers, laboratory tests, imaging, ECG)

Diagnosis #1:  Pregnancy

History finding(s) Physical examination finding(s)
Amenorrhea  
Sexually active, sometimes not using condoms  

Diagnosis #2:  Hyperthyroidism
History finding(s) Physical examination finding(s)
Amenorrhea  
Increased anxiety  
Weight loss with good appetite  

Diagnosis #3:  Hyperprolactinemia

History finding(s) Physical examination finding(s)
Amenorrhea  

Diagnostic studies
Pregnancy test
TSH and T4
Pelvic and breast examination
Prolactin level

This copy of the material is licensed to Mohammed  Mirza.
Copyright © 2015. All Rights Reserved http://www.uworld.com
case35 Scenario  Print

35­year­old female with acute right lumbar and lower abdominal pain

Vitals
PR: 100/min
BP: 110/70 mmHg
Temp: 38.3 C (101 F)
RR: 16/min

 Mental Checklist of DD: 
Renal colic
Ovarian torsion
Urinary tract infection and pyelonephritis
Pelvic inflammatory disease
Mittelschmerz
Appendicitis
Threatened abortion
Ectopic pregnancy
Dysmenorrhea
Endometriosis
Fibroids

This copy of the material is licensed to Mohammed  Mirza.
Copyright © 2015. All Rights Reserved http://www.uworld.com
case35 SP  Print

If the doctor asks you any questions other than these, just say “no” or “things are
normal in my daily life.”

You are a 35­year­old woman complaining of low back and abdominal pain.

History of present illness:
You are a 35­year­old woman who has had 1 day of right low back and lower abdominal
pain.  You were previously well and then had sudden onset of sharp right lower back pain
of 7/10 severity that radiates to the pelvic and lower abdominal area.  The pain is
progressively worsening without relief and is unaffected by positional changes.  You also
have had fever, chills, nausea without vomiting, burning with urination for 2­3 days, and
occasional blood in the urine.  Your last menstrual period was 3 weeks ago.  You are
sexually active with your boyfriend but do not use condoms.

You have no vaginal discharge, chest pain, shortness of breath, diarrhea, or constipation.

At the end of interview you say “I am in a lot of pain.  Please make it stop.”

Past medical history:
G2P2
Pelvic inflammatory disease 2 years ago
UTI twice 2 years ago treated with antibiotics

Past surgical history:  None

Medications:  None

Allergies:  None

Family history:  Father and mother are healthy.  You have 1 healthy sister.

Social history:  You live alone and work as a bank teller.  You have smoked 1 pack a
day for the past 15 years and drink occasionally.  You have not used illicit drugs and
have no tattoos.

Physical examination:
The following are your temperature (T), blood pressure (BP), heart rate (HR), and
respiratory rate (RR).

Vital signs:  T 101 F (38.3 C), BP 110/70 mm Hg, HR 100/min, RR 16/min

Abdomen:  Diffuse abdominal discomfort without focal tenderness, nondistended,
normoactive bowel sounds, no hepatosplenomegaly, mild CVA tenderness on the right,
negative Psoas test

The doctor needs to do the above minimum examinations.

This copy of the material is licensed to Mohammed  Mirza.
Copyright © 2015. All Rights Reserved http://www.uworld.com
Case35 Pt Notes  Print

Clinical Skills Evaluation

Case 35 Patient Notes

History:  Describe the history you just obtained from this patient.  Include only
information (pertinent positives and negatives) relevant to this patient's problem(s).

35­YO woman presents with 1 day of sharp right flank pain radiating to the right
pelvis and lower abdomen
Pain is 7/10
Progressive, worsening pain without relief and not affected by position
Has dysuria, fever, chills, nausea, and occasional hematuria

ROS:  No vaginal discharge, chest pain, shortness of breath, diarrhea, or constipation
PMHx:  G2P2, PID 2 years ago, UTI 2 years ago
PSHx:  None
Meds:  None
Allergies:  None
FHx:  Father, mother, and 1 sister are healthy.
SHx:  1 PPD smoker for 15 years, occasional alcohol use

Physical examination:  Describe any positive and negative findings relevant to this
patient's problem(s).  Be careful to include only those parts of examination performed in
this encounter.

Vital signs:  Temperature, 101 F (38.3 C); blood pressure, 110/70 mm Hg; heart
rate, 100/min; and respiratory rate, 16/min
Abdomen:  Diffuse abdominal discomfort without focal tenderness, nondistended,
normoactive bowel sounds, no hepatosplenomegaly, mild CVA tenderness on the
right, negative Psoas test

Data interpretation:  Based on what you have learned from the history and physical
examination, list up to 3 diagnoses that might explain this patient's complaint(s).  List
your diagnoses from most to least likely.  For some cases, fewer than 3 diagnoses will be
appropriate.  Then enter the positive or negative findings from the history and the
physical examination (if present) that support each diagnosis.  Finally, list initial
diagnostic studies (if any) you would order for each listed diagnosis (eg, restricted
physical examination maneuvers, laboratory tests, imaging, ECG)

Diagnosis #1:  Nephrolithiasis with hydronephrosis

History finding(s) Physical examination finding(s)
Flank pain radiating to lower abdomen Fever, CVA tenderness
No comfort with any position change  

Diagnosis #2:  Pyelonephritis

History finding(s) Physical examination finding(s)
Dysuria CVA tenderness
Hematuria Fever
Fever  

Diagnosis #3:  Pelvic inflammatory disease

History finding(s) Physical examination finding(s)
Previous PID  
Sexually active without condom use  
Lower abdominal pain  

Diagnostic studies
Pregnancy test
Pelvic examination
CT of abdomen
Urinalysis and urine culture

This copy of the material is licensed to Mohammed  Mirza.
Copyright © 2015. All Rights Reserved http://www.uworld.com
case36 Scenario  Print

70­year­old male with insomnia

Vitals:
PR: 58/min
BP: 130/90 mmHg
RR: 16/min
Temp: 98 F (36.7 C)

Mental Checklist of DD:
Depression
Post­traumatic stress disorder
Anxiety disorder
Chronic pain syndromes
Drug induced
Age related sleep changes
Thyroid problems
Sleep apnea
Restless leg syndrome

This copy of the material is licensed to Mohammed  Mirza.
Copyright © 2015. All Rights Reserved http://www.uworld.com
case36 SP  Print

If the doctor asks you any questions other than these, just say “no” or “things are
normal in my daily life.”

You are a 70­year­old man complaining of insomnia.

History of present illness:
You are a 70­year­old man who comes to the physician with 2 months of insomnia.  You
were well until 3 months ago when you were admitted to the hospital with chest pain and
diagnosed with coronary artery disease.  You had a cardiac catheterization and
angioplasty and were placed on medications.  You were very anxious throughout the
hospitalization and were given lorazepam for anxiety.  You have since had difficulty
falling asleep and stay asleep for only 2­3 hours before getting up and pacing around. 
You have no nightmares and do not drink alcohol or caffeine before you go to bed.  You
feel very tired in the morning and throughout the day.  You have also had decreased
appetite and a 5­pound (2.27­kg) weight loss during this time.  Your son lives in town
and feels that you have become more isolated and not interested in any activities.

You have no chest pain, shortness of breath, voice changes, tremor, hair loss, diarrhea,
constipation, palpitations, dizziness, or recent trauma.

Past medical history:  Coronary artery disease

Past surgical history:  None

Medications:
Aspirin 325 mg daily
Metoprolol 50 mg twice daily
Nitroglycerin sublingual as needed

Allergies:  None

Family history:  Father died of heart attack at age 75.  Mother died of breast cancer at
age 68.  You have 1 healthy sister.

Social history:  You live alone since your wife's death 2 years ago and are a retired
accountant.  You have smoked 1 pack a day for the past 50 years and drink beer
occasionally.  You have not used illicit drugs and have no tattoos.

Physical examination:
The following are your temperature (T), blood pressure (BP), heart rate (HR), and
respiratory rate (RR).

Vital signs:  T 98 F (36.7 C), BP 130/90 mm Hg, HR 58/min, RR 16/min

HEENT:  Oropharynx clear

Neck:  Supple without lymphadenopathy

Lungs:  Clear to auscultation bilaterally

Heart:  Regular rate and rhythm without murmurs, gallops, or rubs

Abdomen:  Nontender, nondistended, normoactive bowel sounds, no
hepatosplenomegaly

Neurologic:  Motor 5/5 bilaterally; alert and oriented to person, place, and time; DTR 2+
bilaterally
The doctor needs to do the above minimum examinations.

This copy of the material is licensed to Mohammed  Mirza.
Copyright © 2015. All Rights Reserved http://www.uworld.com
Case36 Pt Notes  Print

Clinical Skills Evaluation

Case 36 Patient Notes

History:  Describe the history you just obtained from this patient.  Include only
information (pertinent positives and negatives) relevant to this patient's problem(s).

70­YO man with 2 months of insomnia after hospitalization for CAD with angioplasty
Has difficulty falling asleep and staying asleep
Decreased appetite and 5­pound (2.27­kg) weight loss
Family thinks the patient feels isolated and lost interest in activities

ROS:  Fatigue present. No chest pain, shortness of breath, voice changes, tremor, hair
loss, diarrhea, constipation, palpitations, dizziness, or recent trauma
PMHx:  CAD
PSHx:  None
Meds:  Aspirin, metoprolol, nitroglycerin as needed
Allergies:  None
FHx:  Father died of MI, mother died of breast cancer
SHx:  1 PPD smoker for 50 years, occasional alcohol use

Physical examination:  Describe any positive and negative findings relevant to this
patient's problem(s).  Be careful to include only those parts of examination performed in
this encounter.

Vital signs:  Temperature, 98 F (36.7 C); blood pressure, 130/90 mm Hg; heart
rate, 58/min; respiratory rate, 16/min
HEENT:  Oropharynx clear
Neck:  Supple without lymphadenopathy
Lungs:  Clear to auscultation bilaterally
Heart:  Regular rate and rhythm without murmurs, gallops, or rubs
Abdomen:  Nontender, nondistended, normoactive bowel sounds, no
hepatosplenomegaly
Neurologic:  Motor 5/5 bilaterally; alert and oriented to person, place, and time;
DTR 2+ bilaterally

Data interpretation:  Based on what you have learned from the history and physical
examination, list up to 3 diagnoses that might explain this patient's complaint(s).  List
your diagnoses from most to least likely.  For some cases, fewer than 3 diagnoses will be
appropriate.  Then enter the positive or negative findings from the history and the
physical examination (if present) that support each diagnosis.  Finally, list initial
diagnostic studies (if any) you would order for each listed diagnosis (eg, restricted
physical examination maneuvers, laboratory tests, imaging, ECG)

Diagnosis #1:  Anxiety disorder

History finding(s) Physical examination finding(s)
Recent hospitalization  
Difficulty falling asleep and staying asleep  
Previous anxiety requiring medications  

Diagnosis #2:  Depression

History finding(s) Physical examination finding(s)
Decreased appetite and weight loss  
Decreased interest in activities  
Insomnia; fatigue  

Diagnosis #3:  Drug­induced

History finding(s) Physical examination finding(s)
Recently started on metoprolol Bradycardia
Insomnia  

Diagnostic studies
Basic metabolic panel
CBC with differential
TSH

This copy of the material is licensed to Mohammed  Mirza.
Copyright © 2015. All Rights Reserved http://www.uworld.com
case37 Scenario  Print

65­year­old male patient with difficulty urinating

Vitals:
PR: 92/min, regular
BP: 130/80 mmHg
RR: 16/min
Temp: 99 F (37.2 C)

 Mental Checklist of DD:
Benign prostatic hyperplasia
Carcinoma of prostate
Stone in the urinary tract (obstructive)
Strictures of urethra
Carcinoma bladder
Sphincter dysfunction
Infection
Neurological dysfunction, like spinal cord trauma and diabetes
Drug induced (anticholinergics)

This copy of the material is licensed to Mohammed  Mirza.
Copyright © 2015. All Rights Reserved http://www.uworld.com
case37 SP  Print

If the doctor asks you any questions other than these, just say “no” or “things are
normal in my daily life.”

You are a 65­year­old man complaining of difficulty with urination.

History of present illness:
You are a 65­year­old man who has had difficulty urinating for 2 months.  You were
previously well and developed difficulty with initiating urination, decreased intermittent
urinary flow, increased straining, and increased urinary frequency during the day.  You
also are getting up 5­6 times at night to urinate and feel that your bladder does not
empty properly.  There is mild burning with urination and 1 episode of blood in the urine.
 You also have a decreased appetite and have lost 10 pounds (4.53 kg) over the last
year.

You have no increased urinary urgency, abdominal pain, flank pain, fever, chills,
diarrhea, constipation, leg weakness, or trauma.

Ask the doctor: “Do you think I have prostate cancer?”

Past medical history:  Diabetes mellitus for the past 10 years

Past surgical history:  None

Medications:  Metformin 500 mg daily

Allergies:  None

Family history:  Father died of prostate cancer at age 75.  Mother died of kidney
problems at age 78.  You have 1 healthy sister.

Social history:  You are an accountant and live with your wife.  You have never smoked
and have consumed 2 beers a day for the past 35 years.  You have not used illicit drugs
and have no tattoos.

Physical examination:
The following are your temperature (T), blood pressure (BP), heart rate (HR), and
respiratory rate (RR).

Vital signs:  T 99.0 F (37.2 C), BP 130/80 mm Hg, HR 92/min, RR 16/min

Abdomen:  Nontender, nondistended, normoactive bowel sounds, no
hepatosplenomegaly, no CVA tenderness

Neurologic:  Motor 5/5 bilaterally, DTR 2+ bilaterally

The doctor needs to do the above minimum examinations.

This copy of the material is licensed to Mohammed  Mirza.
Copyright © 2015. All Rights Reserved http://www.uworld.com
case37 Pt Notes  Print

Clinical Skills Evaluation

Case 37 Patient Notes

History:  Describe the history you just obtained from this patient.  Include only
information (pertinent positives and negatives) relevant to this patient’s problem(s).

65­YO man presents with 2 months of difficulty initiating urination, decreased
urinary flow, nocturia, increased urinary frequency, and incomplete emptying of
bladder
1 episode of hematuria
Decreased appetite and 10­lb (4.53­kg) weight loss over the past year

ROS:  No increased urinary urgency, abdominal pain, flank pain, fever, chills, diarrhea,
constipation, leg weakness, or trauma
PMHx:  Diabetes for past 10 years
PSHx:  None
Meds:  Metformin 500 mg a day
Allergies:  None
FHx:  Father died of prostate cancer, mother died of kidney problems
SHx:  Denies smoking; 2 beers daily for 35 years

Physical examination:  Describe any positive and negative findings relevant to this
patient’s problem(s).  Be careful to include only those parts of examination performed in
this encounter.

Vital signs:  Temperature, 99.0 F (37.2 C); blood pressure, 130/80 mm Hg; heart
rate, 92/min; and respiratory rate, 16/min
Abdomen:  Nontender, nondistended, normoactive bowel sounds, no
hepatosplenomegaly, no CVA tenderness
Neurologic:  Motor 5/5 bilaterally; DTR 2+ bilaterally

Data interpretation:  Based on what you have learned from the history and physical
examination, list up to 3 diagnoses that might explain this patient’s complaint(s).  List
your diagnoses from most to least likely.  For some cases, fewer than 3 diagnoses will be
appropriate.  Then enter the positive or negative findings from the history and the
physical examination (if present) that support each diagnosis.  Finally, list initial
diagnostic studies (if any) you would order for each listed diagnosis (eg, restricted
physical examination maneuvers, laboratory tests, imaging, ECG).

Diagnosis #1:  Benign prostatic hyperplasia

History finding(s) Physical examination finding(s)
Difficulty with urinary initiation  
Incomplete emptying of bladder  
Nocturia, decreased urinary flow  
Diagnosis #2:  Prostate cancer

History finding(s) Physical examination finding(s)
Family history of prostate cancer  
Decreased urinary flow, nocturia  
Weight loss  

Diagnosis #3:  Bladder cancer

History finding(s) Physical examination finding(s)
Gross hematuria  
Incomplete emptying of urine  
Weight loss  

Diagnostic studies
Rectal examination
Urinalysis with urine culture
PSA
Basic metabolic panel

This copy of the material is licensed to Mohammed  Mirza.
Copyright © 2015. All Rights Reserved http://www.uworld.com
case38 Scenario  Print

35­year­old female complains of breathlessness and anxiety
 

Vitals:
PR: 94/min, regular
BP: 130/80 mmHg
RR: 22/min
Temp: 97 F (36.1 C)

Mental Checklist of DD:
Anxiety secondary to medical condition, e.g., hyperthyroidism, arrhythmias,
pheochromocytoma
Substance abuse
Panic disorder
Generalized anxiety disorder
Adjustment disorder with anxious mood
Acute stress disorder or post traumatic stress disorder
Hypochondriasis
Malingering

This copy of the material is licensed to Mohammed  Mirza.
Copyright © 2015. All Rights Reserved http://www.uworld.com
case38 SP  Print

If the doctor asks you any questions other than these, just say “no” or “things are
normal in my daily life.”

You are a 35­year­old woman complaining of shortness of breath.

History of present illness:
You are a 35­year­old woman who has had shortness of breath for 3 months.  You
remember that you may have had slight problems previously, but the symptoms became
worse 3 months ago.  You develop episodes of shortness of breath lasting 30 minutes
associated with palpitations, sweating, and a feeling that you are going to die.  The
episodes occur about 2­3 times a week at any time but are worse in crowded places
outside the house.  The symptoms seem to improve with slow breathing and relaxation,
but you have stopped going to outdoor activities to prevent the episodes.  You have been
to the emergency department many times, but all the tests come back normal without
any clear diagnosis.  You also have occasional diarrhea alternating with constipation. 
You drink 1 cup of coffee a day.

You have no chest pain, headache, nausea, vomiting, tremors, neck swelling, or
abdominal pain.

Ask the doctor:  “Do you think that this is anxiety like my mother has?”

Past medical history:  None

Past surgical history:  None

Medications:  None

Allergies:  Penicillin causes a rash.

Family history:  Father is healthy.  Mother has generalized anxiety disorder.  You have 1
healthy sister.

Social history:  You are a homemaker and live with your husband and 2 children.  You
have never smoked and drink a glass of wine occasionally.  You used marijuana in
college but have not used any since.  You have no tattoos.

Physical examination:
The following are your temperature (T), blood pressure (BP), heart rate (HR), and
respiratory rate (RR).

Vital signs:  T 97.0 F (36.1 C), BP 130/80 mm Hg, HR 94/min, RR 22/min

Neck:  Supple without thyromegaly or lymphadenopathy

Lungs:  Clear to auscultation

Heart:  Regular rhythm without murmurs, rubs, or gallops

Neurologic:  No tremor in extremities

The doctor needs to do the above minimum examinations.

This copy of the material is licensed to Mohammed  Mirza.
Copyright © 2015. All Rights Reserved http://www.uworld.com
Case38 Pt Notes  Print

Clinical Skills Evaluation

Case 38 Patient Notes

History:  Describe the history you just obtained from this patient.  Include only
information (pertinent positives and negatives) relevant to this patient's problem(s).

35­YO woman presents with 3 months of episodic shortness of breath, palpitations,
diaphoresis, and feeling of impending death.
Episodes last 30 minutes and occur more frequently outside of house in crowded
places
Symptoms improve with slow breathing and relaxation
Has been to ED multiple times with normal investigation and no definitive diagnosis

ROS:  Occasional diarrhea with alternating constipation present.  No chest pain,
headache, nausea, vomiting, tremors, neck swelling, or abdominal pain
PMHx:  None
PSHx:  None
Meds:  None
Allergies:  Penicillin causes a rash.
FHx:  Father is healthy.  Mother has generalized anxiety disorder.
SHx:  Denies smoking; occasional glass of wine

Physical examination:  Describe any positive and negative findings relevant to this
patient's problem(s).  Be careful to include only those parts of examination performed in
this encounter.

Vital signs:  Temperature, 97.0 F (36.1 C); blood pressure, 130/80 mm Hg; heart
rate, 94/min; and respiratory rate, 22/min
Neck:  Supple without thyromegaly or lymphadenopathy
Lungs:  Clear to auscultation
Heart:  Regular rhythm without murmurs, rubs, or gallops
Neurologic:  No tremor in extremities

Data interpretation:  Based on what you have learned from the history and physical
examination, list up to 3 diagnoses that might explain this patient's complaint(s).  List
your diagnoses from most to least likely.  For some cases, fewer than 3 diagnoses will be
appropriate.  Then enter the positive or negative findings from the history and the
physical examination (if present) that support each diagnosis.  Finally, list initial
diagnostic studies (if any) you would order for each listed diagnosis (eg, restricted
physical examination maneuvers, laboratory tests, imaging, ECG)

Diagnosis #1:  Generalized anxiety with panic attacks

History finding(s) Physical examination finding(s)
Episodes of palpitations with dyspnea  
Family history of anxiety  
Symptoms worse in crowded places  
Symptoms relieved with slow breathing  

Diagnosis #2:  Hyperthyroidism

History finding(s) Physical examination finding(s)
Episodes of palpitations  
Shortness of breath and diaphoresis  

Diagnosis #3:  Cardiac arrhythmia

History finding(s) Physical examination finding(s)
Palpitations  
Shortness of breath and diaphoresis  

Diagnostic studies
EKG
TSH, T4
Event monitor

This copy of the material is licensed to Mohammed  Mirza.
Copyright © 2015. All Rights Reserved http://www.uworld.com
case39 Scenario  Print

53­year­old male with a long history of epigastric pain

Vitals:
PR: 84/min
BP: 120/70 mmHg
RR: 16/min
Temp: 97 F (36.1 C)

Mental Checklist of DD:
Peptic ulcer
Gastritis
Esophagitis (GERD)
Carcinoma of esophagus, stomach, and pancreas
Chronic pancreatitis
Cholecystitis
Hepatitis

This copy of the material is licensed to Mohammed  Mirza.
Copyright © 2015. All Rights Reserved http://www.uworld.com
case39 SP  Print

If the doctor asks you any questions other than these, just say “no” or “things are
normal in my daily life.”

You are a 53­year­old man complaining of abdominal pain.

History of present illness:
You are a 53­year­old man who presents with 2 years of abdominal pain.  You were
previously well and developed midepigastric pain that occurs on and off for months
before decreasing and then recurring.  The midepigastric pain is sharp, 7/10 intensity,
made worse by food, and is sometimes relieved with antacids.  The pain also occurs in
the middle of the night and sometimes radiates to your back.  You have also had
decreased appetite, the feeling of a full stomach, abdominal bloating, occasional black
stools, and a 15­pound (6.8­kg) weight loss in the past 6 months.

You have no jaundice, fever, chills, vomiting, shortness of breath, diarrhea, or
constipation.

Ask the doctor:  “Can you please stop this pain?  Is it curable?”

Past medical history:  Osteoarthritis of the knee for past 10 years

Past surgical history:  None

Medications:
Over­the­counter antacids as needed
Ibuprofen 600 mg 3 times a day as needed

Allergies:  None

Family history:  Father is healthy.  Mother died at age 60 from pancreatic cancer.  You
have 1 healthy brother.

Social history:  You are a stockbroker.  You live with your wife and have 2 children.  You
have never smoked but have consumed 2 beers a day for the past 25 years.  You have
never used illicit drugs and have no tattoos.

Physical examination:
The following are your temperature (T), blood pressure (BP), heart rate (HR), and
respiratory rate (RR).

Vital signs:  T 97.0 F (36.1 C), BP 120/70 mm Hg, HR 84/min, RR 16/min

Neck:  Supple without thyromegaly or lymphadenopathy

Abdomen:  Nontender, nondistended, normoactive bowel sounds throughout, no
hepatosplenomegaly, no bruits

The doctor needs to do the above minimum examinations.

This copy of the material is licensed to Mohammed  Mirza.
Copyright © 2015. All Rights Reserved http://www.uworld.com
Case39 Pt Notes  Print

Clinical Skills Evaluation

Case 39 Patient Notes

History:  Describe the history you just obtained from this patient.  Include only
information (pertinent positives and negatives) relevant to this patient's problem(s).

53­YO man presents with 2 years of episodic midepigastric pain worse with food
and sometimes relieved with antacids.
Pain also occurs at night and sometimes radiates to the back.
Pain is 7/10.
Has also noticed decreased appetite, the feeling of a full stomach, abdominal
bloating, occasional black stools, and a 15­pound (6.8­kg) weight loss in the past 6
months

ROS:  No jaundice, fever, chills, vomiting, shortness of breath, diarrhea, or constipation
PMHx:  Osteoarthritis of the knee for past 10 years
PSHx:  None
Meds:  Over­the­counter antacids as needed, ibuprofen 600 mg 3 times a day as needed
Allergies:  None
FHx:  Father healthy, mother died at age 60 of pancreatic cancer.
SHx:  Denies smoking; 2 beers a day for past 25 years

Physical examination:  Describe any positive and negative findings relevant to this
patient's problem(s).  Be careful to include only those parts of examination performed in
this encounter.

Vital signs:  Temperature, 97.0 F (36.1 C); blood pressure, 120/70 mm Hg; heart
rate, 84/min; and respiratory rate, 16/min
Neck:  Supple without thyromegaly or lymphadenopathy
Abdomen:  Nontender, nondistended, normoactive bowel sounds throughout, no
hepatosplenomegaly, no bruits

Data interpretation:  Based on what you have learned from the history and physical
examination, list up to 3 diagnoses that might explain this patient's complaint(s).  List
your diagnoses from most to least likely.  For some cases, fewer than 3 diagnoses will be
appropriate.  Then enter the positive or negative findings from the history and the
physical examination (if present) that support each diagnosis.  Finally, list initial
diagnostic studies (if any) you would order for each listed diagnosis (eg, restricted
physical examination maneuvers, laboratory tests, imaging, ECG)

Diagnosis #1:  Peptic ulcer disease

History finding(s) Physical examination finding(s)
Midepigastric pain  
Relieved with antacids  
History of NSAID use  
Diagnosis #2:  Chronic pancreatitis

History finding(s) Physical examination finding(s)
Chronic midepigastric pain  
Pain radiates to back  
History of alcohol use  

Diagnosis #3:  Gastric cancer

History finding(s) Physical examination finding(s)
Midepigastric pain increased with food  
Nighttime pain  
Weight loss  

Diagnostic studies
CBC with differential
Serum amylase and lipase
Upper GI endoscopy
Liver function tests

This copy of the material is licensed to Mohammed  Mirza.
Copyright © 2015. All Rights Reserved http://www.uworld.com
case40 Scenario  Print

45­year­old male complaining of bloody vomiting

Vitals:
PR: 90/min, regular
BP: 100/60 mmHg
RR: 18/min
T: 98.0 F (36.7 C)

Mental Checklist of DD:
Gastric ulcer
Duodenal ulcer
Esophageal and gastric varices
Mallory­Weiss tears
Gastritis
Erosive esophagitis
GI malignancy
Vascular ectasia

This copy of the material is licensed to Mohammed  Mirza.
Copyright © 2015. All Rights Reserved http://www.uworld.com
case40 SP  Print

If the doctor asks you any questions other than these, just say “no” or “things are
normal in my daily life.”

You are a 45­year­old man complaining of bloody vomiting.

History of present illness:
You are a 45­year­old man who has had 2 hours of bloody vomiting.  You were
previously well until 2 hours ago when you developed burning epigastric pain of 8/10
severity radiating to the back.  This was immediately followed by vomiting with a cupful
of bright red blood.  You had a second episode 30 minutes ago similar to the first and
decided to come to the emergency department.  You are also dizzy.

You have had 2 years of heartburn symptoms with mid­epigastric pain 3­4 times a week
after eating that lasts 10­15 minutes.  Your symptoms were relieved with antacids but
have been getting worse over the past 2 months.  The pain was also made worse with
drinking caffeine and alcohol.  You have had occasional black stools for 1 month.

You have no jaundice, fever, chills, shortness of breath, weight loss, urinary symptoms,
diarrhea, or constipation.

Ask the doctor:  “Will I die from this bleeding?  Is it cancer?”

Past medical history:
Hypertension
Tension headaches

Past surgical history:  None

Medications:
Hydrochlorothiazide 50 mg daily
Ibuprofen 400 mg 3 times a day as needed

Allergies:  None

Family history:  Father, mother, and 2 siblings are healthy.

Social history:  You are a sales manager at a marketing company and are under a lot of
stress.  You live with your wife and 2 children.  You have smoked 2 packs a day for the
past 25 years and have consumed 2 beers a day for the past 25 years.  You have never
used illicit drugs and have no tattoos.

Physical examination:
The following are your temperature (T), blood pressure (BP), heart rate (HR), and
respiratory rate (RR).

Vital signs:  T 98.0 F (36.7 C), BP 100/60 mm Hg, HR 90/min, RR 18/min

HEENT:  No jaundice or pallor, oropharynx clear

Neck:  Supple without thyromegaly or lymphadenopathy

Lungs:  Clear to auscultation bilaterally

Heart:  Regular rhythm and rate without murmurs, gallops, or rubs

Abdomen:  Nontender, nondistended, normoactive bowel sounds throughout, no
hepatosplenomegaly, no bruits
The doctor needs to do the above minimum examinations.

This copy of the material is licensed to Mohammed  Mirza.
Copyright © 2015. All Rights Reserved http://www.uworld.com
Case40 Pt Notes  Print

Clinical Skills Evaluation

Case 40 Patient Notes

History:  Describe the history you just obtained from this patient.  Include only
information (pertinent positives and negatives) relevant to this patient's problem(s).

45­YO man presents with acute onset of 2 episodes of hematemesis with a cupful of
bright red blood and dizziness.
Also has midepigastric pain radiating to the back
Has had 2 years of heartburn and chronic midepigastric pain after meals lasting 15­
20 minutes and relieved with antacids
Symptoms are worse with caffeine and alcohol intake
Occasional black stools in the past month

ROS:  No jaundice, fever, chills, shortness of breath, weight loss, urinary symptoms,
diarrhea, or constipation
PMHx:  HTN, tension headaches
PSHx:  None
Meds:  Hydrochlorothiazide 50 mg daily, ibuprofen 400 mg 3 times a day as needed
Allergies:  None
FHx:  Father, mother, and siblings are healthy.
SHx:  2 PPD smoker for 25 years; 2 beers a day for past 25 years

Physical examination:  Describe any positive and negative findings relevant to this
patient's problem(s).  Be careful to include only those parts of examination performed in
this encounter.

Vital signs:  Temperature, 98.0 F (36.7 C); blood pressure, 100/60 mm Hg; heart
rate, 90/min; and respiratory rate, 18/min
HEENT:  No jaundice or pallor, oropharynx clear
Neck:  Supple without thyromegaly or lymphadenopathy
Lungs:  Clear to auscultation bilaterally
Heart:  Regular rhythm and rate without murmurs, gallops, or rubs
Abdomen:  Nontender, nondistended, normoactive bowel sounds throughout, no
hepatosplenomegaly, no bruits

Data interpretation:  Based on what you have learned from the history and physical
examination, list up to 3 diagnoses that might explain this patient's complaint(s).  List
your diagnoses from most to least likely.  For some cases, fewer than 3 diagnoses will be
appropriate.  Then enter the positive or negative findings from the history and the
physical examination (if present) that support each diagnosis.  Finally, list initial
diagnostic studies (if any) you would order for each listed diagnosis (eg, restricted
physical examination maneuvers, laboratory tests, imaging, ECG)

Diagnosis #1:  Bleeding gastric ulcer

History finding(s) Physical examination finding(s)
Hematemesis Hypotension
Midepigastric pain  
Occasional black stools  
NSAID use  

Diagnosis #2:  Gastritis

History finding(s) Physical examination finding(s)
Hematemesis  
Midepigastric pain  
History of NSAID use  

Diagnosis #3:  Esophagitis

History finding(s) Physical examination finding(s)
History of heartburn  
Hematemesis  
Chronic tobacco/alcohol use  

Diagnostic studies
CBC with differential
Upper GI endoscopy
Coagulation studies
Basic metabolic panel

This copy of the material is licensed to Mohammed  Mirza.
Copyright © 2015. All Rights Reserved http://www.uworld.com
case41 Scenario  Print

60­year­old male complains of dizziness

Vitals:
PR: 80/min
BP: 140/90 mmHg
RR: 16/min
Temp: 97.0F(36.1)

Mental Checklist of DD:
TIA or stroke
Drug induced
Coronary artery disease
Autonomic dysfunction
Postural hypotension
Congestive heart failure
Arrhythmias
Hypoglycemia
Intracranial pathology
Ear problems
Anemia

This copy of the material is licensed to Mohammed  Mirza.
Copyright © 2015. All Rights Reserved http://www.uworld.com
case41 SP  Print

SP's Notes
*If the doctor asks you anything other than these, just say 'no,' or say things
that are normal in daily routine life.
You are Robert, 60­years­old
Having dizziness for past 4 days
Occurs occasionally
Lasts for 2­5 minutes
More on getting up from sitting position
Lying down will help reduce the dizziness
Sometimes associated with palpitations or sweating
No chest pain
Never lost consciousness during these episodes
This time noticed weakness in right lower leg and lasted for 10 minutes and
resolved spontaneously; no headache, nausea, or vomiting
No change in bowel or bladder habits

Other medical problems:
You have diabetes for 15 years and take glyburide twice daily; you check blood
sugar twice daily, and it is in the range of 120­160 mg/dL.
You have high BP from the past 10 years. One week back, medication for BP
control was changed from atenolol to terazosin (alfa blocker).
So far, no heart problems and never had any strokes.

Social history:
Smoker for 30 years, 2 packs/day
Not an alcoholic
Exercise regularly
Occupation ­ clerk in food store
Lives with wife; she is healthy

Medications:
Only terazosin and glyburide
Taking medication regularly, as prescribed

Allergies:
You have no known drug allergies

Ask this question, 'Doc, did I have stroke?'

This copy of the material is licensed to Mohammed  Mirza.
Copyright © 2015. All Rights Reserved http://www.uworld.com
case42 Scenario  Print

30­year­old male with new onset of seizure
 

Vitals:
PR: 82/min, regular
BP: 120/80 mmHg
RR: 18/min
T: 99.0 F (37.2 C)

Mental Checklist of DD:
Seizures (secondary to head trauma, infections, drugs, metabolic disorders)
Hypoglycemia
Syncope
Migraine
Stroke
Space occupying lesions
Alcohol or drug withdrawal

This copy of the material is licensed to Mohammed  Mirza.
Copyright © 2015. All Rights Reserved http://www.uworld.com
case42 SP  Print

*If the doctor asks you anything other than these, just say 'no,' or say things
that are normal in daily routine life.

You are Keith, age 30 years
Had an episode of seizure a few hours ago; the episode lasted around 3 minutes,
but you were unconscious probably about 20 minutes. You felt a little nauseous
before the onset of seizures, and then you don't know what happened, but your co­
workers told you that you had jerky movements for a couple of moments.
Did not pass urine or feces during the episode
You bit your tongue during the episode
You have been noticing some weakness in the right hand for the past 3 months
Once in a while, you get very mild headaches, but these days your headache is
constant and more in severity
Having mild fever, cold and flu­like symptoms for the past couple of days
No history of ear discharge or sinusitis
No pain in the neck
No history of head trauma

Other medical problems:
You have type­1 diabetes and have been on insulin for the past few years. You do
not think this is an episode of hypoglycemia, because you know how that looks
like.
No other medical problems

Social history:
Occasionally drinks alcohol. Last drink was 2 days ago.
No smoking history
Never used any illicit drugs
Occupation ­ Clerk in postal department

Family history:
There is no family history of seizures

Medications:
Insulin NPH type 10 units in AM and 8 units at PM. Usually checks blood sugar 2
times a day.

Allergies:
No known drug allergies
Ask this question, 'Doc, do I have a brain tumor?'

This copy of the material is licensed to Mohammed  Mirza.
Copyright © 2015. All Rights Reserved http://www.uworld.com
case43 Scenario  Print

A 23­year­old male with rectal bleeding

Vitals:
PR: 90/min, regular
BP: 110/60 mmHg
RR: 18/min
T: 101.0 F (38.3 C)

Differential diagnosis of lower GI bleed in an young patient:
Anal fistula/fissure
Inflammatory bowel disease
Infectious colitis
Neoplasm
Vascular ectasia
Gonococcal proctitis
Hemorrhoids

Differential diagnosis of lower GI bleed in an elderly patient:
Diverticulosis
Angiodysplasia
Malignancy/polyp
Ischemic colitis
Inflammatory bowel disease

This copy of the material is licensed to Mohammed  Mirza.
Copyright © 2015. All Rights Reserved http://www.uworld.com
case43 SP  Print

*If the doctor asks you anything other than these, just say 'no,' or say things
that are normal in daily routine life.
You are Steve, age 23 years
Having bleeding per rectum for past 3 days
Started mildly with blood streaks in stools for past one month; progressed over 3
days to frank blood in stools; never had black stools.
Associated with mild (2­3/10), crampy lower abdomen pain
Suffered from chronic constipation for past 5 years. Have to strain a lot while
defecating. Also, you use to have severe pain sometimes when defecating, but the
bowel movements increased in frequency to 3 times a day recently.
No nausea or vomiting
Having mild fever (100 F) without chills for the past 4­5 days
No recent change in weight or appetite
No similar episodes in the past. Never admitted in the hospital before.
Do not eat much vegetables or fruits
You are working at local restaurant. No illicit drug use.
Multiple female sexual partners; no homosexual activity; does not always use
condoms.
Not a smoker; not an alcoholic.
Father died from colon cancer at the age of 65
Not using any medications chronically
You have no known drug allergies

This copy of the material is licensed to Mohammed  Mirza.
Copyright © 2015. All Rights Reserved http://www.uworld.com

You might also like